USMLE RX Questions 1 and 2

अब Quizwiz के साथ अपने होमवर्क और परीक्षाओं को एस करें!

brachial pouch mnemonic?

1- ear 2- tonsils 3 - dorsal - bottom - inferior PTH 3 - ventral -to- (Thymus 4 - top (superior PTH) ear tonsils bottom to top

what translocation gives a better prognosis of ALL?

12:21

what chromosome is there trisomy of in Patau? what are symptoms?

13 cleft palate polydactyly (more fingerS) small eye

which chromosome is p53 located on?

17

which CAH causes delayed puberty and HTN?

17 OH deficiency

which classes of antiarrhythmics cause torsades de pointes?

1A 111

which antiarrhythmics block potassium channels?

1A 3

what type of hypersensitivity reaction is a transfusion reaction?

2

what is hemoglobin normally made of?

2 alpha chains and 2 beta chains

what is the typical txtment for HAART?

2 nucleoside/tide inhibitors (tenofovir and emtricitabine plus 1 of the following: pharmacokinetic enhancing protease inhibitors or integrase inhibitors

when do post partum blues resolve?

2 weeks

which familial dyslipidemias are AD?

2- hypercholesterolemia 4- hypertriglyceridemia

amiodarone is what antiarrhythmic class of drug? what are side effects?

3 (K channel blockers) pulm fibrosis hepatotoxicity thyorid abnormalitis photodermatitis

A 52-year-old Asian-American man comes for evaluation because of difficulty hearing in his left ear. He has a history of recurrent ear infections over the past year. For the past 2 months, he has had several episodes of random nosebleeds and reports that his nose feels "all stuffed up." Sometimes, he finds himself unable to breathe through his nose and has had to breathe through his mouth, causing his mouth to dry. He says that he has had a sore throat for but has no difficulty swallowing. He reports losing 9.1 kg (20 lb) of weight during this time despite keeping the same diet. He states he feels warm but has not checked his temperature. He has smoked half a pack of cigarettes daily for the past 25 years. He takes no medications. Temperature is 38.1° C (100.6° F), blood pressure is 118/76 mm Hg, pulse is 71/min, and respiratory rate is 12/min. He is 5'8" tall and weighs 61.2 kg (135 lb); BMI is 20.3 kg/m2. There is a painless cervical lymphadenopathy on inspection during physical examination. Weber test lateralizes to the left ear, and Rinne test shows that his bone conduction is greater than air in the left ear. There is a mass visualized within the nasal cavity which is biopsied, and the results are pending. Which of the following is the most likely underlying cause of this patient's condition? A.Chronic sinusitis B.Epstein-Barr virus C.Human papillomavirus D.Occupational pollutant E.Smoking history F.Viral upper respiratory infection

B.Epstein-Barr virus This patient presents with weight loss, fatigue, low-grade fever, and a mass with non-tender lymphadenopathy highly suggestive of malignancy. Considering these symptoms are chronic, lasting 2 months in this case, there is a strong indication for malignancy. The medical history of recurrent ear infections within the past year is suspect for obstruction of the eustachian tube drainage. There are also localizing symptoms to the nasopharyngeal region in the form of recurrent nosebleeds and nasal blockage. There is a strong indication that this patient's diagnosis is nasopharyngeal carcinoma. Risk factors for nasopharyngeal carcinoma include male gender, Asian ethnicity, and diets consisting of high salt-cured meats and fish. Among Asian patient populations, Epstein-Barr virus (EBV) is associated with nasopharyngeal carcinoma. Other malignancies caused by EBV include gastric carcinoma, Burkitt lymphoma seen in endemic areas, and lymphoproliferative diseases in immunocompromised patients. About 90% of the population is suspected to have a latent EBV, making the transformation to malignancy later in life a possibility. EBV is herpesvirus 4 (HHV-4) is also a common cause of mononucleosis which typically presents with hepatosplenomegaly, pharyngitis and lymphadenopathy most prominent in the posterior cervical nodes. A high yield point is these patients are advised to avoid contact sports to prevent splenic rupture.

A medical student in histology class is studying the peripheral nervous system. While viewing a specimen under high magnification, she notices that the neurons have interruptions in the myelinsheath at regular intervals along the length of their axons. Which of the following is the primary function of the gaps described? A.Sites of insulation B.Sites of ion exchange C.Sites of low membrane capacitance D.Sites of neurotransmitter release E.Sites of phagocytosis

B.Sites of ion exchange Here the current activates voltage-gated sodium channels in the membrane, allowing ions to flow into the cytoplasm and boost the propagating depolarizing signal. The nodes of Ranvier are sites of high membrane capacitance relative to the myelinated axon segments. The higher membrane capacitance facilitates rapid action potential propagation.

Which of the following cytokines would this patient most likely be treated with to prevent mutiple sclerosis progression? A.α-Interferon B.β-Interferon C.γ-Interferon D.Interleukin-11 E.Interleukin-6

B.β-Interferon γ-Interferon is used in patients with chronic granulomatous disease. α-Interferon is used in the treatment of hepatitis B and C, Kaposi sarcoma, malignant melanoma, and certain leukemias. Interleukin-11 is used in the treatment of thrombocytopenia. Interleukin-6 is used in the production of acute phase reactants; although not used directly, inhibitors of IL-6 are use for the treatment of Rheumatoid Arthritis

what is the defect in Bruton X-linked agammaglobulinemia?

BTK, a tyrosine kinase

A 6-year-old boy is being evaluated for intellectual disability and obesity. The father mentions that his son is always eating and is constantly hungry. He states, "He never seems to be full, no matter how much we feed him." The patient's family history is significant for hyperthyroidism in his mother. The patient has been relatively healthy his entire life. Physical examination shows diffuse hypotonia of the muscles. The disorder that this patient likely has is caused by which of the following? A. Anti-thyroid-stimulating hormone antibodies B. Lesion in the amygdala C. Loss of function of a gene inherited from the father D. Loss of function of a gene inherited from the mother E. Tumor in the ventromedial area of the brain

C. Loss of function of a gene inherited from the father This patient likely has Prader-Willi syndrome, which is suspected based on his obese body habitus, intellectual disability, muscle hypotonia, and hyperphagia (constant and excessive hunger). In Prader-Willi syndrome, the paternal copy of the gene, located on chromosome 15q11.2-13, is deleted or mutated. Lack of expression because of the deletion or mutation leads to the development of the characteristic features of this disease: hyperphagia, obesity, intellectual disability, hypogonadism, and hypotonia.Imprinting occurs when one allele of a gene is active, while the other is inactive, usually through DNA methylation. DNA methylation involves the addition of methyl groups to a region of DNA to silence the expression of the gene. In Prader-Willi syndrome, the maternal gene is inactive or silent, while the paternal gene, which should be active, is deleted, sequentially resulting in the disease. Imprinting occurs when one allele of a gene is active, while the other is inactive, usually through DNA methylation. DNA methylation involves the addition of methyl groups to a region of DNA to silence the expression of the gene. In Prader-Willi syndrome, the maternal gene is inactive or silent, while the paternal gene, which should be active, is deleted, sequentially resulting in the disease. Angelman syndrome is caused by a mutation of the maternal gene, not the paternal gene. Patients exhibit inappropriate laughter and experience seizures, ataxia, and severe intellectual disability. This diagnosis is unlikely because the patient does not have these symptoms. Klüver-Bucy syndrome is caused by bilateral lesions in the amygdala. Patients have disinhibited behavior, such as hypersexuality, hyperphagia, and hyperorality. Although it is rare in children, Klüver-Bucy syndrome can occur secondary to herpes encephalitis. This patient has been healthy his entire life, making Klüver-Bucy syndrome unlikely. Anti-thyroid-stimulating hormone antibodies are involved in the mechanism behind Graves disease, the most common cause of hyperthyroidism. Neonatal thyrotoxicosis can occur as a result of placental transfer of thyroid-stimulating antibodies in mothers with hyperthyroidism during pregnancy; however, this patient's lack of autonomic symptoms and his obese body habitus are not consistent with a diagnosis of hyperthyroidism. A tumor in the ventromedial area within the hypothalamus of the brain would disrupt the satiety function of the brain, resulting in constant hyperphagia and a severely obese body habitus. Patients may also experience bitemporal hemianopia, or partial blindness of the outer half of both the right and left visual fields, because of compression of the optic chiasma by the tumor. The lack of symptoms of bitemporal hemianopia in this patient make the presence of a brain tumor, such as a craniopharyngioma, unlikely.

Two most common causes of pelvic inflammatory disease?

C. tracomatis N. gonorrhea

Which of the following cell types has most likely become infected in an HIV patient's central nervous system? A.Astrocytes B.Langerhans cells C.Microglia D.Neurons E.Oligodendrocytes

C.Microglia HIV encephalopathy (HIV-associated dementia). HIV encephalopathy is classically seen in patients with a long history of poorly controlled HIV infection with CD4+ counts below 200. CSF analysis would typically reveal a high HIV viral load, and brain imaging scans might show nonspecific, diffuse cortical and subcortical atrophy. HIV encephalopathy is caused by infection of macrophages and microglia in the brain.

Patient, who recently started an antibiotic, has a new-onset rash, fever, and elevated creatinine with WBC casts detected on urinalysis. Which of the following medications was this patient most likely given? A.Aztreonam B.Clindamycin C.Penicillin V D.Tetracycline E.Vancomycin

C.Penicillin V This overall presentation is consistent with acute interstitial nephritis (AIN). AIN is most commonly caused by medications, including antibiotics. Penicillins, cephalosporins, trimethoprim-sulfamethoxazole, and ciprofloxacin are all associated with AIN. Of these, penicillin V is the only available answer selection. Other common adverse effects of penicillin use include interstitial nephritis, eosinophilia, and hemolytic anemia.

Which of the following autoantibodies is most likely responsible for Graves disease? A.Antithyrotropin antibodies B.Follicle stimulating hormone (FSH) receptor antibodies C.Thyrotropin receptor stimulatory antibodies D.Thyrotropin-releasing hormone receptor inhibitory antibodies E.Thyrotropin-releasing hormone receptor stimulatory antibodies

C.Thyrotropin receptor stimulatory antibodies

which tumors are patients with familial adematous polyposis syndrome at risk for?

CNS - turcot osteomas - gardner

What are the two complex 4 inhibitors of the electron transport chain?

CO/CN 4 letters complex four is the final acceptor, Oxygen, resides (complex 5 is the ATPase)

what does the chloride shift refer to and what is its signficance?

CO2 enters cells and is made into bicarb; bicarb is exchanged for Cl in RBC and bicarb travels to the lungs where the reverse reaction occurs; **bicarb travels free in the plasma to the lungs**

what are the manifestations of CREST syndrome?

Calcinosis Raynaud Esophageal dysmotility Sclerodactyly Telangiectasia

which diuretics are sulfa drugs? aka which are not?

Carbonic anhydrase inhibitors Loop (usually tolerated though) Thiazide spiranolactone

what are the risk factors for toxic megacolon?

Cdiff loperamide - in children UC

how do you remember cystathionine synthase uses B6?

Cy looks like an upside down 6

Which results due to mutations at the different loci resulting in a similar phenotype? A.Allelic heterogeneity B.Heteroplasmy C.Incomplete penetrance D.Locus heterogeneity E.Mosaicism F.Uniparental disomy

D.Locus heterogeneity

metoclopramide MOA?

D2 antagonist

what is the MOA of metoclopramide?

D2 antagonist

dramatic thickening of the capillary basement membrane. No immune complex

Diabetic glomerulosclerosis

A 40-year-old man comes to the physician because of a 2-week history of gradually worsening fatigue, nausea, vomiting, and abdominal distention. Physical examination is remarkable for scleral icterus; sublingual jaundice; a nodular, palpable liver; and a flapping tremor with wrist extension. Abdominal imaging reveals a large, irregular lesion in the caudate lobe of the liver. Which of the following is most representative of the severity of organ involvement? A. Decreased estrogen level B. Decreased level of circulating von Willebrand factor C. Elevated transaminases D. Increased β-human chorionic gonadotropin level E. Increased prothrombin time F. Increased serum albumin level

E. Increased prothrombin time Although liver enzymes (eg, AST and ALT) are often elevated in chronic liver disease, they are not indicative of liver function and therefore not best representative of severity of organ involvement.

Standard precautions (for all patients)

Ensure hand hygiene before and after contact with each patient, regardless of the use of gloves. Use gloves, gowns, and eye protection gear when exposure to blood or body fluids is likely. Respiratory hygiene by patients or healthcare workers includes covering the nose and mouth while coughing, prompt disposal of used tissues, and practicing hand hygiene after contact with respiratory secretions. Ensure safe injection practices with safe disposal of needles and sharp instruments.

Which of the following is the underlying cause of this influenza viral patient's likely disease? A.Bacterial conjunction B.Bacterial transduction C.Bacterial transformation D.Bacterial transposition E.Reassortment of viral genome F.Viral complementation check G.Viral point mutation H.Viral recombination

G.Viral point mutation

dz with c-ANCA? unique respiratory sign?

Granulomatosis with polyangiitis (Wegener) Upper respiratory tract: perforation of nasal septum, chronic sinusitis, otitis media, mastoiditis.

virus with enveloped double-stranded DNA with glycoprotein

HSV-2

Most common cause of acute renal failure in kids in US?

HUS

which interleukin induces T cells into TH1?

IL-12

x linked recessive cause of SCID?

IL-2 receptor defect

which interleukin induces B cells to switch to IgE?

IL-4

which interleukin stimulates eosinophils?

IL-5

which anti Hep B drug causes depression?

INF alpha

which TB drugs are p450 inhibitors?

INH

which renal disease causes deposits in the mesangium?

IgA nephropathy/Berger's disease

what immunoglobulins make the M spike in multiple myeloma?

IgG

what Ig is present in cold autoimmune hemolytic anemia?

IgM

testosterone and estrogen are associated with which testicular tumor?

Leydig cell tumor

how do indirect and direct inguinal hernias lie in relation to the inferior epigastric arteries?

MDs dont LIe Medial = Direct Lateral = Indirect

which MHC is made of 2 chains of equal length?

MHC 2

which oncogene is amplified in neuroblastoma? how does it differ from Wilm's tumor?

MYCN Wilm's tumor remains unilateral, whereas neuroblastoma crosses the midline

what oncogene is mutated in neuroblastoma?

N-myc

what are the three enzymes in respiratory burst?

NADPH oxidase SOD MPO

what is defective in chronic granulomatosis disease?

NADPH oxidase = can make ROS

what is the order of structures found in the femoral triangle from lateral to medial?

NAVEL nerve femoral artery femoral vein femoral empty lymphatics

how do epi and NE differ?

NE does not B2

which neurocutaneous disorder has sphenoid dysplasia?

NF1

what is the anion gap formula? what is a normal anion gap? which diabetes drug can cause metabolic acidosis?

Na- [Cl + bicarb] 8-12 metformin

Adenosine use?

SVT

Which condition? involve diarrhea containing blood and mucus. The onset of symptoms is abrupt, not gradual. Also, tenesmus is a prominent feature.

Shigellosis

inappropriate antidiuretic hormone secretion (ectopic antidiuretic hormone production), characterized by euvolemic hyponatremia caused by which malignancy?

Small cell lung cancer

Bacillus anthracis lethal factor MOA

Stimulation of macrophages to release tumor necrosis factor-α

how do the side effects of calcinuerin inhibitors differ?

Tacrolimus can cause DM Cyclosporine can cause giingival hyperplasia and hirsutism

Norepinephrine [incr/decr] diastolic filling pressure of the heart? Norepinephrine [incr/decr] heart rate?

The effects of norepinephrine at the α1-adrenergic receptor lead to peripheral vasoconstriction. This increases venous return and diastolic filling pressure of the heart. Norepinephrine administration will result in increased venous return to the heart through vasoconstriction in conjunction with increased heart contractility, producing an increase in stroke volume (SV). Because the heart no longer needs to beat as quickly to maintain sufficient cardiac output (CO) (as CO = SV × Heart rate), there is a reflex decrease in heart rate.

Antiepileptic drugs with SE of psychomotor slowing, sedation, and renal stones.

Topiramate

What do the ToRCHeS mnemonic for bugs crossing the placenta stand for?

Toxoplasmosis; Rubella CMV Herpes/HIV; Syphillus

Effect of p53 in cell cycle?

Tumor protein p53 is involved in regulation of apoptosis and angiogenesis.

which vasopressin receptor is on the kidneys?

V2 = 2 for 2 kidneys

what are all muscles of mastication innervated by?

V3

what is the only motor branch of trigeminal?

V3

what is the most significant risk factor for renal cell carcinoma?

VHL

what is delusional disorder?

a fixed false belief > 1 mo differs from paranoid because paranoid last for a lifetime

what is phosphatidylcholine?

a lecithin

what is haptoglobin?

a protein that binds free hemoglobin in the blood

copayment

a provision under which the insured pays a flat % amount each time a covered medical service is received after the deductible has been met

what are signs of lead poisoning in adults?

abd pain constipation mental status changes gingival lead lines anemia

does acetazolamide cause metabolic acidosis or alkalosis?

acidosis because bicarb is lost

imaging finding for small bowel obstruction?

air fluid levels

Txment for echinococcus granulosus?

albendazole

which steroid hormone is still made in a 17OH deficiency?

aldosterone

Busulfan MOA?

alkylating agent

which chemo drugs as a general class are cell cycle non specific?

alkylating agents

what are lab values like in osteoporosis?

all normal

which bleeding/coag tests are off with DIC?

all of them

how can Acute Promyelocytic Leukehmia (AML) be treated?

all trans-retinoic acid - induces differentiation

what binds the repressor in the lac operon when lactose is present?

allolactose

which drug can prevent the buildup of uric acid in tumor lysis syndrome?

allopurinol

norephinephrine receptor affinity?

alpha 1 > alph2 > B1

phenylephrine MOA?

alpha 1 agonist

phentolamine MOA?

alpha blocker

which antiarrhythmic causes pulmonary fibrosis?

amiodarone

what is the MOA of sulfasalazine?

antibacterial (sulfapyridine) and anti inflammatory (5-aminosalicyclic acid)

what can cause the ADAMTS13 deficiency in TTP?

antibodies or inherited

what antibody is present in CREST?

anticentromere

how to remember type 3 reaction components?

antigen-ab-complement activated, 3 things

what causes thrombosis in SLE?

antiphospholpid syndrome

where does Hpylori affect the stomach first?

antrum

what does the left recurrent laryngeal nerve course under?

aortic arch and ligamentum arteriosum

two cardiac issues with crescendo decrescendo murmurs?

aortic stenosis hypertrophic cardiomyopathy

what kind of anemia does fanconi anemia cause?

aplastic anemia - pancytopenia

lymphoid tissue in the mucosa is seen in which GI condition?

appendicitis

associations with Squamous cell carcinoma of esophagus?

associated with tobacco and alcohol use. It tends to involve the proximal and middle sections of the esophagus, rather than the distal esophagus.

what is wrong in an ebstein abrnormality caused by maternal lithium?

atrialization of the right ventricle

cells seen in EBV infection blood smear?

atypical lymphocyte = T cells

what does a coombs test detect?

autoAb

what are two complications of CLL?

autoimmune hemolytic anemia - malignant or normal cell Ab production infections - bone marrow failure

when are anti smooth muscle Ab seen?

autoimmune hepatitis

which antibiotic has the same MOA of vanco?

bacitracin

what is the treatment for methylxanthine overdose?

beta blockers - lower cAMP levels

what is the only cancer drug to affect the G2 phase?

bleomycin

what is the treatment for beta thalasemmia major?

blood transfusions and supportive care

why is there a fixed split S2 in ASD?

blood travels from left to right atrium, keeping that pulmonic valve open longer

disease causing descending flaccid paralysis?

botulism

which tissues possess the GLUT1 transported?

brain and RBCs

Oncogene in Burkitt lymphoma?

c-myc

what is the mnemonic for long thoracic nerve root levels?

c5-7 keeps your wings flying to heaven

what is the skin finding in fanconi anemia?

cafe-au-late spots

what is defective in wiscot aldrich

cant reorganize lymphocyte skeleton, T and B cells decline over time

what are the congenital rubella findings? mnemonic

cataracts, gluacoma, pigmented retinopathy; cardiac malformations; deafness EYE LOVE(HEART) RUBY EARINGS

what do thiazide and loop diuretics do to body pH?

cause alkalosis (decrease Na reabsorption proximally, increases it distally in exchange for K and H, causing low K in the body; body exchanges K inside cells for H outside)

what is the treatment for a chancroid?

ceftriaxone

what drugs are fanconi anemia patients more sensitive to?

chemo drugs - cyclophosphamide

what chromosome is MEN in MEN1 on?

chromosome 11

where is the PKD1 gene in autosomal polycystic kidney disease located?

chromosome 16

malignant risk of chronic h pylori infection?

chronic gastritis → risk for MALT lymphoma (low grade non Hodgekin lymphoma)

which chemo drug is ototoxic?

cisplatin/carboplatin (the ears and kidneys on chemo man)

what does PFK1 do?

convert F6P to F16BP

what is the MOA of lactulose to reduce ammonia levels?

converted to acetic acid and lactic acid; acidifies colonic contents; converting NH3 to NH4 so it cant be absorbed

what are the triad of sxs of paroxysmal nocturnal hemoglobinuria?

coombs negative heomlytic anemia pancytopenia venous thrombosis

what kind of glomerulonephritis occurs in granulomatosis with polyangitis?

crescent

which intrinsic laryngeal muscle is not dervied from the 6th pharyngeal pouch?

cricothyroid

anterior spinothalamic tract contains nerves that respond to

crude touch and pressure

meningitis cause in HIV?

cryptococcus

which chemo drug is MESNA used for?

cyclophosphomide to prevent hemorrhagic cystitis

what is the nephrotoxic organ transplant drug?

cyclosporine

what is the MOA of cytarabine?

cysteine analog, interferes with DNA synthesis

what happens to total iron binding capacity in hemochromatosis?

decreased

what is anti-Jo-1 Ab?

dermatomyositis and polymyositis

lateral corticospinal tract controls

descending voluntary movements of the contralateral limbs

what is the formula for loading dose?

desired plasma conc x Vd / bioavailability

what will muscle tumors stain for?

desmin

how do the functions of desmisomes and hemidesmisomes differ?

desmisomes connect cells to cells hemidemisomes connect cells to the basement membrane

Antibodies in pemphigus Vulgaris? Antibodies in Bullous pemphigoid? [mnemonic] what kind of immunofluorescence is seen in both?

desmoglein 3 in desmisomes; net like pattern type XV11 collagen in hemidesmisomes linear pattern BULLOus is BULLOw the epidermis

which steroid is used in a suppresion test to dx cushings disease?

dexamethazone

what is the mnemonic for the only 3 circular RNA viruses?

dhey are balls delta virus arenavirus bunyavirus

Nodular glomerulosclerosis with thickened basement membranes

diabetic nephropathy.

what is the most common lymphoma in adults?

diffuse large B cell lymphoma

which drug can quinidine (Na channel blocker) potentiate the effects of?

digoxin

what are the 4 D's of botulism?

diplopia dysphagia dysarthria dyspnea

symptoms in alkaptonuria?

discoloration arthralgias

what is the most common trachea-esophageal fistula?

distal

chemo drugs causing cardiotoxicity?

doxorubicin trastuzumab

what is the ABVD protocol for hodgekin lymphoma?

doxorubicin (called adriamycin) bleomycin vinblastine dacarbazine

Vd formula?

drug conc in body/ drug conc in plasma

when are antihistone Ab seen?

drug induced lupus

hpv morphology?

dsDNA circular virus

duchene vs becker muscular dystrophy genetic pathophysiology?

duchene - framshift - truncated protein becker - non frame shit - partially working protein

where is secretin made and what does it do?

duodenal S cells make causes gallbladder and pancreas to secrete bicarb

what are the symptoms of Ménétrier disease?

epigastric pain anorexia weight loss vomiting edema (due to protein loss)

common sickle cell trait symptom?

episodic hematuria

Vesicles surrounded by hemorrhagic maculopapules is characteristic of which condition?

erythema multiforme

what forms the three layers of spermatic fascia?

external - external oblique cremasteric - internal oblique internal - transversalis FASCIA ICE TIE

what changes will be seen in the liver of a Rh positive fetus in a Rh negative mother?

extramedullary hematopoesis - no damage - only thing being damaged are the fetuses RBCs - so the body must generate new ones

where does rubella rash start? where does rubeolla rash start?

face hairline

defect in hemophilia type A?

factor 8 = A = A-ght

most common cause of hypercoagulability?

factor V leiden

Most common genetic cause of increased coagulation/ blood clots? explain it

factor V leiden mutation; mutant factor V is resistant to degredation by Protein C

what forms a cleft palate?

failure lateral palatine shelves to combine or to combine with nasal septum or to combine median palatine shelve

what forms a cleft lip?

failure of maxillary and medial nasal processes

what are the symptoms of freidrich ataxia?

gait falling nystagmus dysarthria pes cavus hammer toes DM HCM

which glycolysis enzymes does Glucose 6 phosphate inhibit?

glucokinase/hexokinase

what is wrong in I-cell disease?

golgi does not target proteins for the lysosome

what is the docking protein in HIV?

gp 120 = furthest from core, highest number

what kind of casts are seen in acute tubular necrosis?

granular muddy brown

which leukemia causes dry tap?

hairy cell

Does hypo or hyperphosphatemia occur in renal failure?

hyper

what immunodeficiency is due to loss of CD40L on Th cells?

hyper IgM without CD40, B cells wont switch

what kind of mutation causes becker MD vs ducchenes MD?

in frame/non frame shift out of frame/frame shift

shiga toxin MOA?

inactivate ribosomes

how does resistance to aminoglycosides develop?

inactivation enzymes

congenital defect of the processus vaginalis when it fails to close causes

indirect inguinal hernia

Corynebacterium diphtheriae toxin MOA

inhibits protein synthesis by adenosine diphosphate ribosylation of elongation factor 2, preventing it from translocating tRNA from A site to P site

Pseudomonas exotoxin A moa

inhibits protein synthesis by adenosine diphosphate ribosylation of elongation factor 2, preventing it from translocating tRNA from A site to P site

what does increased NADH/NAD do to oxaloacetate production?

inhibits so more Malate is made

why does S2 split during inspiration?

inspiration increases preload, which increases blood in right ventricle, which delays closure of the valve so that all the blood is expelled

which hormone levels are high in PCOS?

insulin testosterone LH

which vaccines will only produce humoral immunity?

killed

what is the renal affect of multiple myeloma?

large tubular casts composed of Immunoglobulin

where is the second order SANS neuron located leading to horners syndrome?

lateral horn, synapsing on cervical trunk

how is internal jugular in relation to common carotid and vagus?

lateral to carotid ant to vagus all in carotid sheath

what two things make up surfactant around 30 weeks gestation?

lecithin and sphingomyelin (becomes all lecithin eventually)

Thiazide diuretics [incr/decr/normal]: Na, K, HCO3

low Na, low K, high HCO3 Patients who take either loop or thiazide diuretics may present with hypokalemia, hyponatremia and metabolic alkalosis.

what are the two associated findings with adult t-cell lymphoma?

lytic bone lesions hypercalcemia

which skin sensor that does light touch?

meissners corpuscles

what is wrong in TTP?

metalloprotease deficiency (ADAMS13) that normally breaks down vWF

which valve does libman-sacks endocarditis in SLE affect?

mitral

what does papillary muscle rupture result in after an MI?

mitral regurg

which heart murmur is heard with an opening snap?

mitral stenosis

which valve does rheumatic heart disease affect and what does it do?

mitral stenosis

murmur in marfans?

mitral valve prolapse

what else will worsen the same as hypertrophic cardiomyopathy murmur worsens?

mitral valve prolapse (earlier click)

what kind of bleeding is associated with vWD?

mucocutaneous (tooth pulleD)

Fungus branching at wide angles > 90?

mucor

when is a rouleaux formation seen?

multiple myeloma

what tissues will be targeted by mitochondrial diseases?

muscle and nerve (high energy)

receptors sensitive to change in length of the muscle and the rate of that change responsible for proprioception

muscle spindles

what cells are abundant in CML?

neutrophils

which drug is used to increase HDL levels?

niacin

what is the first line drug for pregnant UTI?

nitrofurantoin

what is the difference between osteoporosis and osteomalacia pathophysiology?

osteoporosis = matrix does not form osteomalacia = matrix forms but does not mineralize

What does cholera toxin do?

permanately activates Gs

Regan Lowe medium grows was?

pertussis Regan secured the border

which phase slope do class 1 antiarrhythmics control? what do all do? what do 1A, 1B, and 1C do to the AP durations?

phase 0 inhibit active channels 1A- increase 1B - decrease 1C - no effect

Antiepileptic drugs with SE of central nervous system depression, sedation, dizziness, and nystagmus

phenobarbital

Deficient enzyme in PKU? which amino acid is supplemented in the diet?

phenylalanine hydroxylase; tyrosine

two causes of Phenylketonuria?

phenylaline hydroxylase missing dihydropterin reductase missing (Bh4 cofactor missing)

is phosphatidylcholine or sphingomyelin a major component of surfactant?

phosphatidylcholine! --sphingomyelin is minimal and the lecithin/sphingomyelin ratio should be high

Non-enveloped, positive-sense, single-stranded RNA

picornaviridae (rhinovirus), caliciviridae (norovirus), and hepeviridae (HEV) viral families

what should loss of peripheral vision make you think of?

pituitary adenoma

what is the formula for maintenence dose?

plasma conc x Cl x dosage interval (ignore) / F

motor speech area: where? lesion here causes what?

posterior inferior frontal gyrus motor aphasia

writing area: where? lesion here causes what?

posterior middle frontal gyrus agraphia

which ion does not increase in concentration in saliva at high flow rates?

potassium

which DNA synthesis pathway is orotic acid made in?

pyrimidine

what is the first irreversible enzyme in gluconeogenesis?(what does it make and where and what does it need)

pyruvate carboxylase (makes pyruvate into oxolate in the mitochondria) needs biotin, ATP; activated by acetyl Coa

which toxin inhibits ribosomes?

ricin

how do rashes rickettsia rickettssii and typhus differ?

rickettsii starts on the wRist typhus starts on the trunk

which TB drugs are p450 inducers?

rifampin ethambutol

how does the blood flow to the right adrenal differ from left?

right = directly to IVC left = left renal vein

normal mean pressure range of 1-8 mm Hg

right atrium

what are the two QT prolongation congenital symptoms and how do they differ?

romano-ward - NO deafness - AD inheritance jervell and lange-nielsen syndrome - AR inheritance -neurosensorial deafness

typhoid fever sxs?

rose spots on abd constipation abd pain fever

H pylori causes what lesions to the gastric surface?

small erosions with nonspecific inflammatory infiltrate

where are most carcinoid tumors found?

small intestine midgut - so ileum and appendix

what is the most significant risk factor for transitional cell carcinoma of the bladder?

smoking

what counters cyanide toxicity?

sodium thiosulfate

what is osteomalacia simply?

softening of bones

where do dopaminergic neurons causing parkinsons originate and where do they go?

substantia nigra putamne

treatment mnemonic for nocardia and actinomyces?

sulfonamides nocardia actinomyces penicillin SNAP

glipizide MOA?

sulfonurea; blocks K channel in Beta cell, causes depolarization, insulin release

which DM drugs cause disulfirm like reactions?

sulfonylruea

what kind of symptoms will photosensitivity present as?

sunburn

Griseofulvin use?

superficial dermatophyte infections

what systemic veins lead to caput medusae in portal HTN?

superifical epigastric

where is the third order SANS neuron located leading to horners syndrome?

superior cervical ganglia C2

which diuretics can cause metbaolic alkalosis?

thiazides and loop

what is the only lab finding for immune thrombocytopenic purpura?

thrombocytopenia only -it is a diagnosis of exclusion

which platelet disorder can cause a fever?

thrombotic thrombocytopenia purpura TTP also immune TP and HUS?

Rubella virus family?

togavirus

what does the thyroglossal duct connect in development?

tongue and thyroid

which diuretics can be used to treat liddle syndrome and why?

triamterene and amiloride; these block the Na channels in the collecting tubule other K sparing diuretics do not work because aldosterone is not the problem in liddle syndrome, but a Na channel is

Two ways giardiasis can dx?

trophozoites or cysts in stool

what happens to the hormone receptor complex before binding DNA?

undergoes confirmational change

what does brucella cause vs borrellia recurrentis?

undulating fever vs relapsing fever

what are the uses for abciximab?

unstable angina PCI

which nerve roots and part of the brachial plexus is damaged in erb palsy?

upper trunk C5-C6

what does c-myc do? what does Bcl-2 do? which cancers are each found in?

upregulates the cell cycle inhibits apoptosis cmyc - burkitt 8:14 bcl-2 - follicular 14:18

what is the risk of thrombosis in heparin induced thrombocytopenia?

very high

what will mesenchymal tumors stain for?

viMEntin

what is the treatment for ALL?

vincristine prednisone L-aspariginase

which lung spirometry value is the same between obstructive and restrictive lung diseases?

vital capacity is reduced in both

excess of which vitamin is a teratogen?

vitamin A

what kind of extrinsic hemolytic anemia occurs with SLE and what is the Ab?

warm IgG

The initial therapy for SIADH is

water restriction, which increases the plasma sodium concentration. In severe (sodium ≤120 mEq/L) or symptomatic cases, hypertonic saline may be administered, sometimes in conjunction with a loop diuretic to increase urinary water losses.

what is the order of measurements affected by failure to thrive?

weight then height then head

brachial arch mnemoinc?

when at golden arches kids first CHEW then SMILE (2) then swallow stylishly (3) then simply swallow (4) then speak (6)

what is the mnemonic for brachial arches?

when at the golden ARCHES children first CHEW (1) then smile (2) then swallow stylishly (3) or simply swallow (4) and then speak (6)

what is the sound of an ASD?

wide fixed split S2

Lupus nephritis renal finding?

wire looping of glomerulus

which immunodeficiency results in thrombocytopenia?

wiskott aldrich/

HOW the risk factors for ATTEMPTING suicide are different than COMPLETING suicide in women and men?

women attempt more, but men complete more

inheritance of Duchenne muscular dystrophy?

x linked recessive. frameshift mutation

Can abx other than clinda cause C diff?

yes

Can chlamydia cause ectopic pregnancy?

yes

are both CD4 and CD8 cells involved in graft rejection?

yes

are simple hallucinations seeing the dead common in grief?

yes

are there mitochondrial inherited myopathies?

yes

can PCR be used to identify carriers of the cystic fibrosis gene?

yes

can Trousseeau syndrome secondary to pancreatitic cancer increase risk of a PE?

yes

can beer cause a hypertensive crisis with MAOIs?

yes

can radiation txtment cause constrictive pericarditis?

yes

can somatic sxs such as headache and nausea be normal in grief?

yes

can there be bronchial carcinoid tumors?

yes

do all trisomy defects cause CHD?

yes

do both 21 OH and 11 OH deficiencies cause virilization?

yes

does diphenhydramine cause antimuscarninic sxs?

yes

what is HbBart made of?

4 gamma chains

what type of thalasemmia does HbBart develop?

4 gene deletion alpha thalasemia

where does the horizontal fissure of the right lung lie?

4 rib

which familial dyslipidemia does NOT have xanthomas?

4- hypertriglyceridemia

which drugs cause acute interstitial neprhtis?

5 Ps PPI pee i diurtetics PCNs cephalosporins Pain free- NSAIDs rifampin

what age gets multiple myeloma?

> 40-50

Which of the following is the mechanism of action of vancomycin? A.Blocks peptidoglycan polymerization B.Blocks protein synthesis at the 30S ribosomal subunit C.Blocks protein synthesis at the 50S ribosomal subunit D.Inhibits DNA gyrase E.Inhibits peptidoglycan polymer cross-linking

A.Blocks peptidoglycan polymerization Vancomycin blocks peptidoglycan synthesis by binding the D-Ala D-Ala portion of cell wall precursors, preventing further polymerization. Vancomycin is used to treat serious gram-positive infections, including methicillin-resistant Staphylococcus aureus. In order to achieve systemic therapeutic levels, vancomycin must be administered IV; therefore, it is not the first-line treatment for this child's condition.

Which type of receptor is functioning abnormally in glucagon knock-out mouse? A.G-protein-coupled receptor B.Intracellular steroid receptor C.Ion channel-linked receptor D.Receptor serine/threonine kinase E.Receptor tyrosine kinase

A.G-protein-coupled receptor

does ALT or AST make pyruvate/oxaloacetate?

ALT makes pyruvate AST makes oxaloacetate

Contact precautions (Colonization with multidrug-resistant bacteria, enteric infections (Norovirus, Clostridium difficile, Escherichia coli O157:H7), viral infections [HSV, RSV, parainfluenza, enterovirus], scabies, impetigo, non-contained abscesses or decubitus ulcers)

Admit the patient to a private room. Wear gloves before entering the room and change them upon exposure to body secretions. Use gown if exposure to body fluids is likely. Place the patient in a gown during transport to reduce environmental contamination. Use non-critical supplies for a single patient.

Transient condition characterized by monocular vision loss due to embolic occlusion of the ophthalmic artery.

Amaurosis fugax

Which condition? diarrhea (notably containing blood and mucus), weight loss, and abdominal pain. His white blood cell count is elevated, indicating a likely infection or inflammatory reaction. The patient is an immigrant from India. The patient's stool sample is positive for a specific antigen, allowing the physician to arrive at a final diagnosis.

Amoebic colitis

A 55-year-old man with diabetes develops end-stage renal disease and requires a kidney transplant. After the transplant surgery, the patient meets with his nephrologist. His urinary frequency and volume are unchanged. Laboratory results show increased serum creatinine and BUN. On renal biopsy, there are dense infiltrates of CD8+ cells and significant inflammation of graft vessels. How long after the transplant did the patient most likely present with this new condition? A.18 months B.2 weeks C.2 years D.3 days E.6 hours

B.2 weeks Acute rejection—a manifestation of cell-mediated immune injury—typically occurs weeks after transplantation, but can occur as early as 5-7 days after transplantation.

The history and findings suggest that she has type IIA familial hyperlipidemia. Patients with this disorder tend to have very high levels of LDL cholesterol, which can be challenging to control with statins alone. Which of the following changes to her daily regimen would be most beneficial? A.Add a second statin B.Add ezetimibe C.Add gemfibrozil D.Discontinue the statin E.Return to an unrestricted diet F.Take statin with cholestyramine

B.Add ezetimibe Patients with this disorder tend to have very high levels of LDL cholesterol, which can be challenging to control with statins alone. Therefore in this special patient population, statin monotherapy often fails, and the most appropriate next step in management is the addition of ezetimibe or a PCSK9 inhibitor. Ezetimibe acts by inhibiting absorption of cholesterol at the brush border of the small intestine. It does this by blocking the sterol transporter, Niemann-Pick C1-Like1 protein (NPC1L1). The result of blocking this transporter is a decrease in cholesterol absorption and subsequent delivery to the liver, decreasing overall hepatic cholesterol stores and increasing clearance of cholesterol from the blood. This ultimately results in decreased LDL and total cholesterol.

A 73-year-old woman presents with extremely painful blisters on her head. She does not recall having similar symptoms before. She is undergoing chemotherapy to treat late-stage breast cancer. Physical examination shows a vesicular rash above her left ear that extends from the vertex to the left side of the back of her head. Which of the following is the root ganglion associated with the most likely cause of the patient's dermatologic lesions? A.C1 dorsal root ganglion B.C2 dorsal root ganglion C.C3 dorsal root ganglion D.C4 dorsal root ganglion E.C5 dorsal root ganglion

B.C2 dorsal root ganglion There is no C1 dorsal root ganglion, as the nerves supply the suboccipital skeletal muscles.

Which of the following features is most likely to be associated with familial adenomatous polyposis? A.Congenital mutation of APC gene B.Congenital p53 mutation C.Mismatch repair defects resulting in increased mutagenesis D.Multifocal lesions E.Polyploid lesions F.Well-differentiated histology

B.Congenital p53 mutation

most important virulence factor for Spneumo?

capsule

is amiodarone a p450 inhibitor?

yes

is diuresis a symptom of hypercalcemia?

yes

what is the most common tumor in the small intestine?

carcinoid

side effect of trastuzumab?

cardiotoxicity

what pathology causes aortic dissection in marfan syndrome?

cystic medial necrosis

what will epithelial tumors stain for?

cytokeratin

what is the defect in paroxysmal nocturnal hemoglobinuria?

decay accelerating factor

DLCO in COPD?

decr

what do ACE inhibitors do to GFR?

decrease

what do all class 1 anti arrhythmics do to the phase 0 slope?

decrease

inh MOA?

decrease mycolic acid synthesis

which vitamin deficiency causes glossitis (inflammation of tongue)?

niacin

what kind of valve lesions does carcinoid tumors cause?

right sided

what does the right recurrent laryngeal nerve course under?

right subclavian artery

normal pressure range of 1-30 mm Hg

right ventricle

is mammography the best way to detect breast cancer?

yes

is proteinuria seen in multiple myeloma?

yes

is ribosome detachment a reversible sign of cell injury?

yes

is smoking a risk factor for duodenal ulcers?

yes

should you give someone oral fluids over IV if they can tolerate it?

yes

main antibiotics causing acute tubular necrosis?

aminoglycosides Beta lactams rifampin vanco

what causes medial medullary syndrome?

anterior spinal artery infarct

ligament torn first in a lateral ankle sprain? what is torn second?

anterior talofibular ligament calcaneofibular

Ab specific for hashimotos?

anti - thyroglobulin, microsomal, thyroid peroxidase

which drug classes are used for tourretes?

anti psychotics

which antibodies are directed at snRNPs and what disease is this?

anti-smith; SLE

cause of appendicitis in adults?

appendicolith/fecolith

what is pellegra and what 3 defiencies can cause it?

niacin tryptophan (makes niacin) pyridoxine (same)

which vitamin deficiency may be present in carcinoid syndrome and why?

niacin (because it is used to make serotonin)

are anti HCV Ab present early in infection?

no

are clara cells found in alveoli?

no

do penicillins work for staph?

no

does osteoarhritis change the hemoglobin or mean corpuscal volume?

no

will an envelope survive the GI tract?

no

what is wrong in Job syndrome?

"a 17 yo guy who still has baby teeth and is very weak (fractures)" Def in JAK/STAT pathway defects of STAT3 signaling pathway (normally activated by cytokines), resulting in overproduction of IgE, but no production of IFN-gamma Eczema + incr IgE + bone/teeth problems + staph abscesses = Job Eczema + incr IgE + thromobocyto = WiskAldr

two drug names and menmonics for Xa inhibitors?

"ban Xa" rivaroxaban apixaban

enveloped positive sense ssRNA, is seen in all age groups; however symptoms are more severe in the elderly and immunocompromised

arbovirus

which sodium channel blockers do you give post MI and which do you avoid post MI?

1b - best post MI 1c - contraindicated post MI

which steps of gluconeogenesis occur in the mitochondria?

1st only

which RNA pol makes mRNA?

2 M looks like 11 a little

what are the symptoms of magnesium toxicity?

diarrhea hyporeflexia hypotension cardiac arrest Mg2+ = Must Go to the bathroom

half life formula?

.7 x Vd / Cl

which benzo has the longest half life?

diazepam

what is the bioavailability of IV drugs?

1

what are the steps of HIV testing?

1 -HIV 1 and 2 Ab plus p24 antigen 2- differentiate between HIV 1 and 2 Ab 3- if 2 is negative, test for HIV-1 nucleic acids *antigen is key cause that'll be there early*

which neurophysins carry oxytocin and ADH respectively?

1 oxytocin 2 ADH

what is the formula for number needed to treat?

1/Absolute risk reduction ARR = c/c+d - a/a+b

how many antigens can IgM bind?

10

when is pincer grasp?

10 months

when does ventricular wall rupture occur after an MI? papillary muscle rupture?

5-14 days 2-7 days

which antimetabolites are not used for leukemia/lymphoma?

5-fluoruracil azathioprine/6MP

translocation of Ewing syndrome and how to remember?

11:22 patrick ewings jersey = 33 (11+22)

how do 21 OH and 11 OH deficiencies mainly differ?

11OH causes hypertension 21OH causes hypotension

when does a babinski sign disappear?

12-14 mo

proteins made in follicular lymphoma?

14:18 (IgH and BCL-2 - anti apoptosis)

what is the most common non hodgkin lymphom?

diffuse large b cell lymphoma

two diseases without thymic shadow?

digeorge SCID!

which aortic arch gives rise to the common carotids?

3rd

when do delerium tremens start after stopping alcohol?

48 hrs

when does von gierke present?

6 mo

when can an infant babble?

6 months

what is the risk of being a carrier of a AR if your sibling has the disease?

67%

which aortic arch does a PDA arise from? what arises from the second aortic arch?

6th stapedial artery

when does stranger anxiety begin?

7-9 months

is pyloric stenosis present a birth?

no, few weeks after

when can a infant crawl by?

8 months

can parents refuse blood transfusions for their children in emergencies?

no; parents cannot refuse lifesaving treatments for children

what percent of the population falls between two Standard deviations of the mean?

95%

what translocation is present in CML? which type of gene is that?

9:22 BCR ABL proto-oncogene

which branching bacteria (nocardia or actinomyces) causes pulmonary infections?

nocardia

what does diarrhea do to pH?

non anion gap metabolic acidosis

E. coli Heat stable toxin MOA?

overactivates guanyl cyclase → ↑ cGMP → decr resorption of NaCl and H2O in gut

how are stanford type A aneurysms vs type B aneurysms treated?

A - surgery B - Beta blockers, then vasodilators

what is damaged in the unhappy triad of the knee?

ACL medial meniscus MCL

What cancer has Tdt positive cells?

ALL

what are the short acting benzos?

ATOM alprazolam triazolam --- (temazepam is outside the liver) oxazepam midazolam

uses for octreotide?

Acromegaly, carcinoid syndrome, gastrinoma, glucagonoma, esophageal varices.

where are antigens packaged onto MHC class 1?

rough er

what is measles also called? what are the 3 C's of measles?

rubeola conjunctivitis cough coryza

Droplet precautions (known or suspected infection withNeisseria meningitidis, Haemophilus influenzae type B, Mycoplasma pneumoniae, Bordetella pertussis, diphtheria, pneumonic plague, influenza, rubella, mumps, adenovirus, parvovirus B19, and RSV)

Admit the patient to a private room. Wear a mask when within 3 ft of the patient. Mask the patient during transport. Ensure respiratory hygiene.

what is the treatment for hodgkin lymphoma?

Adriamycin (doxorubicin), Bleomycin, Vinblastine, and Dacarbazine.

what is the ABVD treatment of hodgkins?

Adrimycin (doxorubicin) Bleomycin Vinblastine Dacarbazine

uses for amiodarone?

Afib Aflutter Vfib last resort

thinning and splitting of GBM

Alport syndrome

which blood vessel runs along the superior pancreaus?

splenic

what is U1-RNP involved in?

spliceosome

what defense mechanism is used in borderline personality disorder?

splitting

can both impetigo and pharyngitis strep pyogenes cause PSGN? rheumatic fever?

yes; no, only pharyngitis strains

which amino acids become essential during periods of growth?

Arganine and histidine

which tumor is alpha feto protein elevated in addition to HCC?

yolk sac

which organs are responsible for hematopoesis in a fetus?

yolk sac - week 3 to 8 liver - week 6 to birth spleen - helps week 10 to 28 bone marrow - week 18 +

which NRTI HIV drug can cause macrocytic anemia?

zidovudine

Dobutamine MOA and when is it used?

B1 > B2; for cardiogenic shock

what are the causes of post renal failure?

BPH kidney stones

which renal disease resembles the use of loop diuretics?

Bartter

Antiepileptic drugs with SE of ataxia, dizziness, and sedation

Benzodiazepines, Gabapentin

which antiarrhythmic causes impotence?

Beta blocers

which disorder is caused by splice site mutations?

Beta thalassemia

Nystatin MOA?

Binds ergosterol to form pores in membranes

ACE inhibitor-induced angioedem pathophysiology?

Bradykinin is also degraded by neutral endopeptidase (NEP), aminopeptidase P (APP), dipeptidyl peptidase-4 (DPP-4), and kininase I, which take over when ACE is inhibited.Studies indicate that patients with ACE inhibitor-induced angioedema have a deficiency of APP or DPP-4, preventing the normal compensatory response. Angioedema associated with ACE inhibitors is treated with fresh frozen plasma that contains ACE.

Periodic acid-Schiff-positive remnants of autophagic vacuoles

Bunina bodies, are found in the anterior horn of the spinal cord in patients with ALS

what vitamin fascilitates iron absorption?

C

Which bug will cause crepitus in a skin wound?

C. perfringes, due to its phospholpase -lecithinase

which compliment proteins control anaphylaxis?

C3a C4a C5a

who should not get triptans?

CAD

what is the on the end of tRNA that carries amino acids?

CCA

treatment for prinzmetal anginga?

CCB nitrates

which medications can cause GERD by relaxing the LES?

CCB (amlodipine etc) anticholinergics alpha blockers Beta agonists

what cells are involved in polymyositis?

CD8 and Mphages

which cancer can present with autoimmune hemolytic anemia?

CLL

which autoimmune disorder causes atrophy of the smooth muscle in the lower 2/3 of the esophagus?

CREST syndrome

which chemokine receptor does HIV bind on Mphages/T cells?

CXCR4 - T cells - CD4 mphages CCR5

what does CREST syndrome stand for?

Calcinosis Raynauds Esophageal dysfunction Sclerodactyly Telangiectasia

intense vulvovaginal itchiness (pruritus), accompanied by a thick "cottage cheese"-like discharge.

Candidiasis

ipsilateral Horner syndrome, characterized by ptosis, miosis, and anhidrosis caused by

Cervical sympathetic chain compression

Narrowing out the anterior chamber, which impairs aqueous humor outflow. associated with progressive peripheral vision loss and typically very painful

Closed angle glaucoma

Enveloped, positive-sense, single-stranded RNA

Coronaviridae

Enveloped, positive-sense, single-stranded RNA

Coronaviridae family of viruses, which includes common cold viruses

which vitamin should patients treated with steroids get?

D

what is a common presentation of Acute Promyelocytic Leukehmia (AML)?

DIC

what is the common presentation of AML?

DIC

what is acanthosis nigricans most often associated with rather than paraneoplastic syndromes?

DM

what causes fanconi anemia?

DNA repair defect

Erythematous, vesicular rash is characteristic of which condition?

Dermatitis herpetiformis

most common cause of urinary tract infections in sexually activie women?

E coli - lactose fermenter

what are the mnemonics for the serious disorders of fructose and galactose metabolism?

FAB GUT fructose Aldolase B Galactose UridylTransferase

what is the sxs mnemonic of TTP?

FAT RN fever anemia thrombocytopenia renal neuro

hyperIgE sxs mnemonic?

FATED coarse Faces Abscess primary Teeth retained hyperIgE Derm (eczema)

Which gene is involved in the lengthening of limbs?

FGF

which lysosomal storage disease affects the heart?

Fabry

what are the sounds heard when taking blood pressure called?

korotkoff sounds

This preadolescent patient presents with signs and symptoms of pancytopenia, or aplastic anemia, which occurs when a person experiences a decrease in all three blood cell types: red blood cells, white blood cells, and platelets. His "dizzy spells" and the "funny feeling" in his chest are presyncope and palpitations caused by anemia (RBCs). His bleeding gums and nosebleeds are a symptom of thrombocytopenia (platelets), while his frequent infections suggest leukocytopenia (WBCs). In conjunction with pancytopenia, the patient's abnormal skin (pigmented spots), skeletal (wide thumbs), and facial findings (small eyes) suggest

Fanconi anemia

which murmurs are increased by hand grip?

Mitral regurg aortic regurg VSD

MOA of metronidazole?

Forms toxic free radical metabolites in the bacterial cell that damage DNA. Bactericidal, antiprotozoal.

what rapidly reverses warfarin toxicity?

Fresh frozen plasma

what cycles do stable/quescent cells go through?

G0 to G1 (hepatocytes/lymphocytes)

SIADH [incr/decr/normal]: Na, K, HCO3

low Na, normal K, normal HCO3

Large vessel disease seen in adults (>50 years-old). It is due to cell-mediated immune injury to elastin and granuloma formation.

Giant cell arteritis

what are the ekg findings in cardiac tamponade?

low amplitude QRS, alternating height

tell the body how much force is being generated by the muscle. When a muscle contracts, ___ signal for the antagonist muscle to relax

Golgi tendon organs

rare idiopathic autoimmune condition that usually presents with rapidly progressive glomerulonephritis, basement membrane thinning and splitting, and linear IgG immunofluorescent deposits

Goodpasture syndrome

associations with MALT lymphoma

H pylori and chronic autoimmune disease (eg, Sjögren)

what are the 4 causes of target cells?

HALT hemoglobin C Asplenia Liver disease Thalessemia

manifestations of VHL?

HARP hemangioblastomas angiomas RCC pheochromocytoma

how HBV-associated HCC may occur in younger adults (20-40 years old) without cirrhosis?

HBV may integrate into the host genome of hepatocytes and induce oncogenic changes.

what is the most common hemoglobin?

HbA1 (2 alpha chains and 2 beta chains)

Helmet-shaped RBCs seen in which condition?

Helmet-shaped red cells are seen in disseminated intravascular coagulation (DIC). The blood in DIC shows a microangiopathic hemolytic anemia, characterized by schistocytes (fragmented red cells, such as helmet cells).

Which health care plan will cover this patient's hospice care?

Hospice care is covered by Medicare Part A

Causes of normal anion gap metabolic acidosis

Hyperalimentation, Addison disease, Renal tubular acidosis, Diarrhea, Acetazolamide, Spironolactone, and Saline infusion (Mnemonic: HARDASS)

painless unilateral loss of vision due to endothelial damage and fibrinous necrosis. On fundoscopy, flame-shaped, arteriovenous nicking, and thickening of the arterial walls may be present.

Hypertensive retinal hemorrhage

what is the treatment for Kaposi sarcoma?

IFN-alpha

where does the inferior rectal vein blood supply go?

IVC

Most common immunodeficiency?

IgA deficiency

which GN is HSP associated with?

IgA nephropathy

Mesangial expansion with deposits

IgA nephropathy.

where does ilioinguinal nerve arise and supply?

Ilioinguinal nerve runs on top of the spermatic cord and is from L1 and supplies the scrotum/labia and medial thigh

Patient presents with loss of sensation in his fingers and toes. He has a core body temperature of 35° C (95° F) after having been submerged in 4.4° C- water (40° F). [incr, decr?] Central blood volume, baroreceptor firing, ANP, neprilysin?

In a situation of hypothermia, homeostatic mechanisms will work diligently to maintain critical core function. Sympathetic stimulation will decrease blood flow to the periphery and increase supply to central organs in an effort to decrease heat loss and organ damage. As a result, central blood volume increases relatively while peripheral volume decreases. Baroreceptors will register this relative increase as volume overload and increase the rate of firing. The nervous system will downregulate the transport and release of antidiuretic hormone (ADH), a molecule carried along the posterior pituitary with neprilysin, a natriuretic peptide endonuclease that cleaves atrial natriuretic peptide (ANP) and brain natriuretic peptide (BNP). As a result, ADH and neprilysin levels decrease which allows ANP levels to increase. This is the same mechanism that explains increased and frequent urinary urgency while swimming in any body of water cooler than core temperature 37° C (98.6° F).

Shiga toxin MOA?

Inactivate 60S ribosome by removing adenine from rRNA --> rRNA cant move on mRNA

Enveloped, segmented, negative-sense, single-stranded RNA

Influenza is a orthomyxovirus that is enveloped, segmented and contains negative-sense, single-stranded RNA. It contains hemagglutinin, which binds sialic acid promoting viral entry, and neuraminidase, which promotes progeny virion release.

Bordetella pertussis toxin MOA

Inhibition of Gi --> Activation of adenyl cyclase by adenosine diphosphate ribosylation

what are the JONES criteria for dxing acute rheumatic fever?

Joint - migratory polyarthritis O - heart - carditis N - nodules on skin E - erythema marginatum S - sydenham chorea

Which cells secrete renin? where are they?

Juxtaglomerular cells. These cells are modified smooth muscle cells located in the tunica media of afferent glomerular arterioles (seen in the anatomical image).

how does hyperKalemia in type 4 renal tubular acidosis cause acidosis?

K moves into cells, H moves out

which leukotriene is for neutrophils?

LTB4 ---- neutroBhil

loading dose formula?

LD = plasma conc target x Vd / F

what is defective familial hypercholesterolemia? what are the sxs? what is defective inhyperchylomicronemia?

LDL receptor Achilles xanthoma corneal arcus elevated LDL C-11 or lipoprotein lipase

what three tests do you check for amiodarone?

LFT PFT TFT

mnmonic for mcmurry test result upon rotation?

LIME lateral meniscus - internal rotation medial meniscus - external rotation

what is the treatment of choice for a PE?

LMWH - dalteparin/enoxaparin

which illicit drug will dilate the pupils and cause no vital sign changes?

LSD

Lactate dehydrogenase is associated with which testicular tumor?

Lactate dehydrogenase is a non-specific serum marker for testicular cancers in general

what are the respiratory quinolone?

Levofloxacin = L for lungs

what are the pentad of symptoms in thrombotic thrombocytopenia purpura?

MAHA neuro sxs fever renal sxs thrombocytopenia

what are U1-RNP ab specific for?

MCTD

What are the 3 paraneoplastic syndromes associated with a thymoma?

MG; pure RBC aplasia; hypogammaglobulinemia

what is the treatment for seizure preventation in preeclampsia patients?

Magnesium

A renal biopsy specimen is eventually obtained, and light microscopy reveals large, hypercellular glomeruli with mesangial proliferation, along with thickening and splitting of the basement membrane. Immunofluorescent staining shows granular deposits of C3 and IgG.

Membranoproliferative glomerulonephritis

The mnemonic MUDPILES is used to remember the etiologies of high anion gap metabolic acidosis:

Methanol (eg, formic acid)UremiaDiabetic ketoacidosisPolyethylene glycolIron and IsoniazidLactic acidosisEthylene glycol (eg, oxalic acid)Salicylate (eg, aspirin) toxicity

MOA of anthrax edema factor and cholera toxin?

Mimics adenylate cyclase --> increase cAMP

what is hemophagocytic lymphohistiocytosis?

Mphages engulfing RBCs in bone marrow

what enzyme is deficient in chronic granuloamtous disease?

NADP oxidase (first enzyme in resp burst)

which electron carrier is used in steroid synthesis?

NADPH

what ion does digoxin compete with?

Na K ATPase this is why loop diuretics increase digoxin toxicity because in a hypokalemic state there is no K to compete with digoxin

what kind of kidney injury does rhabdomyolysis, say from statins, causes?

Nephrotoxic acute tubular necrosis due to myoglobin not ischemic

which joints are affected in the hand by OA vs RA?

OA - PIP and DIP RA - MCP and PIP

three benzos metabolized outside the liver?

Oxezepam Tamezepam Lorazepam outside the liver

what are the 4 seronegative spondyloarthritis?

PAIR (Rhematoid factor negative) psoriatic arthritis ankylosing spondylitis inflammatory bowel disease reactive arthritis

which vasculitis is treated with steroids and cyclophosphamide?

PAN granulomatosis with polyangitis microscopic polyangitis

where does probenecid work?

PCT

three drugs under zero order elimination?

PEA phenytoin etoh asparin high dose

which enzyme in glycolysis does citrate inhibit?

PFK1

What does JC virus cause in AIDS pt?

PML progressive multifocal leukoencephalopathy (nonenhancing areas on MRI)

PR prolongation =? QT prolongation =?

PR prolongation is a sign of first-degree atrioventricular (AV) block. QT prolongation can be congenital, or an adverse effect of certain medications, including some antiarrhythmics, phenothiazines, and tricyclic antidepressants.

what hematologic tests are off in antiphospholipid syndrome and why?

PTT is prolonged (even though antiphospholpid antibody causes clots in vivo) AND NOT corrected by mixing fresh frozen plasma; lupus anticoagulant aka antiphospholipid antibody is responsible

what lab test monitors heparin? 2

PTT- intrinsic pathway platelet count!! for HIT

Non-enveloped, single-stranded DNA

Parvovirus B19

Interactions between probenecid and amoxicillin?

Probenecid inhibits the organic anion transporter (OAT) --> inhibits the reabsorption of uric acid in the proximal convoluted tubule, and inhibits secretion of penicillin; therefore, probenecid increases the serum concentration of penicillin.

normal pressure reading of 9-25 mm Hg

Pulmonary artery pressure

how can quinidine (Na channel blocker) potentiate the effects of digoxin?

Quinidine, a class IA antiarrhythmic, decreases digoxin clearance and displaces digoxin from tissue-binding sites. Quinidine will increase the level or effect of digoxin by P-glycoprotein (MDR1) efflux transporter. Also, quinidine will increase the level or effect of digoxin by basic (cationic) drug competition for renal tubular clearance. These two forms of interaction can cause dangerous levels of digoxin, which can be rectified by the use of digoxin immune Fab (digoxin-specific antibodies). Quinidine is also associated with the development of cinchonism.

how does the pulmonary artery lie in relation to the lung hilums?

RALS right pulm artery is Anterior Left pulm artery is Superior

Effect of Rb gene in cell cycle?

RB1 is a tumor suppressor gene that normally binds the E2F transcription factor complex in quiescent cells, which prevents the cell from progressing through the G1/S checkpoint. Under appropriate conditions for cell replication, E2F is released when the Rb protein becomes phosphorylated by cyclins and their cyclin-dependent kinases. Loss of function of the RB1 gene is also associated with osteosarcoma.

what supplies the AV and SA nodes of the heart?

RCA

which cancer is recombinant IL-2 used to treat?

RCC

how do iron deficiency anemia and thalassemia lab values differ?

RDW is normal and RBC is increased in mild thalassemia RDW is increased and RBC is decreassed in iron

mutation in MEN 2?

RET gene

what are the two RA antibodies?

RF and anti CCP

two causes of nutmeg liver?

RHF budd chiari

where in the abd is meckels located?

RLQ

Recessive polycystic kidney disease mutation on chromosome?

Recessive polycystic kidney disease (mutation on chromosome 6) patients have multiple renal cysts, but the condition manifests in infancy and can often be fatal in utero.

Separation of the neurosensory layer of the retina. presents with a prodrome of "flashes" and "floaters" before eventual monocular painless vision loss. This patient's presentation is too acute to suspect this diagnosis.

Retinal detachment

how do you remember if right to left shunts or left to right shunts present with cyanosis earlier?

Right to Left is eaRLy left to right is LateR

which ToRCHES bug causes PDA in fetuses of infected mothers?

Rubella

causative agent of osteomyelitis in IV drug use?

S aureus; also Pseudomonas, Candida

which type of strep viridans causes endocarditis?

S. sanguinis

which heart sound is associated with dilated cardiomyopathy?

S3

what are the heart sounds of hypertrophic cardiomyopathy?

S4

where in the cell are steroids made?

SER

what is the treatment for a woman with osteoporosis and fam h/o breast cancer?

SERMs

what transports glucose and galactose across apical GI lumen? fructose?

SGLT-1 (sodium dependent; gradient set up by Na K transporter on basal side) GLUT 5

which gene is involved in tissue patterning; however, a mutation would cause a form of holoprosencephal?

SHH. This would present as a wide range of symptoms with the most severe type resulting in cyclopia and the mild type resulting in cleft lip.

which disease can cause a false positive for testing of syphillus?

SLE

non cancer causes of anemia of chronnic disease?

SLE IBD RA

___ are a portion of the reflex arc that allows ions to pass through when the muscle contracts.

Stretch receptors

how do T cell vs B cell ALL cancers differ in age?

T cell = adolescent males B cell = young children

how do a B cell lymphoma vs T cell lymphoma differ?

T cell will have a mediastinal mass

is CD28 on T cells or APCs?

T cells

what is the cause chronnic mucocutaneous candida infections?

T cells, not B cells

how to differentiate nocardia and TB?

TB does not gram stain positive

cocaine has the same MOA as what antidepressants?

TCAs

which drugs treat tumor lysis syndrome?

TLS is treated with allopurinol, a xanthine oxidase inhibitor, or rasburicase, an exogenous enzyme that converts uric acid to allantoin which is then excreted in urine.

what is the first line txtment for UTIs?

TMP SMZ

what is the treatment for pneumocystis jiroveccii PNA?

TMP-SMZ

what is the treatment for pneumocytis pneumonia?

TMP-SMZ

what are the 5 T's of neonatal cyanosis?

TOF truncus arteroiusus transposition of great vessels tricuspid atresia total anomolous pulmonary venous return

what does TRH stimulate the release of?

TSH and prolactin

what are ALL cancer positive for?

TdT

which NRTI is a nucleotide?

Tenofovir

out-of-pocket maximum

The most money you will pay during a year for coverage. It includes deductibles, copayments, and coinsurance, but is in addition to your regular premiums. Beyond this amount, the insurance company will pay all expenses for the remainder of the year.

Antiepileptic drugs with SE of Stevens-Johnson syndrome (

The most serious adverse reaction to lamotrigine is Stevens-Johnson syndrome (SJS), which is an immune complex-mediated hypersensitivity reaction that involves the mucocutaneous surfaces and is potentially fatal. It is important to discontinue lamotrigine in patients who develop a rash while taking it. SJS is also associated with the use of the anticonvulsants phenytoin, ethosuximide, and carbamazepine.

The term "histiocyte" is equivalent to? function?

The term "histiocyte" is equivalent to "macrophage," which is a cell of the innate immune system that phagocytoses cell debris, infected cells, and pathogens.

Why pt with medullary thyroid cancer (calcitonin) has HIGH Ca?

This pt may have MEN 2A, which includees medullary thyroid cancer (calcitonin = low Ca) and parathyroid hyperplasia (incr Ca). The latter overrides the former --> hyper Ca.

what is the treatment for heparin induced thrombocytopenia?

argatroban

what are the hallmark findings of rheumatic heart disease? what cells are in them?

aschoff bodies activated Mphages, anitschkow cells

what is the physiologic dead space formula?

Vd = TV x PaCO2 - PeCO2/ PaCO2

where does human adult T cell virus cause lesions?

bone (causes hyperCa) and skin

what gene is deficient in wiskott aldrich and what is wrong?

WAS protein; cant present antigen due to cytoskeleton defect

what are the symptoms of wiskott aldrich?

WATER wiskot aldrich thrombocytopenia eczema recurrent pyogenic infections

what sxs are not present in cholelithiasis?

WBC count fever jaundice

two disorders related to multiple myeloma without CRAB findings?

Waldenstrom Macroglobulinema - sxs Monocolonal gammopathy of undetermined significance - no sxs

which test is SDS-PAGE with antibody-epitope interaction?

Western blot

What is pulsus paradoxus?

When the BP > 10 mmHg on expiration than on inspiration

Decreased serum ceruloplasmin

Wilson disease

Which gene is associated with dorsal-ventral axis patterning of the limb bud. As such, it has been associated with skeletal deformities of the limbs?

Wnt-7

what is the preferred treatment for Afib?

X inhibitors

what do neural crest cells in brachial arches become?

bone and cartillage

What is myelofibrosis?

bone marrow fills with fibroblasts leading to fibrosis

when are acanthocytes seen?

abetalipoproteinemia liver disease

how to tell if someone has a prion disease?

acute onset

what is an example obstetric complications that causes DIC?

amniotic fluid embolism

What is the txment for enterococcus UTIs?

ampicillin

Long- and short-term memory loss that includes a patient forgetting his name would be seen in?

an individual with dissociative amnesia. Memory for self is strongly affected in dissociative amnesia.

visual speech area: where? lesion here causes what?

angular gyrus alexia

thyroid papillary carcinoma histo findings?

annie orhpan eyes psammoma bodies

who gets alpha thalassemia?

asian; african

which fungus is catalase positive?

aspergillous

common cause of sudden SOB in children?

aspiration

what is a diverticulosis described as being?

attenuated muscularis propria

most common location for stroke due to uncontrolled HTN?

basal ganglia lacunar stroke- most common type - lenticulostriate arteries

what happens to the portal vein in cirrhosis?

becomes fibrotic

midazolam MOA and use?

benzo for surgery

when is HbA2 increased?

beta thalassemia major

what is the inflammation like in rheumatoid arthritis?

bilaterall and symmetric

What is the MOA of glitazones?

bind PPAR which increases sensitvity to insulin

what is the MOA of bisphosphonates?

bind hydroxyapetite, inhibiting osteoclast activity

what is late menopause a risk factor for?

breast cancer

what two medications are used to treat neuroleptic malignant syndrome? what causes it?

bromocriptine dantrolene antipsychotic use

Most common cancer associated with asbestos exposure?

bronchogenic carcinoma

down syndrome effect on eye?

brushfield spots - spots on iris

what is the acute mechanism of buffering Hydrogen ions in respiratory acidosis?

buffering of hydrogens during acute respiratory acidosis occurs through intracellular proteins, H-K exchanger, and H binding hemoglobin in RBCs, reducing Hgb affinity for O2

which local anesthetic is cardiotoxic?

bupivaicaine

which antidepressant is contraindicated in bulemia/anorexia?

bupropion - causes seizures

how can you tell busulfan and bleomycin side effects apart?

busulfan causes myelosuppression

what are the triad of congenital defects caused by rubella?

cataracts PDA (also pulm artery stenosis) deafness EYE HEART RUBY EARRINGS

what does anorexia do to bone?

cause stress fractures

what does bartonella cause in children vs AIDS pt?

children - cat scratch fever (chronic local lymphadenitis) AIDs - baciliary angiomatosis (

what is blocked in cholecystitis vs choledocolithiasis?

cholecysititis - cystic duct choledocolithaisis - common bile duct

what is the macrolide liver toxicity?

cholestatic hepatitis

what are the 3 C's of TCAs?

coma cardiotoxic convulsions

which breast cancer has a central area of necrosis?

comedocarcinoma

what is the MOA of methotrexate?

competitive inhibitor of DHFR

NRTI mechanism of action?

competitively compete with nucleotides for reverse transcriptase

most common cause of pericarditis?

coxsackie B (picornavirus) smallest RNA virus --- picoRNA --- small RNA

Brain tumor with Stratified squamous epithelium in spongy reticular stroma with prominent peripheral gliosis

craniopharyngioma

what kind of murmur is heard with aortic stenosis?

crescendo decrescendo

which antibiotic can cause nephrogenic DM insipidous?

demeclocyline

Autoimmune hemolytic anemia is what Coomb's test positive? [explain?] what will the same Coomb's test result be for G6PD? what will be elevated? what will be low?

direct; Ab are added directly to patient RBC and react with Ab bound to RBC; negative, no Ab LDH and indirect bilirubin elevated haptoglobin low

what does stylopharangeaus do?

elevate pharynx and larnyx

which drugs most significantly reduce triglyceride levels?

fibrates

which lipid lowering drug can cause gallstones?

fibrates

what protein is defective in marfans?

fibrillin - which forms a sheet around elastin FBN1 is the mutation

which brachial arch does a cleft palate arise from?

first

Stomach distention and vagal stimulation increase release of

gastrin

what do the glans clitoris and glans penis derive from?

genital tubercle

what is struma ovarii?

germ cell ovarian teratoma that has thyroid tissue

what age are fibroadenomas most common?

girls in their teens and 20s

Brain tumor with Pleomorphic, anaplastic cells with foci of necrosis in a palisading pattern

glioblastoma multiforme

which insulin drug causes HF, edema, and wieght gain?

glitazones

what are the main side effects of ganciclovir?

hematologic nephrotoxic

what are the two main types of normocytic anemia?

hemolytic and non hemolytic

side effect of ribavirin?

hemolytic anemia

disease associated with Poly arteritis nodosa? vessels affected?

hep B small or medium

what are the 3 circular DNA viruses?

hep B hpv polyomavirus

what gives the charge the basement membrane in the glomerulus that is lost in minimal change disease?

heparin sulfate

how does the lymph node spread of hodgkins lymphoma differ from non hodgekins?

hodgekins is continuous non is not

what lab values are increased in b12 megaloblastic anemia?

homocysteine mma

what do IL's do?

hot T bone stEAK 1- fever 2- T cells 3- bone marrow 4- IgE 5- IgA 6- aKute phase

what is a common cause of cyanide poisoning?

house fires - furniture releases

dromotropic meaning?

how fast current moves through AV node

which casts can be a normal finding?

hyaline casts

treatment for essential thrombocythemia and MOA?

hydroxyurea inhibits ribonucleotide reductase (decreasing DNA synthesis)

what are the lab valvues in osteoitis fibrosa cystica? calcium alkaline phosphotase phosphate

hypercalcemia increased Alkaline phosphotase hypophosphotemia osteoporotic lesions on xray

side effects of ARBs?

hyperkalemia

what is the hyperglycemia crisis in older adults called?

hyperosmolar hyperglycemic state

what is Ménétrier disease?

hyperplasia of gastric mucosa with atrophied rugae and parietal cells

what thyroid issue occurs in molar pregnancy?

hyperthyroidism

what disease is x-linked dominant?

hypophosphatemic rickets

where is the first order SANS neuron located leading to horners syndrome?

hypothalamus, runs to lateral horn

what are fatigue, weight gain, and constipation sxs of?

hypothyroidism

what are the thyroid side effects of amiodarone? what are the skin effects of amiodarone?

hypothyroidism/hyperthyroidism blue grey skin

what enzyme is defective in hunter syndrome?

iduronate sulfatase

MOA of aminoglycosides?

iinhibit initiation complex formation

what is wrong in Job disease?

il 17 defective so nphils cant be recruited

Homocysteine in B9 def? [incr/decr/normal]

incr Folate is a necessary nutrient that has numerous biochemical functions. It acts as a one-carbon carrier for methylation reactions, is involved in DNA synthesis, and is required for the conversion of homocysteine to methionine. Deficiency results in increased homocysteine levels as well as impaired nucleic acid synthesis. Thymidylate synthase, which makes deoxythymidine monophosphate (dTMP) from deoxyuridine monophosphate (dUMP), is particularly susceptible to folate deficiency, which prevents the formation of thymine.

what does inspiration do to right heart sounds?

increase

what does low levels of dopamine do to renal blood flow?

increase

what normally happens to transferin saturation in pregnancy?

increases

how does exercise increase insulin sensitivity?

increases GLUT 4

what is the MOA of adenosine and what can you think of it as?

increases K efflux which decreases Ca influx through L type calcium channels so you can think of it like a CCB

how does retinoblastoma usually develop?

inherited mutation followed by sporadic mutation

what do sulfa drugs do to p450?

inhibit

what is the specific MOA of macrolide abx?

inhibit 50S by inhibiting transloaction

why do COX 2 inhibitors cause thromboses?

inhibit COX2 prostacyclins only, leaving COX-1 free to make TXA2 which can activate platelets

methotrexate MOA?

inhibit DHFR, same as TMP (in tmp-smz)

what is the MOA of gliptins?

inhibit DPP4, which normally breaks down GLP-1

what forms the cremasteric muscle and fascia?

internal oblique

what is the most common cause of bloody discharge?

intraductal papilloma

what is defective in pernicious anemia?

intrinsic factor

what kind of renal failure is acute tubular necrosis?

intrinsic renal

Invagination of mesenteric tissue seen in which GI condition?

intussusception

what effect does digoxin have on the heart?

ionotropic

what kind of anemia will colon cancer cause?

iron deficiency microcytic hypochromic

what platelet action will be deficient in bernard soulier syndrome?

platelet binding to the subendothelium via vWF Gp1b receptor

why is poxvirus notable?

it is the largest DNA virus

why is esmolol chosen over metoprolol for trial treatment of Atrial flutter?

it's shorter acting

what are the symptoms of Weills disease?

jaundice azotemia fever hemorrhage anemia

what are the sxs of parvovirus infection in adults?

joint pain in hands and fatigue after URI sxs

What is the most common form of arthritis in children? what are the symptoms of the systemic form?

juvenile idiopathic arthritis; quotidian fevers, arthritis and pink salmon rash

what teratogenic effect do TMP/SMZ drugs have?

kernicterus

what is the treatment for cushings?

ketoconazole

which azole causes skin darkening?

ketoconazole

what are the two alpha and beta blockers?

labetolol carvedilol

what do the urogenital folds form in females?

labia minora

what are the scrotum and labia majora derived from?

labioscrotal swelling

small, angular fibers indicative of muscular atrophy due to

lack of neuronal input (denervation atrophy)

what are lab findings in cyanide poisoning?

lactic acidosis (electron transport chain is shut down = glycolysis)

which cells are found in pagets disease of the breast?

large cells with clear cytoplasm (resmembling a halo) surrounding hyperchromic nuclei

auditory area: where? lesion here causes what?

posterior superior temporal gyrus sensory aphasia

where does blood go from the left gastric vein in cirrhosis?

left gastric → esophageal → azygous

when are hemosiderin laden macrophages seen in the lungs?

left sided heart failure

normally have a dramatically higher peak systolic pressure (>90 mm Hg)

left ventricle

how long does grief usually last?

less than 6 mo

what is benzene exposure associated with?

leukemia myelodysplastic syndrome aplastic anemia

what are the symptoms of leishmaniasis?

leukopenia (which malaria does not cause) anemia hepatosplenomagaly fevers

which tissue is involved in granulomatosis with polyangitis that is not with PAN?

lungs

what is anti-dsDNA specific for?

lupus

wire-loop appearance with subendothelial basement membrane deposits

lupus nephritis.

does IVC compression occur in pregnancy and how is this treated?

lying on left side at night

which amino acids are histones rich in?

lysine and arginine

what is chediak disease?

lysosomal issue

what is a hematologic side effect of TMP/SMZ and why?

macrocytic anemia blocks formation of THF - both block the folate pathway = folate deficiency TMP blocks the enzyme after SMZ blocks the enzyme

only protein synthesis Abx causing QT prolongation?

macrolides

why does squamous cell carcinoma cause hypercalcemia?

makes PTH

what is the diagnosis of bipolar one? what is the diagnosis of bipolar two? what is cyclothymia?

manic mood for 1 week hypomanic and a depressive episode mild bipolar lasting 2 years

normally 4-12 mm Hg

mean pulmonary artery wedge pressure

which ligament forms the aortic hiatus?

median arcuate ligament

who gets beta thalassemia?

mediteranian

thyroid cancer with amyloid?

medullary thryoird

which thyroid tumor secretes calcitonin?

medullary thyroid tumor

what two things can homocysteine be made into?

methionine and cystathionine (cysteine)

which drug can be used to treat ectopic pregnancies?

methotrexate

cancer drugs causing pulmonary fibrosis?

methotrexate busulfan bleomycin carmustine

Methylglobinemia txtment? what is methehoglobin?

methylene blue and Vitamin C Fe3

what is the first line treatment for gastroparesis?

metoclopramide

Txment for giardiasis?

metronidazole

what is Km also called?

michaelis-menton constant

what anemia does lead poisoning cause?

microcytic hypochromic

when does micronodular vs macronodular cirrhosis occur?

micronodular cirrhosis is seen in alcoholic cirrhosis whereas macronodular is seen in chronic viral or drug induced

which two diseases have antineutrophil cytoplasmic antibodies (p-ANCA)? both involve what organ systems?

microscopic polyangitis Eosinophilic granulomatosis with polyangitis (Churg-Strauss); kidney and respiratory

why does vincristine/vinblastine cause peripheral neuropathy?

microtubule assembly inhibition inhibits transport of NT

griseofulvin MOA?

microtubule inhibitor

what are the T cell lymphomas?

mycosis fungoides (derm) / sezary syndrome (leukemia) adult t cell lymphoma - caused by HTLV

what are the side effects of ganciclovir?

myelosuppression nephrotoxic

what are side effects of fibrates?

myopathy cholesterol gallstones

disease causing testicular atrophy?

myotinc MD

what disease is caused by a CTG repeat?

myotonic MD

how is chronic granulomatosis disease dx?

negative dihydrorhodamine test - tests the function of NADPH

is IgA nephropathy nephritic or nephrotic?

nephritic

is alport nephritic or nephrotic syndrome?

nephritic - GBM disrupted

what drug is used to increase HDL?

none! we dont use therapy to increase HDL

what are calcium and phosphate levels like in Pagets?

normal

what is alk phos like in multiple myeloma?

normal

what are lab values of Pagets?

normal Calcium and phosphorus elevated alk phos

Loops diuretics [incr/decr/normal]: Na, K, HCO3

normal Na, low K, High HCO3 Loops diuretics are less likely to cause hyponatremia because while their action increases ADH levels by inducing volume depletion, responsiveness to ADH is reduced because of impairment of the medullary gradient. Therefore, the patient will present with normal sodium levels in addition to hypokalemia and metabolic alkalosis.

what can the MCV be in fanconi anemia?

normal or macrocytic

what kind of anemia is spherocytosis?

normocytic intrinsic hemolytic

are fatty changes seen in the liver in acute hepatitis?

not really

congenital syphillus facial defects?

notched teeth saddle nose short maxilla

does a nucleotide or nucleoside posses a phosphate?

nucleoTide has phosphaTe

ipsilateral, gaze evoked, broken smooth pursuits

nystagmus

first line txtment for oral thrush?

nystatin

Cotton-wool spots on fundoscopy?

occlusion of the central retinal artery secondary to carotid artery atherosclerosis

cells attacked in MS? what does MRI show?

oligodendrocytes; periventricular white matter plaques

Brain tumor with Regular round cells with spherical nuclei with finely granular chromatin in clear cytoplasm

oligodendroglioma

what are omega 6 and omege 3 acids also called?

omega 6 - linoleic acid omega 3 - alpha linolenic acid

what are the neuroaminidase inhibitors of influenza?

oseltamivir zanamivir

which cell is deficient in osteopetrosis? why do they have bone marrow loss?

osteoclasts medullary cavity is filled with bone that is not removed

does the oxygenated or deoxygenated form of Hb bind O2 better?

oxygenated binds better

which mutation causes Li Fraumeni syndrome? where are 3 places they get cancer?

p53 lung breast colon

which skin sensor does vibratory touch?

pacinian corpuscles

lateral spinothalamic tract contains nerves that respond to

pain and temperature

Granuloma inguinale (caused by Klebsiella granulomatis) initially presents as a

painless genital ulcer without adenopathy.

only Beta blockers used for migraines?

propanolol timolol

which HIV drugs cause lipodystrophy?

protease inhibitors

what is the main virulence factor of s. aurues?

protein A

functions of pepsin

protein digestion

where are T cells located in a LN? B cells?

paracortex follicles (in the outer cortex)

difference between parainfluenza virus morphology and norovirus morphology?

parainfluenza - ssRNA linear negative sense norovirus (calcivirus) - ssRNA linear positive sense

what is the mullerin duct also called?

paramesonephric duct

what are the uterus, uterine tubes, cervix, and superior 1/3 of vagina derived from?

paramesonephric ducts

does bromocriptine treat psychosis or parkinsons?

parkinsons

why does parvovirus precipitate symptoms in someone with heridatary spherocytosis?

parvovirus causes aplastic anemia by destroying myeloid cells in bone marrow, so an already defunctional blood cell production is now weakened

Severe retrograde amnesia in combination with mild anterograde amnesia is seen in?

patients with traumatic brain injury.

what does basal cell carcinoma look grossly? what does it look like microscopically?

pearly borders fine telangiectasias; palisading nuclei islands of tumor cells

what is the first line treatment for actinomyces isreali? mnemonic for actinomyces and nocardia?

penicillin SNAP sulfa nocardia actinomyces pcn

what do chief cells make?

pepsinogen

what is defective in diaphragmatic hernias in infants?

pleuroperitoneal membranes

what PNA should you suspect in a immunocomprimised pt?

pneumocystis jiroveccii

Agar used to grow pertussis?

potatoes

which virus causes molluscum contagiosum?

poxvirus

treatment for flukes?

praziquantal

what is pulmonary capillary wedge pressure a marker for?

preload

what are bone side effects of taking test?

premature closure of epiphyseal plates

which hypothalamus makes GnRH?

preoptic

amifostine use?

prevent cisplatin nephrotoxicity

what is the MOA of neuroaminidase inhibitors?

prevent viral progeny release

MOA of cocaine?

prevents reuptake of NE, 5HT, dopamine

which malaria drug should not be used with G6PD and what is it used for?

primaquine vivax /ovale

what are the two types of antiphospholipid syndrome?

primary or secondary - due to autoimmune condition

Ursodeoxycholic acid is used in what?

primary biliary cirrhosis treatment

what is the most common cause of hypercalcemia?

primary hyperparathyroidism

what is addisions disease?

primary hypoadrenalism (low cortisol, aldosteroone and tesT)

The anterior corticospinal tract is involved in controlling

proximal muscles (eg, trunk)

Exaggerated cremasteric muscle contraction is the cause of

pseudocryptorchidism

Most common cause of otitis externa? media?

pseudomonas; s. pneumo

what is characteristic histo finding with GBM?

pseudopallisading necrosis

what cells normally line bronchi and what cells line it after smoke exposure?

pseudostratefied columnar epithelia; sqaumous

how does mannitol cause pulmonary edema?

pulls fluid into the interstitial space

normally 4-12 mm Hg

pulmonary artery diastolic pressure

what is the most common cause of death in congenital diaphragmatic hernia?

pulmonary hypoplasia

most important prognostic factor in tetrology of fallot?

pulmonic stenosis

x ray findings in multiple myeloma?

punch out lesions

how to tell pcp from coke/meth?

pupils are normal and nystagmus distinguish from cocaine or amphetamine

what is the MOA of Cladribine?

purine analog, inhibit DNA pol

what is the speed of conduction through different areas of the heart?

purkinje > atria > ventricles > bundle of his > SA/AV node

Sxs of IgA deficiency?

recurrent respiratory and GI infections

what is BUN/Cr like in intrinsic renal failure?

reduced

how could hydralazine cause angina?

reflex tachycardia, thus it should be used for people with coronary artery disease

what does neurofibromin (NF1 endocded) normally do?

regulate RAS

what is the MOA of nitroprusside?

release NO = increase cGMP = activate MLCphosphorylase = vasodilation

in who should ACE inhibitors be used in caution with?

renal artery stenosis pt

which two cancer metastases in the brain cause intracranial hemorrhage?

renal cell carcinoma melanoma

what is selection bias?

researcher chooses to not use data from patients who have dropped out of study (whether on their own or from death)

where do patients with von hippel lindau disease get hemangioblastomas?

retina cerebellum brainstem (medulla)

most common cause of the common cold?

rhinovirus

What kind of stool does cholera cause?

rice water stool

Most common cause of osteomyelitis?

s. aureus

two bugs with super antigens and what are the toxins called and what are their MOA?

s. aureus - Toxic shock syndrome toxin -1 s. pyogenes - exotoxin A both bind MHC and TCR → cytokine release

Pathogen causing acute bacterial endocarditis? Pathogen causing subacute?

s. aureus; viridans strep

most common cause of lobar PNA?

s. pneumo

most common cause of meningitis in young children?

s. pneumo

Most common bacteria causing osteomyelitis in sickle cell?

salmonella

vector for leishmaniasis?

sand fly

erythema nodosum is seen in which disease?

sarcoidosis

which cancers are associated with Li Fraumeni cancer syndrome?

sarcoma breast leukemia adrenal glands

what tumors are peds with retinoblastoma at risk for?

sarcomas osteosarcomas melanoma brain cancer

what is a hallmark symptom of toxic shock syndrome? it also gets a name from it

scalded looking skin scalded skin syndrome

slow and slurred, monotonous and irregular speech

scanning speech

difference between avoidant and shizoid personality disorder?

schizoid dont care they dont have friends

are the schwann cells are peripheral neurons directly attacked in GBS?

schwann

HIV screening vs confirmation? blot mnemonics?

screening - ELISA confirmation - Western blot SNoW DRoP

Segmental sclerosis with hyalinosis

segmental glomerulosclerosis.

Placental alkaline phosphatase (PLAP) is elevated in which testicular tumor?

seminoma

what causes chest pain in bacterial endocarditis?

septic emboli in lung = pain is similar to PE pain

Lorcaserin MOA?

serotonin 2C agonist

what thyroid issue causes hyperprolactinemia?

severe hypothyroidism causes release of TRH which can stimulate prolactin release

what is mnemonic for things that dont need parental consent for txtment?

sex drugs rock/nroll- (trauma)

what is a classic cause of hormone issues in a woman post partum?

sheehan syndrome - everything is low

which anemia can vitamin B6 deficiency cause?

sideroblastic

ADP receptor inhibitors have a MOA similar to what bleeding disorder?

similar to Glanzmann thrombasthenia

Sirolimus vs Tacrolimus MOA?

sirolimus inhibits response to IL-2 through mTOR tacrolimus inhibits transcription of IL-2

what are the triad of sxs in kartagener syndrome?

situs inversus chronic sinusitis bronchiectasis

what cycles do labile/dividing cells go through?

skip G0, go directly to G1

what vessels are affected by Beurgers disease? who gets it? what symptom may they have?

small and medium young men who are heavy smokers Raynauds

which cancer cause lambert eaton myasthenic syndrome?

small cell lung

does a smaller or larger VSD sound louder?

smaller

what is the morphology of parvovirus?

ssDNA linear

is grade or stage more prognostic of cancer prognosis?

stage

what must occur in good samaratin laws?

standard care must be given

what are the mood changes in cushings syndrome called?

steroid psychosis

what medication can cause avascular necrosis of the hip?

steroid use

side effect of the anti seizure drug lamotrigine?

stevens johnson syndrome

reserpine use?

stop dyskinesias in parkinsons

are strep or staph in chains?

strep = chains

how is Low molecular weight heparin given? does it require monitoring?

subQ; no

What is a rare complication of measles?

subacute sclerosing panencephalitis

which abx will cause kernicterus in infants?

sulfa- displace UCB from bilirubin

which part of the hypothalamus makes ADH?

supraoptic nucleus

Cause and treatment of neonatal respiratory distress syndrome?

surfactant deficiency give surfactant (phosphatidylcholine)

which veins make up the systemic portion and the portal portion of esophageal varicies?

systemic - aszygous portal - left gastric vein

which veins make up the systemic portion and the portal portion of hemorrhoids?

systemic - middle and inferior rectal v. portal - superior rectal v.

which nucleotide is absent/present in RNA?

t gone; uracil present

RBCs with an outer ring of hemoglobin surrounding a central condensation of hemoglobin.

target cells

what are the smear findings in an asplenic patient?

target cells howell jolley bodies

what cells are seen in myelofibrosis?

tear drop cells bone marrow is crying because it is fibrosed and dry tapped

what age gets ewing sarcoma?

teens

what is the taut form of hemoglobin? what is its oxygen affinity like? where is it found?

the deoxygenated form low affinity taut form is found in peripheral tissues and will give off oxygen

what connects the third ventricle to the lateral ventricles? the thrid to the fourth?

the foramen of monro; aqueduct of sylvius

where do the initial steps of the urea cycle (up to citruline) take place?

the mitochondrial matrix

which part of the lymph node will expand in response to a viral infection?

the paracortex

how do the GAA repeats in freidirch ataxia cause disease?

the repeats expand so much that the normal gene (frataxin) cannot be transcribed

what is the mechanism of action of fibrates? it uprelu

they are PPAR-alpha transcription factor agonists, which lead to upregulation of lipoprotein lipase

what happens to thyroid binding globulin levels in pregnancy?

they increase

what are the two super antigens?

toxic shock syndrome toxin - s.aureus s. pyogenes - exotoxin A

actions for child abuse?

treat then call authorities

what is HbA2 made of?

two alpha chains and 2 gamma chains

what is bupropion contraindicated for?

use with low seizure threshold - anorexia

what is the most common inherited bleeding disorder?

von willebrand disease

The patient is presenting with mucosal bleeding, epistaxis, and symptoms of anemia. Laboratory values also reveal increased bleeding time and increased partial thromboplastin time. His mother has a history of anemia. Dx?

von Willebrand disease (vWD), the most common inherited bleeding disorder

what med prevents strokes with Afib?

warfarin

which anticoagulant drug crosses the placenta?

warfarin

why is lidocaine better for ventricular arrhythmias?

works on inactviated channels better and atrial action potentials are shorter

A 9-year-old boy is brought to the emergency department by his mother because of new-onset abdominal pain in his right lower quadrant, accompanied by nausea and vomiting. His belly appears bloated and distended and is tender to the touch. He has stopped eating and drinking over the past 24 hours. His temperature is 38.5°C (101.3°F), and his white blood cell count is 13,000/mm³. Release of external pressure on the abdomen causes more pain than does deep palpation. Which of the following is the most likely etiology of this child's condition? A.Congenital anomaly B.Consumption of a foreign body C.Recent viral infection D.Trauma E.Tumor

C.Recent viral infection Appendicitis occurs when the lumen of the appendix is obstructed, resulting in mucosal injury and bacterial proliferation. In children, the most common cause of acute appendicitis is lymphoid hyperplasia. This commonly occurs as a response to a viral infection or vaccination.

A 24-year-old woman presents to her gynecologist with a report of increasing, thick vaginal discharge for the past 3 days and new-onset pain during sexual intercourse. She takes oral contraceptive pills and uses condoms infrequently with her new sexual partner. Her gynecologist treats her empirically and also cultures a sample of the discharge. The culture reveals gram-negative cocci in pairs. Which of the following describes the cell type that mounts the initial immune response against this organism? A.Bilobular nucleus with numerous pink granules on hematoxylin and eosin stain B.Bilobular nucleus with numerous purple granules on hematoxylin and eosin stain C.Contains a single, slightly off-center nucleus with heterochromatin in a clock-face arrangement D.Multilobular nucleus with large, spherical, azurophilic granules E.Similar in size to erythrocytes and containing one large nucleus with little surrounding cytoplasm

D.Multilobular nucleus with large, spherical, azurophilic granules For bacterial infections, polymorphonuclear cells (see image below) are the most common cell type in an acute response to bacterial infection. They have a characteristic segmented nucleus with three to five connected lobes with large, spherical, azurophilic granules and hydrolytic enzymes.

A researcher team is studying diabetes mellitus. The team is trying to understand the molecular pathways involved in insulin resistance. They create a strain of genetically altered mice with a mutated insulin receptor that is only partially activated upon ligand binding. Under normal cellular conditions, the insulin-receptor mediated signaling cascade is analogous to which of the following? A.ACTH B.BNP C.Cortisol D.Erythropoietin E.IGF-1

E.IGF-1 The actions of insulin are mediated at the cellular level by binding of insulin to its receptor, followed by autophosphorylation of tyrosine residues on the insulin receptor. This generates a tyrosine kinase that participates in an intracellular signaling cascade. This is the same signaling pathway of IGF-1 (insulin growth factor-1), PDGF (platelet derived growth factor), and FGF (fibroblast growth factor).

Which of the following features is most likely to be associated with sporadic colorectal cancer and colorectal cancer in patients with familial adenomatous polyposis? A.Congenital mutation of APC gene B.Congenital p53 mutation C.Mismatch repair defects resulting in increased mutagenesis D.Multifocal lesions E.Polyploid lesions F.Well-differentiated histology

E.Polyploid lesions

A 61-year-old woman complains of a tingling sensation in her feet that has become progressively worse over the past several months. The patient works as a pastry chef in New York City and has not traveled anywhere outside of the city in the last 5 years. On physical examination, she appears mildly jaundiced, and her tongue has a glazed appearance. Neurologic findings include decreased vibration sense and decreased muscle strength in her lower extremities. A peripheral blood smear is shown in the image. Which of the following is the most likely cause of this patient's disorder? A.Bacterial overgrowth B.Dietary deficiency C.Gastrectomy D.Infection with Diphyllobothrium latum E.Pancreatic insufficiency F.Pernicious anemia

F.Pernicious anemia In patients without a clear travel history or dietary deficiency (eg, veganism), the most common cause of cobalamin deficiency is pernicious anemia, a disorder in which dietary cobalamin is not absorbed as a result of gastric parietal cell atrophy and the subsequent absence of intrinsic factor. It is reversible in its early stages by administration of vitamin B12, but without B12 replacement, the neurologic changes will eventually become permanent.

Which of the following genetic mechanisms is the most likely cause of chronic granulomatous disease? A.Anticipation B.Incomplete penetrance C.Loss of heterozygosity D.Mosaicism E.Pleiotropy F.Skewed X-linked inactivation G.Variable expressivity

F.Skewed X-linked inactivation CGD is inherited in an X-linked recessive manner. It is passed on the X chromosome from parents to their offspring. As females have two X chromosomes, they are carriers of disease, but do not express the disease. Males that inherit the mutated X chromosome subsequently express the disease. Thus, CGD is commonly seen almost exclusively in males.

The graph illustrates a typical action potential in the sinoatrial (SA) node. Each phase of the trace can be attributed to the conductance of ions through their respective channels. Which of the following drugs has the most significant effect on the flow of ions through voltage-gated channels during phase 0 of the pacemaker action potential? A.Adenosine B.Atropine C.Nifedipine D.Procainamide E.Propranolol F.Verapamil

F.Verapamil For a pacemaker action potential, phase 0 upstroke is caused by the opening of voltage-gated calcium channels, allowing an influx of calcium. Phase 3 downstroke is caused by inactivation of calcium channels and activation of potassium channels, causing an efflux of potassium. Phase 4 (funny current) is caused by a mixed sodium/potassium inward current and T-type calcium channels which contributes to further depolarization, accounting for the automaticity of SA and atrioventricular (AV) nodes.

Modafinil MOA

Inhibit norepinephrine and dopamine reuptake is used to treat narcolepsy, although its exact mechanism of action is unknown. Patients suffering from cataplexy may also be treated with sodium oxybate.

Pt presents with pulmonary embolism and hyperventilation. How are pH? PCO2? PO2? HCO3?

incr, decr, decr, decr (E)

what hypersentivity is atopic dermatitis vs contact?

1 vs 4

what is wrong in type 1 vs type 2 vs type 4 renal tubular acidosis?

1- H secretion in distal tubule is off 2- no bicarb reabsorption in PCT 4- hypoaldosteronism or lack of response

what do the brachial pouches develop into?

1- ear 2- tonsil 3- dorsal - inferior PT gland (bottom) 3- ventral - thyroid (TO) 4- superior PT gland (top)

what marker is increased in a 21OH deficiency?

17 hydroxyprogesterone

what is the second gen cephalosporin mnemonic?

2nd graders wear FAke FOX FUR to TEa parties ceFAclor ceFOXitin ceFURoxime cefoTEtan

which two hemophilias are clinically identical?

A- V111 B- 1X

A 43-year-old woman with a history of lupus developed a steroid-nonresponsive nephropathy. As a result, she underwent kidney transplantation 1 year ago. She is now on long-term immunosuppressant therapy. Which of the following medications is she most likely taking? A.5-Fluorouracil B.Azathioprine C.Cyclophosphamide D.Cytarabine E.Methotrexate

B.Azathioprine The patient must be on a long-term immunosuppressant to decrease the risk of organ rejection after her kidney transplant. Azathioprine is used to reduce the risk of acute graft rejection after kidney transplantation and to treat autoimmune disorders such as glomerulonephritis and rheumatoid arthritis. It interferes with purine nucleotide (adenine, guanine) synthesis and hence DNA synthesis and repair. In the body, hepatic metabolism converts azathioprine to 6-mercaptopurine; the two drugs therefore have the same mechanism of action.

A 12-month-old boy has been hospitalized multiple times for recurrent infections, including otitis media, pneumonia, and upper respiratory infections. Further family history reveals recurrent infections in males at a young age. Laboratory studies show a decreased IgG level and undetectable levels of IgM and IgA. What is the disease and the first line preferred treatment? A.Bruton agammaglobulinemia; bone marrow transplant B.Bruton agammaglobulinemia; intravenous immunoglobulin C.Wiskott-Aldrich syndrome; antibiotics D.Wiskott-Aldrich syndrome; bone marrow transplant E.Wiskott-Aldrich syndrome; intravenous immunoglobulin We Value Your Feedback! thumb_upthumb_down You correctly chose B [ 53% ]

B.Bruton agammaglobulinemia; intravenous immunoglobulin The condition is due to a defect in the Bruton tyrosine kinase (BTK) gene, which halts all B lymphocyte differentiation and is associated with low levels of all classes of immunoglobulins. Patients with Bruton agammaglobulinemia often experience recurrent bacterial infections after 6 months of age, when maternal IgG levels decline. The disease is treated with intravenous immunoglobulin; there is no curative therapy for this condition.

His mother was a heavy drinker, and he does not want to "go through what she went through." He does not think he drinks like his mother though, so he does not need to cut back yet. What stage of change is the patient in? A.Action B.Contemplation C.Maintenance D.Pre-contemplation E.Preparation F.Relapse

B.Contemplation Even though he says he doesn't want to "go through what she went through," and understands that his drinking is affecting his health, he does not believe he needs to cut back. In pre-contemplation, a patient does not yet recognize that they have a problem. This patient recognizes that his drinking is affecting his health, but does not yet feel the need to change his behavior. This is consistent with the second stage, contemplation.

Which of the following therapies is most likely to help correct the hyperkalemia? A.Administer bicarbonate B.Administer eplerenone C.Administer furosemide with normal saline D.Administer lisinopril with normal saline E.Large-volume fluid resuscitation with normal saline

C.Administer furosemide with normal saline Step-wise, to treat hyperkalemia, Give calcium gluconate to stabilize the cardiac cell membranes and prevent arrhythmias. Determine the cause of the hyperkalemia. Utilize short-term pharmacotherapy (insulin + glucose, ß-agonists, sodium bicarbonate) to shift K+ back into the intracellular environment Increase K+ excretion by giving normal saline with a loop diuretic, via a cation-exchange resin like sodium-polystyrene or by hemodialysis. Normal saline + furosemide work by increasing the excretion of Na+ and K+ in the loop of Henle.

A 57-year-old woman comes to the physician because of a fever of 101.8°F (38.8°C); her temperature increases during the day and decreases at night. She reports having night sweats as well as diffuse muscle and joint pain, and says that she feels tired all the time and has lost weight. The patient was previously in good health. Six weeks ago, she returned from a vacation to Greece where she spent her time visiting ruins, snorkeling, and trying the local specialties, including fresh goat cheese. One month ago, she adopted a newborn kitten from a local animal shelter. The patient's physical examination is notable for a strong moldy body odor, hepatosplenomegaly, and generalized lymphadenopathy. Which of the following organisms is most likely responsible for this patient's symptoms? A.Bartonella henselae B.Borrelia recurrentis C.Brucella melitensis D.Listeria monocytogenes E.Pasteurella multocida F.Plasmodium malariae

C.Brucella melitensis undulant fever or brucellosis, caused by infection with Brucella species. Brucella melitensis (B. melitensus) is a small, intracellular gram-negative coccobacillus which, like Nocardia, stains with a modified acid-fast stain. It is endemic to the Mediterranean basin, the Arabian peninsula, and India, as well as Mexico and Central and South America, making this patient's recent travel to Greece a likely setting for transmission. Clinical manifestations of brucellosis include those seen in this patient: undulating fever, weakness, and loss of appetite. Pertinent exam findings include generalized lymphadenopathy, hepatosplenomegaly, and a characteristic moldy body odor. Anti-LPS antibody titers are the most effective method of diagnosis. Blood cultures may also be used. Brucellosis can be treated with doxycycline, with the addition of rifampin in cases of relapse. Borrelia recurrentis, a spirochete, is spread between humans by a body louse. Infection may cause relapsing fever, which lasts about 3-5 days with 7-9 days between relapses. Hepatosplenomegaly can be a feature of Borrelia recurrentis infection; however, the patient would most likely present with additional features, such as jaundice, ecchymoses and petechiae, mental status changes, focal neurologic findings, or respiratory distress.

Which of the following has a role in the regulation of the rate-limiting step of carbohydrate metabolism? A.Acetyl-coenzyme A B.Alanine C.Citrate D.Glucose-6-phosphate E.Reduced nicotinamide adenine dinucleotide

C.Citrate The rate-limiting step of glycolysis is the conversion of fructose-6-phosphate to fructose-1,6-bisphosphate by the enzyme phosphofructokinase. Citrate is an intermediate produced in the Krebs cycle that has a role in the regulation of glycolysis by inhibiting the action of phosphofructokinase.

A 50-year-old man presents to his primary care physician with gradual onset of shortness of breath on exertion and a dry cough, both of which have worsened over the past year. He denies nosebleeds. He has a 20-pack-year history of cigarette smoking. He is a plumber by trade. Physical examination reveals mild hypoxemia and a bluish tinge to his skin and mucous membranes. Arterial blood gas values on room air are: pH, 7.42; partial pressure of carbon dioxide, 42; and partial pressure of oxygen, 79. Pulmonary function testing reveals increased physiologic dead space and decreased total lung capacity. Carbonide monoxide diffusion in the lungs (DLCO) is 50% of predicted value. A high-resolution CT scan of the chest is performed and demonstrates diffuse ground-glass opacities with some cystic changes. Which of the following is the most likely diagnosis? A.Acute respiratory distress syndrome B.Chronic obstructive pulmonary disease C.Desquamative interstitial pneumonia D.Granulomatosis with polyangiitis E.Silicosis

C.Desquamative interstitial pneumonia The name desquamative interstitial pneumonia is misleading. No desquamation of alveolar epithelium is present; instead, the cells that fill the alveoli are pigment-laden macrophages. This patient, with gradual onset of shortness of breath on exertion and a dry cough of a year's duration, presents with hypoxemia and a bluish tinge to the skin and mucous membranes. These symptoms, in combination with findings on a CT scan of diffuse ground-glass opacities and cystic changes, suggest a likely diagnosis of desquamative interstitial pneumonia (DIP). This rare disorder manifests clinically as gradual onset of dyspnea in a middle-aged patient with a history of cigarette smoking. Distribution can be similar to idiopathic pulmonary fibrosis, involving the basilar peripheral lung parenchyma, and typically follows a restrictive lung disease pattern. However, it is important to note that DIP is distinct from usual interstitial pneumonia (UIP), both pathologically and clinically. A CT scan of the chest of a patient with DIP does not usually show the honeycombing seen in UIP. Instead, diffuse ground-glass opacities are present, usually involving the lower lobes and sometimes with small superimposed cysts (see images). Clinically, there will be a mild decrease in total lung volume and a decrease in diffusion capacity. Forced expiratory volume in 1 second (FEV1) and forced vital capacity (FVC) will both be decreased, but the ratio of FEV1 to FVC will be >80% because of a larger decrease in FVC compared with FEV1. About 70% of patients with DIP survive for 10 or more years, with smoking cessation and corticosteroid treatment resulting in clinical improvement.

She has a 20-year history of rheumatoid arthritis and was treated with methotrexate. However, she stopped taking the medication after experiencing several adverse effects. She has had persistent pain in her fingers and knees since then. Which of the following medications will provide long-term treatment for her rheumatoid arthritis? A.Alendronate B.Celecoxib C.Etanercept D.Glucocorticoids E.Indomethacin F.Naproxen

C.Etanercept Rheumatoid arthritis is an autoimmune disorder that attacks the joints, most commonly the hands and knees. It usually appears in the second or third decade of life. Etanercept is an injectable drug used for rheumatoid arthritis that works by blocking TNF-alpha binding. It is a good choice for chronic rheumatologic conditions.

A 10-year-old boy is brought to the pediatrician for the first time by his parents. The parents report that he has had frequent nosebleeds, bleeding gums when he brushes his teeth, and frequent sinus and skin infections. He says he often has episodes where he "feels dizzy" and his "chest feels funny." On examination, the child appears pale and is found to have small and wide thumbs, small eyes, and pigmented spots on his abdomen. Laboratory results are as follows:Erythrocyte count: 3.5 x 106/mm3 Hemoglobin: 9.7 Hematocrit: 31% Leukocyte count: 2.5 x 103/mm3 Platelet count: 70 x 103/mm3 What congenital disorder is most consistent with this patient's presentation? A.Diamond-Blackfan anemia B.Edwards syndrome C.Fanconi anemia D.Hereditary spherocytosis E.Neurofibromatosis type 1

C.Fanconi anemia Fanconi anemia is the most common cause of hereditary aplastic anemia and is an important cause of acute myeloid leukemia in children. It is characterized by defective DNA repair that results in increased chromosomal breakage, rearrangements, and deletions. Patients with this condition are particularly sensitive to alkylating agents. Other characteristics of Fanconi anemia include predisposition to malignancy, and physical abnormalities, including short stature, microcephaly, developmental delay, and café-au-lait skin lesions. Radial ray abnormalities result in abnormal or absent radii or thumbs. Kidney malformations, hypogonadism, microcephaly, and high fetal hemoglobin concentration (relative to normal) are often seen as well.

A 66-year-old man is being treated for a Pseudomonas aeruginosa infection. His vital signs are: heart rate, 90; blood pressure, 130/84 mm Hg; respiratory rate, 14; and temperature, 101.5°F (38.6°C). Urinalysis reveals the presence of muddy brown casts. After 1 week of antibiotic therapy, his urine production drops significantly. Laboratory studies show: BUN: 26 mg/dLSerum creatinine: 2.4 mg/dL Which of the following drugs most likely caused this patient's condition? A.Chloramphenicol B.Erythromycin C.Gentamicin D.Meropenem E.Oxacillin

C.Gentamicin nephrotoxicity associated with aminoglycoside administration. Aminoglycosides such as gentamicin, tobramycin, and streptomycin effectively treat gram-negative rod infections. Other adverse effects of the aminoglycosides include ototoxicity and neuromuscular blockade.

A 37-year-old woman (gravida, 2 para 2) comes to her gynecologist complaining of a 6-month history of heavy and painful menstrual bleeding.She reports that she has regular cycles every 27 to 31 days and does not take oral contraceptives. The pain is worse during her cycles and does not typically occur between cycles, with bowel movements, or during sex. She underwent menarche at age 12 and her cycles/period have been unremarkable prior to the past 6 months. Her first child was delivered by spontaneous vaginal delivery, and her second child was delivered by emergency cesarean section for severe preeclampsia. On physical examination, her vitals are within normal limits, BMI is 24, and her abdomen is soft and nontender without palpable masses. Adnexae are small and nontender. Her uterus is soft and globally enlarged. A transvaginal ultrasound confirms a diffusely large uterus and reveals a 2-cm thin-walled cyst with fluid in her right ovary. Endometrial stripe is 3 cm (normal < 5 cm). Which of the following is the most likely cause of this patient's symptoms? A.Endometrial gland proliferation present on ovary B.Excessive endometrial gland proliferation within uterus C.Glandular tissue in the myometrium D.Hyperplastic growth from uterine stroma E.Smooth muscle proliferation in the myometrium

C.Glandular tissue in the myometrium Adenomyosis is the extension of endometrial (glandular) tissue into the myometrium. The extension of tissue is often secondary to hyperplasia of the basal layer of endometrium or idiopathic endomyometrium invagination. Clinical clues to the diagnosis include uniform uterine enlargement and softening(ie, boggy) and can be confirmed by endometrial biopsy or transvaginal ultrasound. Definitive treatment for adenomyosis is hysterectomy, but the disorder can be managed symptomatically with gonadotropin-releasing hormone agonists. Endometriosis is a disorder of ectopic endometrial tissue that is frequently found on the ovary ("chocolate ovary"). Although this patient's ultrasound reveals a cyst on her ovary, a small, thin-walled cyst is often benign. Additionally, endometriosis is clinically associated with cyclic pelvic pain (worse a few days before menstrual cycles), dyspareunia, dyschezia, or infertility. The patient denies experiencing pain during sex or with bowel movements, nor does she appear to be infertile. Ovary Ultrasound Cyst Endometriosis Cyclic Pelvic pain Menstrual cycle Dyspareunia Infertility Bowel movement Then correctly chose C [ 29% ]

A 55-year-old black man comes for a routine medical examination. He has no history of serous illness and takes no medications. His last visit to a doctor was 3 years ago. He reports smoking ½ pack of cigarettes per day for the past 40 years. The patient denies any physical problems but says has been checking his blood pressure at home during the past month and has been getting readings usually higher than 145/90 mm Hg. On evaluation today, he is afebrile; blood pressure is 147/98 mm Hg, pulse is 62/min, and respiratory rate 18/min. His BMI is 29.8 kg/m2. Physical examination is significant for truncal obesity. In addition to educating the patient on lifestyle modifications, which of the following is the most appropriate next step in management? A.Fenoldopam B.Hydralazine C.Hydrochlorothiazide D.Labetalol E.Lisinopril F.Losartan

C.Hydrochlorothiazide Given his race, thiazide diuretics (eg, hydrochlorothiazide) or calcium channel blockers (eg, amlodipine) are the first-line pharmacologic treatment options.

A 17-year-old boy comes to the clinic due to concerns about acne on his face. He mentions that he had similar episodes when completing school projects and studying for semester examinations. He does not take any medications or use recreational drugs. His vital signs are within normal limits. On physical examination, erythematous papulopustular lesions are present on his forehead, cheeks, and jaw. He is appropriately prescribed medication for his symptoms. This patient presents with erythematous papulopustular lesions on his face only. Which of the following is the most likely mechanism of action of the treatment provided? A.Binds to 30S subunit and prevents attachment of aminoacyl-tRNA B.Formation of toxic free radicals. C.Increased epidermal keratinocyte turnover D.Inhibits bacterial isoleucyl-tRNA synthetase.

C.Increased epidermal keratinocyte turnover This 17-year-old boy presents with erythematous papulopustular lesions on his face, which have occurred in the past during stressful conditions (completing school projects and studying for semester exams). These lesions are seen predominantly on the forehead, cheeks and jaw. Therefore, the most likely diagnosis is mild acne vulgaris. Acne vulgaris is a common skin condition with multifactorial etiology which affects most individuals at some point in their lives. It is caused by increased sebum production which is stimulated by androgens and decreased keratinocyte desquamation, leading to obstruction of the pilosebaceous unit. When the unit is obstructed, Cutibacterium acnes is the most common organism to colonize the unit. This causes inflammation resulting in papules/pustules, nodules and cysts. Cutibacterium is part of the normal skin flora. Multiple factors are taken into consideration when initiating pharmacological therapy. These include the severity and type of acne, concern for adverse effects (teratogenicity) and patient adherence. Since this patient is suffering from mild acne vulgaris, the most likely treatment would include topical retinoids. Retinoids act by increasing epidermal keratinocyte turnover that facilitates desquamation. As a result, the keratinocyte plugs are removed and obstruction is relieved. They also reduce the size of the sebaceous glands and inhibit sebum production. In women of childbearing age, a serum or urine pregnancy test should be administered and two modes of contraception is recommended before initiating topical retinoids

A 33-year-old woman comes to her physician's office because of heat intolerance, anxiety, and palpitations. She also reports an unintentional 6.8-kg (15-lb) weight loss over the past 3 months. She reports no change in her diet or exercise habits during this time. The patient's medical history is unremarkable, and she takes no medications regularly. She weighs 53.5 kg (118 lb). She is thin but in no acute distress; she has bulging eyes. Pulse is 109/min and blood pressure is 128/83 mm Hg. On physical examination, a thick, erythematous rash is observed on the anterior side of the lower extremities. Results of laboratory studies show a decreased concentration of thyroid-stimulating hormone (TSH) and an increased concentration of free thyroxine (T4). Results of a radioactive iodine uptake study reveal diffuse uptake in the thyroid gland. If her condition is left untreated, this patient is most at risk for development of which of the following? A.Hypercholesterolemia B.Increased bone density C.Increased fractures D.Non-Hodgkin lymphoma E.Spontaneous abortion in future pregnancies

C.Increased fractures Patients with untreated Graves disease are also at risk for osteoporosis and thus increased fractures. An increased thyroxine level stimulates bone resorption by increasing osteoclast activity, resulting in increased porosity of cortical bone and reduced volume of trabecular bone. The increased bone resorption leads to increased serum calcium. Increased calcium levels inhibit parathyroid hormone secretion and conversion of 25-hydroxyvitamin D to 1,25-dihydroxyvitamin D. Alkaline phosphatase levels are expected to be elevated because of increased bone turnover.

Which of the following classes of drug is the best choice to treat Alzheimer? A.Cyclooxygenase inhibitor B.Dopamine reuptake inhibitor C.NMDA receptor antagonist D.No pharmacologic treatment available E.Serotonin reuptake inhibitor

C.NMDA receptor antagonist Moderate-to-severe AD is an indication for memantine, an N-methyl-D-aspartate (NMDA) receptor antagonist. This drug blocks excitatory neurotoxicity, which is mediated by calcium influx, to prevent further neuronal damage.

A 30-year-old man presents with complaints of right neck pain and tingling that radiates into the middle finger on his right hand. Neurologic examination reveals weakness of the right triceps and an absent right triceps reflex. His history is significant for a skiing accident in which he injured his neck a year earlier. Which of the following lesions is the most likely cause of his symptoms? A.Right C4-C5 disc herniation B.Right C5-C6 disc herniation C.Right C6-C7 disc herniation D.Right axillary nerve compression at the humeral head E.Right radial nerve compression at the midshaft of the humerus F.Right ulnar nerve compression at the elbow

C.Right C6-C7 disc herniation Because cervical nerve roots emerge above the corresponding vertebrae (unlike other nerve roots that emerge below), a right C6-C7 disc herniation would impinge on the right C7 nerve root.

which gut hormone decreases gastric emptying time?

CCK

what CD markers do reed sternberg cells have?

CD15 and CD30 2 nuclei x 15 = 30

which markers are on NK cells?

CD16 and CD56

what are the markers on B cells?

CD19, 20, and 21

which cancer undergoes richter transformation to diffuse large b cell lymphoma?

CLL/SLL

Tumor metastasis to which of the following sites is most likely responsible for this patient's carcinoid syndrome secondary to carcinoid tumor? A.Adrenal cortex B.Adrenal medulla C.Kidney D.Liver E.Small intestine F.Thyroid

D.Liver Although the primary carcinoid tumor is often located in the small intestine, particularly in the midgut (jejunum, ileum, cecum), these tumors do not cause the symptoms of carcinoid syndrome until they metastasize to the liver.

If the cut-off for HbA1c is increased to ≥7% for the diagnosis of diabetes mellitus, which of the following best describes how would it affect the sensitivity and specificity of this test? A.Sensitivity and specificity will decrease B.Sensitivity and specificity will increase C.Sensitivity and specificity will not change D.Sensitivity will decrease and specificity will increase E.Sensitivity will increase and specificity will decrease

D.Sensitivity will decrease and specificity will increase

how can acute promyelocytic leukemia present? what is the treatment and why?

DIC (when cancer cells lyse) all trans retinoic acid forces promyelocytes to differentiate

Which of the following is the mechanism of action of the drug most likely prescribed for streptococcal pharyngitis? A.Blocks peptidoglycan polymerization B.Blocks protein synthesis at the 30S ribosomal subunit C.Blocks protein synthesis at the 50S ribosomal subunit D.Inhibits DNA gyrase E.Inhibits peptidoglycan polymer cross-linking

E.Inhibits peptidoglycan polymer cross-linking Penicillins are β-lactam antibiotics that bind to and inactivate proteins present on the bacterial cell membrane, blocking cross-linking of peptidoglycan and preventing cell wall synthesis. These proteins are called penicillin-binding proteins. They also activate autolytic enzymes. Streptococci remain sensitive to penicillin V, making it the first-line choice for treatment. In young children; amoxicillin may be preferred since its oral suspension is more palatable.

Which of the following characteristics of the anesthesia drug most likely to have been administered explains this patient's rapid recovery? A.Hepatic removal of drug B.Highly water soluble C.Increased renal excretion D.Rapid glucuronidation E.Readily redistributed

E.Readily redistributed propofol is then readily redistributed to other organs (eg, skeletal muscle and fat), and its action is quickly terminated because of its high lipophilicity.

Celiac disease is associated with which of the following conditions? A.Acute lymphoblastic leukemia B.Colonic polyp C.Diverticula D.Hepatic abscess E.Selective IgA deficiency

E.Selective IgA deficiency Patients typically present with abdominal pain, weight loss, and changes in the consistency of their stools. They are also more likely to have iron-deficiency anemia, autoimmune thyroid disease, and type 1 diabetes. Celiac disease is associated with IgA deficiency. Patients with IgA deficiency are at risk for recurrent infections and anaphylaxis when receiving blood products.

A 55-year-old man with a medical history of autoimmune thyroiditis visits the clinic. He complains of 3 months of epigastric discomfort and nausea. He further complains of new fatigue and a sensation of tingling in his toes and lips. After a course of omeprazole failed to relieve his symptoms, an esophagogastroduodenoscopy (EGD) is performed, which reveals a chronic inflammatory process affecting the fundus and body of the stomach, but sparing the antrum. The patient is a nonsmoker who rarely consumes alcoholic beverages. He does not use aspirin or nonsteroidal anti-inflammatory drugs. He denies having loose stools or constipation. Which of the following is the most likely etiology of this patient's disease? A.Antibodies to gliadin B.Gastrin-secreting tumor C.Infection with Helicobacter pylori D.Infection with Tropheryma whipplei E.Self-reactive CD4+ T cells

E.Self-reactive CD4+ T cells These symptoms suggest that he has pernicious anemia. This disease occurs as a result of autoimmune destruction and inflammation of the stomach known as autoimmune gastritis (type A gastritis), which primarily affects the fundus and body but spares the antrum (see image).Type A gastritis results from CD4+ (helper) T cell-mediated immunity directed against parietal cells (by destruction of H+-K+ ATPase). Autoimmune gastritis is also characterized by reduced levels of serum pepsinogen 1.

A 25-year-old woman visits her physician and reports a 3-week history of lower abdominal cramps and intermittent bloody stools that occur twice per day. She has not had fevers or contact with sick persons. Also, she has not traveled outside the country over the past year and does not recall eating anything out of the ordinary. Her laboratory studies show the following results:Hemoglobin: 13.0 g/dLHematocrit: 39%Leukocyte count: 6000/mm3Platelet count: 200,000/mm3Erythrocyte sedimentation rate: 35 mm/h Which of the following is the most important initial treatment for this patient's condition? A.Emergent surgery B.Infliximab C.Loperamide D.Rituximab E.Sulfasalazine

E.Sulfasalazine This patient presents with abdominal pain, intermittent bloody stools, and a mildly elevated erythrocyte sedimentation rate. She has no history of exposure to infection (travel, food, contact with sick persons). Together, the symptoms and history suggest a possible diagnosis of ulcerative colitis. Ulcerative colitis is more common in patients with Ashkenazi Jewish ancestry and those with a family history of the disease.

The mechanism of action of gemfibrozil is best described by which of the following? A.Increases expression of triglyceride receptors B.Inhibits peroxisome proliferator-activated receptor a nuclear transcription (PPAR-α) regulator C.Promotes hepatocyte lipolysis D.Slows the conversion of VLDL to LDL cholesterol E.Upregulates lipoprotein lipase

E.Upregulates lipoprotein lipase Fibrates work by PPAR-α activation which in turn promotes synthesis of lipoprotein lipase.

A 71-year-old patient with a history of multiple vaginal deliveries and stroke presents with a urinary urgency ("sudden urge" to urinate) in the setting of low post-residual volume (normal <150 ml). This is most suggestive of urge incontinence. Which of the following is the most likely cause of the patient's condition? A.Detrusor muscle weakness B.Functional incontinence C.Medication side effect D.Pelvic floor laxity E.Uninhibited bladder contraction

E.Uninhibited bladder contraction Urge incontinence is the result of detrusor muscle overactivity. Patients present with sudden urges to void and may endorse a history of dribbling or complete soaking of their garments. Urge incontinence can be idiopathic or can result from neurological damage (causing detrusor hyperactivity due to decreased activity of descending spinal inhibitory pathways) or urinary tract infection. A low post-residual volume, which indicates lack of obstructive/overflow pathology, often supports this diagnosis. Why the incorrect answers are wrong: Detrusor muscle weakness describes overflow incontinence, which can be seen in patients with neurologic disease (ie, history of stroke). Patients present with urinary dribbling in the setting of a high post-void residual volume rather than a sudden urge to urinate and low post-residual volume, as seen in this patient. Pelvic floor laxity is a common cause of stress incontinence. Patients with a history of multiple vaginal births or a complicated gynecologic history may have incontinence secondary to pelvic floor damage or urethral hypermobility. However, stress incontinence is typically characterized as leakage of urine with increased intra-abdominal pressure (ie, coughing), not incontinence with sudden uncontrollable urges, as seen in this patient. Although the patient is taking diphenhydramine, this medication's anticholinergic activity would cause urinary retention (and may even present similarly to overflow incontinence, described above). Thus, medication side effect is unlikely to be the cause in this case. Functional incontinence refers to an inability to physically transport oneself to the bathroom. There is no underlying urinary tract pathology.

A 44-year-old man presents to the clinic with complaints of severe headaches which began 3 weeks ago. He describes the headaches as occurring primarily on the right side of his head, concentrated around his right eye and right temple. Patient mentions that during these episodes his right eye turns red and his right eyelid drops. He reports that the headaches tend to occur while he is asleep and wake him repeatedly in the early morning. They last anywhere from 30 minutes to 2 hours. The patient is fearful that he might have a brain tumor. Results of ophthalmologic examination, including fundoscopy, are normal. What is the best next step in management of this patient's condition? A.Computed tomography scan of the brain B.Electroencephalography C.Lumbar puncture D.Nonsteroidal anti-inflammatory agent E.Verapamil

E.Verapamil This patient presents with recurrent, severe, and unilateral pain localized around the eye, which indicates cluster headaches. The best next step to managing this patient's condition would be the use of verapamil, a calcium channel blocker that has proven effective in preventing migraines and cluster headaches.

what virus is hodgkin lymphoma associated with?

EBV

what HLA is RA associated with?

HLA-DR4 4 walls in a RHEUM

what are the two common causes of SCID and what are inheritances?

IL-2 deficiency - most common - x linked ADA deficiency - AR

which amino acid functions to donate methyl groups?

Methionine

which enzyme is deficient in chronic granulomatous disease?

NADPH oxidase

which test has capillary transfer of RNA from an electrophoresis gel to a membrane?

Northern blot

deductible

a specified cost that the insured patient must pay before the insurance will pay a claim

what causes damage to RBC in beta thalasemmia major?

alpha homotetramers

when can children use two word sentences?

by 2

how to tell cholangitis from cholelithiasis?

charcots triad - use it RUQ pain fever jaundice

what types of drugs have a low Vd?

charged, large protein bound drugs that stay in the vasculature vd = drug in body/ drug in plasma

which antipsychotic causes agranulocytosis?

clozapine

which atypical antipsychotic causes agranulocytosis?

clozapine

cardiac abnormality in turner's syndrome?

coarctation of the aorta

two cardiac sxs in turners syndrome?

coarctation of the aorta bicuspid aortic valve

what are the symptoms of I-cell disease?

coarse face joint issues clouded cornea high lysosomal enzymes

what do loop diuretics do to calcium levels?

decrease

what is the MOA of niacin?

decrease hepatic VLDL and TG synthesis

what does azathioprine do? what drug can it not be taken with due to toxic metabolite being madE?

decrease purine synthesis allopurinol (xanthine oxidase inhibitor, which normally breaks down the toxic metabolite

what does glucose do to cAMP levels in the lac operon?

decreases

what does leuprolide do to sex hormone levels?

decreases (GnRH agonist used continously will inhibit)

how does excercise change total peripheral resistance?

decreases it

what does bisphosphoglycerate do to Hb affinity?

decreases it

which nerve dorsiflexes the foot?

deep peroneal nerve Deep Dorsflexes

what are brachial pouches derived from?

endoderm

two protozoal causes of liver abscesses?

entamoeba eccinococcus

how does growth hormone cause hyperglycemia?

insulin resistance

which muscle opens the jaw?

lateral pterygoid lateral lowers

what happens to acetyl coa with increased NADH/NAD?

made into ketones (not enough oxaloacetate to combine with to make citrate

Diffuse ST-segment elevation on ECG =?

pericarditis

which amino acid deficiencies could cause deficient thyroid homrone?

phenylalanine tyrosine (made from phenylalainine

where do superior rectal veins drain?

portal vein

what do muscle fibers appear like in mitrochondiral myopathies?

ragged and red

does basal cell carcinomas metastasize?

rarely

what symptoms are present in all NNRTIs for HIV?

rash and hepatotoxicity

how does CO shift the Hb curve?

right

ocreotide MOA?

somatostatin

which dopamine pathway controls prolactin?

tuberoinfundibular

what causes Thrombotic thrombocytopenia purpura?

vWF protease no longer works

what are the symptoms of WAGR complex?

wilms aniridia (part of iris missing) GU malformations Retardation

do both 21 OH and 11 OH deficiencies cause zona reticularis hyperplasia?

yes

does multiple myeloma increase infection risk'?

yes

which hypersensitivity are the vasculitis?

3

when does HbH form?

3 allele alpha thalasemia deletion

what brachial pouches dont develop in DiGeorge?

3rd and 4th

A 6-year-old boy is brought to the clinic by his mother because of a 2-week history of a persistent headache and malaise. The mother report that she found a tick on the boy after their family went hiking in New York 3 weeks ago . No other family members have symptoms. On evaluation today, his temperature is 38.3°C (101°F). On physical examination, there is an erythematous rash with central clearing on the anterior tibia. There is no meningismus; neurological examination shows no abnormalities. Which is the most appropriate treatment for this patient? A. Amoxicillin B. Azithromycin C. Cephalexin D. Doxycycline E. Metronidazole F. Non-steroidal anti-inflammatory drugs

A. Amoxicillin First-line treatment of Lyme disease is doxycycline, but tetracyclines are contraindicated in pregnant women because of the teratogenic effects and in children younger than 8 years because of teeth discoloration and disruption of bone development, particularly when used repeatedly or for a long period of time. For these reasons, this patient should be treated with amoxicillin.

A 55-year-old man goes to a busy emergency department because of persistent pain in his upper abdomen, fever, nausea, and dark stools. An esophagogastroduodenoscopy shows gastric ulcer; a Helicobacter pylori urea breath test is positive. After the patient is stabilized, the physician writes a prescription for clarithromycin, amoxicillin, and omeprazole before discharging him from the hospital. Which of the following best represents a latent error that could affect this patient's recovery? A.A handwritten prescription by the physician B.Difficulty communicating with a patient due to language barrier C.Missing physician during emergency D.Physician lacks adequate knowledge of the medical condition E.Violation of hospital rules by a nurse

A.A handwritten prescription by the physician Latent errors are defined as conditions in a healthcare setting that contribute to an error that can cause harm to patients. Poor design of elements in a healthcare system produce the latent conditions, which result in patient safety incidents. Latent conditions are accidents waiting to happen; for example, fewer clinical pharmacists on the wards serves as a latent condition. It would not be noticed until an unsafe medication order was prescribed and then, left unchecked, was carried out. Similarly, limited staff in emergency department would not be noticed until increased workload created an unsafe environment. The other answers are incorrect for the following reasons: Difficulty communicating with a patient due to a language barrier and lack of knowledge of the medical condition on the part of the physician are error-producing factors, that is, they contribute to the possibility that an error may occur A missing physician in an emergency situation is an example of lack of defenses factor Violation of hospital rules by a nurse is an example of an active error that can result in harm to the patient.

A research group is studying different groups of carrier proteins. They decide to focus their studies on a specific protein called neurophysin I and create a knockout mouse to study the effects on mice. Which of the following is most likely to occur as a result of the experiment? A.Absent let-down reflex B.Hypocalcemia C.Hypocortisolism D.Hypothyroidism E.SIADH

A.Absent let-down reflex The protein neurophysin I is essential for the transport of oxytocin from the paraventricular nucleus. Female mice without the protein are unable to release oxytocin and thus have absent let-down reflex and difficulty feeding their pups, as oxytocin facilitates milk secretion (and also stimulates uterine contractions during labor). Oxytocin and ADH (vasopressin), which mediates water absorption at the renal collecting ducts, are synthesized by the neurons of the supraoptic and paraventricular nuclei in the hypothalamus. They are transported to the posterior pituitary gland on direct axonal connections called the supraoptic hypophyseal tract, where they are stored and eventually released into the capillaries draining into the hypophyseal vein. The transport of oxytocin and ADH from the hypothalamus requires carrier proteins. Neurophysin I is the transporter protein for oxytocin, whereas ADH is transported by neurophysin II.

A 60-year-old woman comes to her primary care provider to discuss management of her type 2 diabetes mellitus. She has been taking metformin as prescribed without problems, but still has a markedly elevated A1c level. Her physician decides to initiate an additional agent for better glycemiccontrol. In a few weeks, the patient returns to her physician for a follow-up appointment and reports diarrhea and flatulence. On laboratory studies, her liver enzyme levels are elevated compared with her baseline levels. The physician reassures the patient that this is one of the most common adverse effects of this therapeutic agent. Which of the following agents is most likely responsible for this patient's new complaints? A.Acarbose B.Canagliflozin C.Chlorpropamide D.Exenatide E.Glipizide F.Metformin G.Orlistat

A.Acarbose Acarbose is an α-glucosidase inhibitor that decreases the hydrolysis and absorption of disaccharides and polysaccharides at the intestinal brush border, which reduces postprandial hyperglycemia. This drug can be used as monotherapy or in combination with oral hypoglycemic medications in the management of type 2 diabetes mellitus. Acarbose commonly causes GI-related adverse effects including abdominal cramps, diarrhea, and flatulence. Elevation of transaminases also is an effect of acarbose. Acarbose is contraindicated in patients with cirrhosis and requires frequent monitoring of liver function.

A 22-year-old college student presents to the emergency department with sudden-onset severe epigastric pain that radiates to the back. He has a history of heavy alcohol abuse, and lab tests show a markedly elevated amylase level.The organ involved in this patient's pathology plays a key role in normal gastrointestinal physiology. Under normal physiologic conditions, the organ is stimulated by a hormone that increases the secretion of bicarbonate. Which of the following will increase release of this hormone? A.Acid and fatty acids in the duodenal lumen B.Fasting state C.Increased stomach distention D.The presence of fatty acids, amino acids, and oral glucose in the duodenum and jejunum E.Vagal stimulation

A.Acid and fatty acids in the duodenal lumen The hormone secretin acts on pancreatic duct cells to stimulate bicarbonate secretion, which facilitates functioning of pancreatic enzymes by neutralizing gastric acid. Secretin is released by duodenal S cells in response to increased acid and fatty acids in the lumen of the duodenum.

A 16-year-old boy with a history of nosebleeds and frequent bruising comes to his pediatrician because he recently began passing blood-tinged stool. Physical examination reveals mild gingival bleeding. Laboratory studies show: Platelets: 250 × 103/ μLProthrombin time: 13 secPartial thromboplastin time: 49 secBleeding time: 15 min Which of the following is the function of the protein most likely mutated in this patient? A.Acts as a ligand in platelet adhesion B.Acts as a receptor for fibrinogen C.Acts as a receptor for von Willebrand factor D.Cleaves large von Willebrand factor multimers to proper size E.Inactivates factors V and VIII

A.Acts as a ligand in platelet adhesion This boy has nosebleeds and gingival bleeding, frequently bruises, and has blood in his stool. His lab tests show an elevated bleeding time (BT) and a slightly elevated partial thromboplastin time (PTT). All of these signs and symptoms are clues to a bleeding disorder.His physical exam only reveals gingival bleeding. The lack of petechiae and purpura make a pure platelet disorder less likely. Coagulation disorders such as hemophilia or vitamin K deficiency are possibilities, but again, they aren't terribly common. Hemophilia usually has a family history of disease presentation, and vitamin K deficiency is found in patients with dietary restrictions. Thus, it's best to think about the most common causes of bleeding disorders. The most common bleeding disorder is the mixed platelet and coagulation disorder von Willebrand disease (vWD).von Willebrand factor (vWF) is produced by the Weibel-Palade bodies of endothelial cells and acts as a ligand for platelet adhesion; it also plays a role in fibrin clot formation by acting as a carrier for factor VIII. The PTT may be normal or prolonged, depending on the degree of reduction of the factor VIII level. Lab tests may show a prolonged BT, despite adequate numbers of platelets, but BT is often normal in mild to moderate vWD.Defects in the glycoprotein IIb/IIIa receptor for fibrinogen are involved in Glanzmann thrombasthenia. Defects in the GpIb receptor (the primary receptor for vWF) are involved in Bernard-Soulier disease. ADAMTS13 metalloprotease cleaves vWF multimers to make them the appropriate size; without ADAMTS13, thrombotic thrombocytopenic purpura can result. Protein C inactivates factors V and VIII.

A 4-year-old boy is brought to his pediatrician's office for an annual check-up examination. He and his parents emigrated from Greece 2 years ago. The mother reports the boy has been healthy. On further questioning, she reports a personal history of anemia. In addition, she reports several other family members with similar problems. She is worried that her son may develop the same problems and wants him to undergo diagnostic testing. On physical examination, the boy is well developed and cooperative. No petechiae, purpura, ecchymosis, or conjunctival pallor are noted. Laboratory analysis reveals a hemoglobin of 11.5 mg/dL and hematocrit of 43%. Hemoglobin electrophoresis shows HbA2 at 7%. Genetic testing of the boy and his mother reveals different mutations in the beta hemoglobin gene. Which of the following most likely explains this finding? A.Allelic heterogeneity B.Heteroplasmy C.Incomplete penetrance D.Locus heterogeneity E.Mosaicism F.Uniparental disomy

A.Allelic heterogeneity Genetic testing of the boy and his mother yields different mutations in the same locus resulting in a similar phenotype. This phenomenon is known as allelic heterogeneity. Allelic heterogeneity has to be differentiated from locus heterogeneity, which results due to mutations at the different loci resulting in a similar phenotype.

A 2-year-old-boy is brought to the pediatrician by his parents after they discovered an enlarged, right-sided testicular mass during his bath. Transillumination of the scrotum does not reveal translucency, and cancer is suspected. After explaining to the parents that a biopsy can not be performed due to risk of tumor seeding of the biopsy site, the boy's parents agree to allow surgical removal of the mass and testicle. Pathologic examination of the excised tissue shows a yellow testicular mass with primitive cells resembling glomeruli on hematoxylin and eosin stain. Which of the following serum markers is most specific for this patient's testicular mass? A.Alpha-fetoprotein B.Human chorionic gonadotropin C.Lactate dehydrogenase D.Mucin E.Placental alkaline phosphatase F.Testosterone

A.Alpha-fetoprotein This child has a testicular mass with cells resembling glomeruli on H&E stain. The patient's presentation is consistent with a yolk sac (endodermal sinus) tumor, which most often affects children under the age of 4. The mnemonic "SAC" can be used to remember Schiller-Duval bodies, AFP, and Children.

A 60-year-old man who is hospitalized reports a 3-day history of burning on urination and a 1-day history of pain in the right flank. Last week, he underwent cystoscopy to check for bladder cancer. He has a 30-pack-year history of smoking. His medical history includes hypertension and dyslipidemia. On physical examination, his heart rate is 88, and his temperature is 38.3°C (101°F). Urine cultures show infection with Enterococcus faecalis. Susceptibility testing shows widespread susceptibility. Which of the following is the most appropriate pharmacotherapy for this patient? A.Ampicillin B.Ciprofloxacin C.Fosfomycin D.Nitrofurantoin E.Trimethoprim-sulfamethoxazole

A.Ampicillin Ampicillin monotherapy is appropriate, because it is the first-line treatment for susceptible Enterococcus infections. For susceptible strains, ampicillin is often preferred for the treatment of enterococcal complicated UTIs, bacteremia, and endocarditis. Isolates of E. faecalis are typically susceptible to ampicillin, whereas most strains of E. faecium are resistant to it. Susceptible enterococcal infections may also be treated with penicillin or vancomycin. The success of these antibiotics may be attributable to the lack of beta-lactamase in most enterococci. Some particularly hardy strains may resist vancomycin, in which case linezolid is commonly used.

An 89-year-old man is brought to his primary care physician by his two children. They say that their father was a reserved and good-natured person, but they are concerned about changes in his behavior over the past year. They say they can no longer bring him to church after he made lewd remarks to various acquaintances. At a recent family gathering, he was found trying to put his grandchildren's toy blocks into his mouth. His medical history is notable for two strokes over the last year. Physical exam is remarkable for residual weakness in his right arm, mild dysarthria, and gait unsteadiness. Testing of primitivereflexes is negative. Damage to which of the following regions would most likely result in the new symptoms observed in this patient? A.Amygdala B.Arcuate fasciculus C.Frontal lobe D.Mammillary bodies E.Right parietal lobe

A.Amygdala This patient's recent episodes of hypersexuality, hyperorality (fixation on oral exploration), and disinhibited behavior are suggestive of Klüver-Bucysyndrome, a constellation of symptoms that can also include visual agnosia, anterograde amnesia, and decreased emotional responsiveness. It results from insult to the amygdala bilaterally, often secondary to ischemic stroke. A history of previous strokes, as evidenced by this patient's clumsiness, ataxia, and dysarthria (characteristic of lacunar infarcts), is a strong risk factor for the development of Klüver-Bucy syndrome. Other causes of amygdala injury include infection (encephalitis) and trauma. Although patients with frontal lobe lesions may exhibit findings that are seen in this patient (hypersexuality and inappropriate speech), a frontal lobe lesion would result in the return of primitive reflexes, such as the snouting, Moro, grasp, and palmomental reflexes.

Which of the following mechanisms underlies the test that is used to screen for HIV? A.Antibodies in the host's serum bind to viral antigens in the test and are detected by an activity-based dye B.Capillary transfer of RNA from electrophoresis gel to a membrane C.Indirect microscopic detection of antigens using antibody fluorescence D.Nucleic acid amplification testing E.Sodium dodecyl sulfate-polyacrylamide gel electrophoresis (SDS-PAGE), protein transfer, and antibody-epitope interaction

A.Antibodies in the host's serum bind to viral antigens in the test and are detected by an activity-based dye This mechanism describes an enzyme-linked immunosorbent assay (ELISA), which tests for antibodies in the patient's serum using known antigens, or vice versa. In the case of HIV, it is used to determine whether a person's immune system has antibodies to HIV proteins.

A 40-year-old woman presents to her gynecologist with a lump in her left breast. She is not sure how long the lump has been there, but she first noticed it in the shower 2 weeks ago. She has never had a mammogram and notes that her mother died of breast cancer. On physical exam, the lump is located in the upper outer quadrant of the left breast, and it is very firm and immobile. Additionally, the patient has a few firm lymph nodes in her left axilla. A mammogram is obtained and shows calcifications in the area of the lump. A biopsy specimen is taken and it stains positively for an antibody against the protein produced by mutations in erbB2 (formerly referred to as HER-2/neu). She begins chemotherapy with a drug that is specific to this particular type of breast cancer. What is the mechanism of action of the drug she was most likely given? A.Antibody-dependent cytotoxicity B.Decreased secretion of LH and FSH from the anterior pituitary gland. C.Inhibition of angiogenesis D.Inhibition of estrogen receptor signaling E.Reduction in microtubule dynamics

A.Antibody-dependent cytotoxicity Trastuzumab is a recombinant human monoclonal antibody to ERBB2 used to treat women with metastatic breast cancer that over-expresses the erbB2 oncogene. It acts by binding to the extracellular domain of the ERBB2 receptor on cancer cells, preventing receptor stimulation and inhibiting cell growth. Additionally, the binding of the antibody facilitates cell-mediated cytotoxicity.

A 50-year-old woman presents to her physician because she has not been feeling well for the past 2 to 3 weeks. She has generalized malaise and a nagging cough that occasionally produces blood-tinged sputum. Today, for the first time, she noticed dark-colored urine.Physical examination reveals an ill-appearing middle-aged woman with a blood pressure of 180/110 mm Hg. Diminished air entry in the lungs bilaterally and an ulcerated lesion of the mucosa of the right naris are noted. The patient has no history of asthma or allergies. Urinalysis is grossly positive for blood, and a serum chemistry panel reveals a creatinine level of 1.7 mg/dL. What additional finding would confirm the most likely diagnosis? A.Antineutrophil cytoplasmic antibodies (ANCAs) in the serum B.Eosinophilia on WBC differential C.Hepatitis B surface antigen in the serum D.IgA deposition in glomerular mesangium E.Linear IgG deposition in the kidney

A.Antineutrophil cytoplasmic antibodies (ANCAs) in the serum This patient's constellation of signs and symptoms (malaise, hemoptysis, mucosal ulcers in the naris, and increased creatinine) is most consistent with granulomatosis with polyangiitis, formerly called Wegener granulomatosis. Typically, patients with this condition present with focal necrotizing vasculitis, necrotizing granulomas of the upper and/or lower airways, and necrotizing glomerulonephritis. In most cases, positive titers for antineutrophil cytoplasmic antibodies with a cytosolic staining pattern (c-ANCA) are also present. The disease is caused by systemic granulomatous inflammation, particularly of small vessels, such as those present in the kidneys and lungs. If not treated with immunomodulating drugs, focal glomerulonephritis can progress to a crescentic form, with ensuing renal failure.Eosinophilia is associated with eosinophilic granulomatosis with polyangiitis, previously known as Churg-Strauss syndrome or allergic granulomatous and angiitis. Patients with this condition often have asthma and/or allergies.

A 78-year-old man with a history of poorly controlled hypertension comes to the emergency department because of unilateral weakness and slurred speech. A CT scan reveals ischemia within the left temporal lobe. Histological examination 1 to 2 weeks after ischemic cell injury is likely to show activation of which of the following cell types? A.Astrocytes B.Ependymal cells C.Fibroblasts D.Microglia E.Oligodendrocytes F.Schwann cells

A.Astrocytes After a trauma or ischemia, cell death and inflammation is followed by cell proliferation and tissue remodeling. Gemistocytic astrocytes accumulate glial fibrils and form a glial scar. Unlike fibrotic scars seen in other tissue types of the body, glial scars do not contain collagen and are formed from astrocytes.

A 66-year-old man comes to his general practitioner for an annual check-up examination. He has no health concerns; his past history is significant for 45-pack-years of smoking. EKG reveals no significant findings. However, on physical examination, a pulsating midline abdominal mass is seen and a consultation with a vascular surgeon is scheduled. Which of the following is the etiology of this patient's presentation? A.Atherosclerotic plaque B.Calcification C.Cystic medial degeneration D.Fatty streaks E.Hypertension F.Intimal tear

A.Atherosclerotic plaque This man has a pulsating midline abdominal mass, a finding characteristic of an abdominal aortic aneurysm (AAA). It may or may not be associated with a midline abdominal bruit. Although the patient had no medical concerns prior to his visit, his 45-pack-year history of smoking constitutes a risk factor for abdominal aortic aneurysm. Other risk factors include build-up of atherosclerotic plaque or atherosclerosis, older age, male sex, and family history. Although abdominal aortic aneurysms may initially be monitored and treated with pharmacologic intervention, surgery may be necessary in the event the aneurysm becomes too large.

A 78-year-old man comes to the doctor after a 3-day history of crampy left lower quadrant abdominal pain accompanied by diarrhea and slight fever. He reports that there was a small amount of blood in his stool earlier in the day, but denies any long-term bleeding. His BMI is 31. He has chronic constipation and takes a daily stool softener. Family history is negative for colon cancer. On examination, he has a temperature of 38.2°C (100.7°F) and nonradiating tenderness in the left lower quadrant. Laboratory studies reveal a leukocyte count of 14,000/mm3, a hematocrit level of 35%, and platelet count of 250,000/mm3. Contrast-enhanced CT of the abdomen shows a thickened, enhancing colonic wall and pericolic fat stranding. Histopathology of the site would most likely show which of the following? A.Atrophied muscularis propria B.Dilated submucosal varices C.Intramural inflammation and crypt abscesses D.Invaginated mesenteric tissue E.Lymphoid tissue in mucosa

A.Atrophied muscularis propria Diverticula occur when pressure causes herniation of the colonic mucosa and submucosa through attenuated muscularis propria, creating a false diverticulum. Diverticulitis is an inflammation of colonic diverticula, due to retention of material within a diverticulum and fecalith formation.

A 40-year-old man presents to the physician with trouble breathing. He reports that recently he has developed bruising after minor injuries and that he has been feeling weak more often than usual. The patient has a long history of medically managed Crohn disease. Further questions reveal that the patient had an acute episode of gout 3 months ago and was prescribed a medicine to prevent future attacks. Physical examination is notable for multiple ecchymoses. Respiratory rate is 26/min. Laboratory tests show: Hemoglobin: 5.5 g/dLHematocrit: 13%WBC count: 2200/mm³Platelet count: 39,000/mm³ Adverse interactions of which two drugs most likely caused this patient's condition? A.Azathioprine and allopurinol B.Azathioprine and colchicine C.Infliximab and colchicine D.Prednisone and allopurinol E.Prednisone and colchicine

A.Azathioprine and allopurinol When taken concomitantly, allopurinol (and febuxostat) interferes with the metabolism of azathioprine's toxic metabolite, 6-mercaptopurine, which can lead to fatal hematologic disorders, such as aplastic anemia. (The metabolite of azathioprine, 6-mercaptopurine, is normally inactivated by xanthine oxidase. Allopurinol and febuxostat are xanthine-oxidase inhibitors).

A middle-aged man who presents with new-onset difficulty speaking and a tremor. The labs show modest elevations in ALT, AST, and alkaline phosphatase. Physical exam findings reveal irregular brown discoloration in the cornea, and he has difficulty pronouncing words with a tremor at rest in both upper extremities. Diagnosis? Which of the following structures is most likely affected in this patient? A.Basal ganglia B.Cerebellum C.Frontal cortex D.Occipital lobe E.Parietal cortex F.Pons

A.Basal ganglia Wilson disease, also called hepatolenticular degeneration, is caused by an autosomal recessive mutation in the hepatocyte copper-transporting ATPase (ATP7B gene; chromosome 13) gene. The ATP7B gene prevents excretion of copper from the body. In Wilson disease, there is a decrease in copper incorporation into apoceruloplasmin, and a decrease of copper excretion into the bile, causing a decrease in serum ceruloplasmin. The Parkinsonian-like movement abnormalities seen in Wilson disease are a direct result of copper accumulation in the basal ganglia.

A 73-year-old man is admitted to the hospital because of cellulitis of the left lower extremity and is subsequently treated with clindamycin. Although the cellulitis resolves, he has diffuse lower abdominal cramping and watery, nonbloody diarrhea 5 days later. On examination, his temperature is 38°C (100.5°F); laboratory studies show mild leukocytosis. A CT scan of the abdomen shows thickening of the wall in the transverse colon. Sigmoidoscopy discloses raised white-yellow exudative plaques adherent to the colonic mucosa. Which of the following best describes the mechanism of action of the first-line antibiotic most appropriate to treat this patient's condition? A.Binding D-ala D-ala portion of the cell wall B.Binding the 30S subunit, inhibiting protein synthesis C.Blocking transpeptidase cross-linking D.Forming toxic free radical metabolites that damage bacterial DNA E.Inhibiting RNA polymerase

A.Binding D-ala D-ala portion of the cell wall The broad-spectrum antibiotic treatment shifts the balance of normal intestinal flora and allows C. difficile to grow in abundance. Subsequent treatment of pseudomembranous colitis includes immediate discontinuation of the original antibiotic. The new first-line therapy for treating severe C. difficile is oral vancomycin, whereas metronidazole was the former first-line therapy. This change in the first-line therapy is the result of several clinical trials demonstrating increased cure rates and decreased rates of recurrence following vancomycin treatment compared with metronidazole treatment. Oral vancomycin can be used for 10 to 14 days to treat a C. difficile infection. Vancomycin inhibits cell wall peptidoglycan formation by binding D-ala D-ala. Fidaxomicin is recommended as first-line therapy for mild or moderate cases of C. difficile infection but would not be appropriate in this patient's case.

A 6-year-old boy is brought to his pediatrician's office with a fever and pain in the right ear. His mother states that he has been tugging at his right ear for the past 3 days and that she recorded a temperature of 102.2°F (39°C) yesterday. His appetite has been normal, and he has been tracking in the 75th percentile for height, weight, and head circumference. His medical history is significant for recurrent sinus infections over the past year. His mother's pregnancy was uncomplicated, and she obtained appropriate prenatal care. Physical examination reveals an erythematous bulging tympanic membrane in the right ear. His vital signs are: temperature, 100.4°F (38°C); blood pressure, 98/70 mm Hg; pulse, 75/min; respiratory rate, 18/min; and oxygen saturation, 97% on room air. Heart sounds are not appreciated at pulmonic, tricuspid, and mitral positions; but a regular rate and rhythm are auscultated at the aortic position. Lungs are clear to auscultation bilaterally. This patient has an increased risk for development of which of the following complications? A.Bronchiectasis B.Emphysema C.Failure to thrive D.Heart failure E.Mitral regurgitation F.Pancreatic insufficiency

A.Bronchiectasis Recurrent sinusitis caused by a failure in bacteria and particle clearance and bronchiectasis caused by a nonfunctional mucociliary elevator are both features of Kartagener syndrome.

Steroid hormones are unique in that they enter the cell and act directly on DNA to effect gene expression level changes, rather than acting only through intermediary signaling proteins. Which of the following steps immediately precedes steroid hormone-receptor complex binding to DNA? A.Conformational change of the hormone-receptor complex B.Hormone binding to a DNA enhancer element C.Hormone binding to a hormone-specific globulin D.Hormone binding to a membrane receptor E.Hormone binding to its intracellular receptor

A.Conformational change of the hormone-receptor complex The steroid hormone circulates in the plasma bound to a hormone-specific binding globulin. At the target organ, it crosses the cell membrane due to its lipophilic properties and binds to an intracellular receptor either in the cytoplasm or within the nucleus. The hormone-receptor complex then undergoes a conformational change, which reveals the receptor's DNA-binding domain; without this step, it is unable to carry out its action and bind to DNA. Once the binding domain is revealed, the hormone-receptor complex binds the DNA enhancer element and causes changes to gene expression at the transcriptional level.

A 50-year-old man comes to his primary care physician because of chronic blistering, itchy skin over his forearms. Laboratory records show that the patient has a longstanding history of mild elevation of transaminases in blood. The patient reports that his urine is "tea-colored." On examination, there small tense vesicles, bullaeand erosions over the dorsal aspect of both forearms and hands. In which cellular compartment is the defective enzyme most likely located? A.Cytoplasm B.Mitochondria and cytoplasm C.Mitochondria only D.Nucleus E.Rough endoplasmic reticulum

A.Cytoplasm This 50-year-old man with chronic skin blistering, tea-colored urine, and elevated transaminases most likely has porphyria cutanea tarda (PCT), which is caused by inherited or acquired defects in the uroporphyrinogen decarboxylase enzyme. Different steps of the heme synthesis pathway occur in the mitochondria or the cytoplasm. Mitochondria are involved in the first and final three steps of the pathway. The defective enzyme in PCT, uroporphyrinogen decarboxylase, is present in the cytoplasm.

A 42-year-old man comes to the emergency department because of severe lower back pain. The pain started abruptly as he was lifting a piano while helping his daughter move into a new apartment. He describes the pain as sharp and shooting and says it radiates down his right leg. The patient has a history of hypertension and type 2 diabetes mellitus. On examination, there are no obvious signs of trauma. The pain is exacerbated while slowly lifting the leg while the leg is straight at the knee and the patient is supine. The patient describes the pain as excruciating at an angle of 45 degrees. After he is given medication for his pain, the patient undergoes CT of his lumbar region, which reveals a herniated disc at the L5-S1 level. Which is the most likely finding on neurologic examination? A.Diminished ankle-jerk reflex B.Diminished knee-jerk reflex C.Diminished sensation to the medial aspect of foot D.Weakness to dorsiflexion of foot E.Weakness to knee extension

A.Diminished ankle-jerk reflex

A 32-year-old primigravid woman presents at 32 weeks' gestation because of sudden vaginal bleeding and painful abdominal cramps. She denies trauma. She has a 10-pack-year smoking history and admits to cocaine use during her pregnancy. Her blood pressure is 95/60 mm Hg, pulse is 112/min, and blood oxygen saturation is 97% on room air (normal: 95%-99%). Physical examination reveals a tense abdomen with a firm, tender uterus. Urinalysis shows no protein, leukocytes, or bacteria, with few RBCs. Pelvic examination reveals dark-red blood in the vaginal vault and a hypertonic uterus. Pelvic ultrasound shows a high posterior placenta with no abnormalities of placentation. Fetal heart tones indicate fetal distress. This patient is at increased risk for which of the following complications? A.Disseminated intravascular coagulation B.Infertility C.Postpartum bleeding D.Preeclampsia E.Sheehan syndrome

A.Disseminated intravascular coagulation Placental abruption is the partial or complete premature separation of placenta from the uterine wall. Risk factors include smoking and cocaine use as seen in this patient, as well as trauma, hypertension, and preeclampsia. Patients present with sudden painful bleeding in the third trimester, as seen in the vignette. Complications are life threatening and include DIC from tissue factor entering maternal circulation, maternal shock, and fetal distress or death.

Which of the following inhaled anesthetics was most likely caused seizures without fever? A.Enflurane B.Halothane C.Isoflurane D.Nitrous oxide E.Thiopental

A.Enflurane Enflurane is more likely to produce central nervous system excitation and seizure activity compared with other inhaled anesthetic agents. Enflurane causes seizures by destabilizing the cortex and delaying inhibitory circuits. Seizures induced by enflurane are usually self-limited and are not believed to have any permanent neurologic effects. It is important to note that this patient does not have a fever. For these potential complictions, enflurane is not commonly used however its side effect profile is a testable subject. Halothane, another inhaled anesthetic, can cause life-threatening malignant hyperthermia, which causes muscle rigidity, fever, and severe muscle contractions. Although enflurane is a less potent anesthetic than halothane, it produces a more rapid induction and a quicker recovery than halothane. This is because enflurane has a lower blood solubility than halothane.

A 40-year-old man is admitted to the neurology service for evaluation of persistent numbness over his left lower jaw and lower face. On examination, the patient's jaw deviates to the left when he attempts to open his mouth. MRI reveals a Schwannoma compressing a cranial nerveas it exits the skull. The cranial nerve involved in this case exits the skull through which of the following foramina? A.Foramen ovale B.Foramen rotundum C.Foramen spinosum D.Jugular foramen E.Stylomastoid foramen F.Superior orbital fissure

A.Foramen ovale CN V3 provides sensory innervation to the lower face and motor innervation to the masseter, medial pterygoid, temporalis, and lateral pterygoid. Compression of CN V3 causes numbness over the ipsilateral jaw and lower face and deviation of the mandible toward the side of the lesion as a result of unopposed action of the contralateral pterygoid muscles. It is likely this patient has a schwannoma of the mandibular division of the trigeminal nerve (CN V3) as the nerve exits the skull through the foramen ovale.

A nineteen-year-old woman comes to the clinic because she has never had a menstrual period. On physical examination, the patient has normally developed breasts, lacks hair in the groin or axillary areas, and has small hypoplastic labia; uterus is impalpable. Which of the following molecular mechanisms is likely impaired in this patient? A.Gene transcription B.Protein synthesis C.Protein translation D.mRNA degradation E.mRNA localization F.mRNA processing

A.Gene transcription This patient presents with androgen insensitivity syndrome (also known as testicular feminization), which is an X-linked recessive condition that leads to a failure of development of normal external genitalia in chromosomally male individuals due to mutations in genes encoding the androgen receptor. Androgen insensitivity syndrome presents with elevated levels of testosterone, estrogen, and LH. The testes develop normally in these patients, so the secretion of Mullerian-inhibitory factor (MIF) remains intact. MIF prevents the development of female internal structures including the fallopian tubes and uterus, as seen in this patient. There is a spectrum of androgen insensitivity syndrome.

Laboratory testing in a 43-year-old woman reveals an elevated serum insulin growth factor 1 (IGF-1) level. Through which of the following veins does IGF-1 first enter systemic circulation? A.Hepatic veins B.Hypophyseal portal vein C.Inferior vena cava D.Portal vein E.Splenic vein

A.Hepatic veins Growth hormone, which is secreted by the anterior pituitary gland, stimulates production of IGFs/somatomedins in the liver. These proteins are then secreted into systemic circulation through the hepatic vein and then into the inferior vena cava. Thehypophyseal portal veinsupplies blood from the hypothalamus to the anterior pituitary, not to systemic circulation. IGFs/somatomedins eventually reach theinferior vena cava,but these proteins are first secreted into systemic circulation via the hepatic vein. Theportal vein,and thesplenic veinthat leads into it, deliver blood from the spleen, pancreas, and gastrointestinal tract to the liver and do not carry IGFs/somatomedins into systemic circulation.

A 72-year-old man is brought to the emergency department because of fatigue, confusion, and anorexia. He says that he feels "funny" and that he missed his routine hemodialysis appointment 2 days ago. His wife reports that he has gotten progressively worse during the past 2 days. His past medical history is significant for type 2 diabetes mellitus, diabetic retinopathy, and chronic kidney disease. His medications include lisinopril and insulin. Temperature is 37.7° C (100° F), blood pressure is 121/78 mm Hg, pulse is 72/min, and respiratory rate is 24/min. Laboratory values show BUN of 80 mg/dL and creatinine of 2.2 mg/dL. Which of the following is the most likely additional laboratory finding? A.High anion gap metabolic acidosis B.Metabolic alkalosis C.Normal anion gap metabolic acidosis D.Respiratory acidosis E.Respiratory alkalosis

A.High anion gap metabolic acidosis uremia due to his missed hemodialysis appointment. Uremia is defined as elevated levels of urea in the blood, typically measured with blood urea nitrogen (BUN). Uremia is associated with high anion gap metabolic acidosis. In the setting of chronic kidney disease, the ability of the kidneys to excrete acid is impaired, leading to retention of hydrogen ions. Bicarbonate (HCO3-) is used to buffer this excess acid, depleting bicarbonate stores and leading to metabolic acidosis and uremia.

34-year-old black man is brought to the emergency department because of recurrent vomiting and epigastric pain. Laboratory studies show elevated serum lipase and amylase concentrations. He is admitted to the intensive care unit for conservative management. The Acute Physiology and Chronic Health Examination (APACHE) II score is then calculated by the physician for this patient.. Which of the following best describes the APACHE II scoring system that is used for this patient? A.It does not differentiate between sterile and infected necrosis. B.It does not predict the in-hospital mortality of the patient. C.It helps to differentiate the cause of pancreatitis. D.It is verified only for patients with acute pancreatitis in the intensive care unit. E.It should not be used in this scenario.

A.It does not differentiate between sterile and infected necrosis. The Acute Physiology and Chronic Health Examination (APACHE) II score was originally developed for critically ill patients in ICUs. The APACHE II scoring system is calculated when any patient is admitted to the intensive care unit (ICU), no matter the cause. Acute pancreatitis is one of many reasons why a patient may be admitted to the ICU. APACHE II has 12 physiologic measures, including age, temperature, and pH (Students do not have to memorize these variables for the USMLE.). In summary, the APACHE II score is calculated when the patient is admitted to the ICU to help determine the patient's mortality risk for hospital admission. Although it is not mandatory, the APACHE score is a useful tool to compare the care received by patients with similar scores in different hospitals. It is not calculated after the initial assessment, because it is not used to direct medical management. A higher APACHE II score is associated with a higher mortality.

A 16-year-old presents to her family physician because she believes she is pregnant. An office pregnancy test confirms her suspicion. The biological marker detected by the blood test to assess pregnancy is most structurally similar to which of the following? A.The hormone that promotes rupture of the ovarian follicle during ovulation B.The hormone that promotes the release of cortisol from the adrenal cortex C.The hormone that promotes the release of thyroid-stimulating hormone from the anterior pituitary D.The hormone that stimulates milk production from mammary glands E.The hormone that stimulates uterine contractions during labor

A.The hormone that promotes rupture of the ovarian follicle during ovulation The physician has likely ordering a pregnancy test that measures the amount of β human chorionic gonadotropin (β-hCG). Structurally, β-hCG shares the identical α subunit as thyroid-stimulating hormone (TSH), luteinizing hormone (LH), and follicle-stimulating hormone (FSH). LH promotes rupture of the ovarian follicle during ovulation. Given the identical α subunit, the β subunit confers specificity to LH, FSH, TSH, and β-hCG. These hormones (except β-hCG) are secreted from the anterior pituitary gland. β-hCG is secreted by the syncytiotrophoblasts after fertilization.

what do ACE inhibitors do to the efferent glomerular arteriole?

ACE normally constricts efferrent with Gq so ACE dilate efferent by inhibiting Gq ACE Constricts Efferent

ARDS: [incr/decr] PaO2/FiO2, DLCO, Lung compliance, peak pulmonary pressures on ventilator

ARDS is associated with a severely decreased PaO2/FiO2 caused by impaired gas exchange in the injured lung. Diffusing capacity of the lungs for carbon dioxide (DLCO) would likely be reduced in ARDS secondary to thickened alveolar membranes (fluid, hyaline membrane lining). Lung compliance is usually reduced in patients with ARDS secondary to interstitial edema, hyaline membrane formation, collapse of fluid-filled alveoli, and atelectasis. Because lung compliance is usually reduced in patients with ARDS, peak pulmonary pressures are increased on the ventilator. Peak pulmonary pressures correlate inversely with lung compliance and correlate directly with airway resistance.

What is Metachromatic leukodystrophy caused by? what accumulates? what pigment effects does this have? which two lysosomal diseases have foam cells? two causing cherry red spot on macula? how to distinguish them?

Arylsulfatase A deficiency; sulfatides; can change the color of toluidine blue dye to pink; Gaucher and Nieman pick; Tay Sachs and Neiman pick; [tay sachs does not have hepatosplenomegaly, while neiman pick does] Marty p picks nose and looks like he has hepatosplenomegaly

A 29-year-old man is brought to the emergency department after being found unconscious in the garage by his wife. She explains that he was working on restoring a classic car that evening and she became concerned when he did not come to dinner. When she found him, she noticed the car was running. EMS arrive to asses his condition. The patient does not have a history of seizures and does not take any medication. On the way to the hospital, he regains consciousness and is able to speak without slurring his words; he does not recall ingesting any substances. However, he is confused and states he has a headache. Vital signs include temperature 37° C (98.6° F), blood pressure 125/84 mmHg, pulse is 94/min, respirations are 24/min, and oxygen saturation is 99% on room air. The patient's skin is covered in large red patches that do not itch. His lungs are clear to auscultation. Cranial nerves II-XII are grossly intact. No lesions or injuries are observed on the patient's scalp and his head is non-tender. Which of the following treatments is the best choice for this patient? A. Hydroxocobalamin B. Hyperbaric oxygen C. IV sodium thiosulfate D. Intravenous thrombolytics E. Intubation F. Methylene blue

B. Hyperbaric oxygen Carbon monoxide binds to Hb (hemoglobin) with 200 times greater affinity than O2 (oxygen). In carboxyhemoglobin formation, the patient has less available Hb to bind oxygen, making the patient hypoxic. There is a left-shift of the oxygen dissociation curve, which decreases oxygen unloading at the tissue as well as a decrease in maximum total oxygen carrying capacity (O2/mL blood). Treatment of carbon monoxide poisoning is with hyperbaric oxygen. The process involves exposing the patient to highly concentrated oxygen levels in an attempt to displace CO bound to Hb. High concentrations of oxygen are required as CO is bound with increased affinity.

The function of which of the following muscles is most likely affected by ulnar n. injuries? A.Abductor pollicis brevis B.Adductor pollicis C.Extensor pollicis brevis D.Lumbricals (1 and 2) E.Opponens pollicis

B.Adductor pollicis The adductor pollicis muscle has an oblique and transverse head, both of which are innervated by the deep branch of the ulnar nerve, allowing the muscle to participate in thumb adduction and thumb MCP flexion.

A 72-year-old man comes to the emergency department after noticing a lot of blood in his urine on awakening that morning. He also has pain in his left side. The patient states that he has never had anything like this before. He does not smoke and only drinks socially during the holidays. His medical history includes hypertension and chronic low back pain that has been getting worse. He has been taking ibuprofen on an ongoing basis for the back pain. The only other medication he takes is lisinopril. His temperature is 36.4°C (97.5°F), pulse is 103/min, respirations are 16/min, and blood pressure is 148/92 mm Hg. Physical examination of his abdomen reveals diffuse tenderness to palpation with no rebound, guarding, or rigidity. Bowel sounds are normoactive in all four quadrants, and no hepatosplenomegaly is detected. Urinalysis shows 3+ blood, proteinuria, and no casts or glucose. Serum creatinine concentration is 1.4 mg/dL. Which of the following is the most likely cause of this patient's symptoms? A.Acute glomerulonephritis B.Analgesic use C.IgA nephropathy (Berger disease) D.Nephrolithiasis E.Renal cell carcinoma F.Transitional cell carcinoma

B.Analgesic use This man with gross hematuria and flank pain has no genitourinary history. Although his symptoms initially suggest nephrolithiasis, his description of the pain is not consistent with renal colic. His recent increase use of ibuprofen, a nonsteroidal anti-inflammatory drug (NSAID), has likely caused acute tubular necrosis leading to renal papillary necrosis (RPN). RPN is ischemic and suppurative necrosis of the tips of the renal pyramids and/or renal papillae. Clinical presentation includes gross hematuria, proteinuria, and flank pain. Risk factors for RPN can be remembered by using the mnemonic SAAD: Sickle cell disease/trait, Acute pyelonephritis, Analgesics (NSAIDS +/- acetaminophen), and Diabetes mellitus. The differential diagnosis of gross hematuria is extensive and includes urinary calculus, genitourinary malignancy, RPN, genitourinary trauma, and genitourinary infection or hemorrhagic cystitis. Nephrolithiasis typically presents with colicky pain and urinary symptoms. Renal and transitional cell carcinomas are painless, and this patient is at low risk because he does not smoke. Because this patient has no history of autoimmune disease or recent illness, acute glomerulonephritis is unlikely. IgA nephropathy may present with isolated hematuria in the absence of proteinuria, which is not consistent with the urinalysis findings for this patient.

Which of the following is the most likely underlying cause of orthostatic hypotension? A.Activation of α1-receptors, leading to increased phospholipase C and calcium release B.Blockade of α1-receptors, leading to decreased phospholipase C and calcium release C.Blockade of α1-receptors, leading to increased generation of cAMP and protein kinase A D.Blockade of β2-receptors, leading to decreased generation of cAMP and protein kinase A E.Blockade of histamine (H1)-receptors, leading to decreased phospholipase C and calcium release

B.Blockade of α1-receptors, leading to decreased phospholipase C and calcium release Blockage of α1-receptors can lead to reduced arteriolar resistance and postural hypotension. α1-Receptors are of the Gq class, and blockage of α1-receptors will lead to a decrease in phospholipase C and calcium release.

A 42-year-old African American woman from Connecticut presents to her physician with pain that she has had for several years. She states she always worries about her pain and feels as though "something is seriously wrong." She describes the pain as constant, located in her neck, lower back, and hips. Over the years she has had extensive work-ups, including laboratory testing and multiple modalities of imaging, none of which has led to an explanation for her symptoms. She exhibits persistent anxiety about her symptoms and feels as though they interfere with her day-to-day life. She has had no history of trauma, but does report feeling "down" occasionally. When asked about her sexual history, she states that she has had one sexual partner over the last 5 years. Which treatment would be most appropriate for her at this time? A.A 14-day course of oral doxycycline B.Cognitive behavioral therapy C.Exercise D.Nonsteroidal anti-inflammatory drugs (NSAIDs) E.Reassurance; her symptoms will resolve spontaneously with time

B.Cognitive behavioral therapy This patient has unspecified pain in multiple areas with no confirmative diagnosis and is anxious and perhaps depressed about her symptoms. These are findings associated with somatic symptom disorder. This diagnosis comes under the new Diagnostic and Statistical Manual of Mental Disorders, fifth edition (DSM V) categorization (see chart). Somatic symptom disorder is characterized by somatic symptoms (eg, pain) that cause psychosocial distress or impairment. There is typically a history of anxiety or depression in these patients, and such disorders are seen more commonly in women than men. Treatment typically includes rehabilitation, such as cognitive behavioral therapy, physical therapy, and psychotherapy. Analgesia is usually not helpful. Selective serotonin reuptake inhibitors (SSRIs) are first-line pharmacotherapy. Tricyclic antidepressants (TCAs) and venlafaxine may also be therapeutic.

An 11-year-old boy is brought to the emergency department by his mother because of seizure, excessive sweating, dizziness, and nausea. These symptoms occurred following a 32-hour trek without any food, as the boy was stuck on a mountain because of bad weather. His mother mentions that the patient has had similar complaints in the past with fasting. He takes no medications. Temperature is 37.2° C (99° F), pulse is 120/min, and respiratory rate is 22/min. On physical examination, there is generalized decreased muscle tone, and the liver and spleen are palpated a few centimeters below the left costal margin. Laboratory studies show positive urinary ketones, increased creatinine kinase and decreased glucose levels; lactate levels are within reference ranges. Which of the following enzymes is most likely deficient in this patient? A.Branching enzyme deficiencies B.Debranching enzyme C.Glucose-6-phosphatase deficiency D.Lysosomal acid maltase deficiency E.Myophosphorylase deficiency

B.Debranching enzyme In Cori disease, there is a deficiency of the debranching enzyme which leads to accumulation of limit-dextrin-like structures in the cytosol of hepatocytes, which leads to hepatomegaly and can be demonstrated on histology (see illustration). Since glycogen cannot be broken down to release glucose, prolonged fasting leads to hypoglycemia and fatigue.The other options are incorrect for the following reasons (also, see table): Glucose-6-phosphatase deficiency leads to Von Gierke disease which disease presents as severe fasting hypoglycemia, increased blood lactate, triglycerides, and uric acid levels. Since blood lactate levels are within reference ranges in this patient, this option can be ruled out. Lysosomal acid maltase deficiency can present as hypotonia but most children die by the age of this patient. Lysosomal acid maltase deficiency leads to Pompe disease which leads to hypertrophic cardiomyopathy, hypotonia, exercise intolerance, and early death. Myophosphorylase deficiency leads to McArdle disease which presents with myoglobinuria with strenuous exercise and arrhythmias from electrolytes abnormalities. Branching enzyme deficiency will not lead to symptoms of hypoglycemia. Branching enzyme deficiency leads to Anderson disease which presents as hepatosplenomegaly.

Which of the following is a common physiologic effect of norepinephrine? A.Decrease in diastolic filling pressure of the heart B.Decrease in heart rate C.Decrease in platelet aggregation D.Decrease in pupillary dilator tone E.Decrease in systemic vascular resistance

B.Decrease in heart rate Norepinephrine administration will result in increased venous return to the heart through vasoconstriction in conjunction with increased heart contractility, producing an increase in stroke volume (SV). Because the heart no longer needs to beat as quickly to maintain sufficient cardiac output (CO) (as CO = SV × Heart rate), there is a reflex decrease in heart rate. The effects of norepinephrine at the α1-adrenergic receptor lead to peripheral vasoconstriction. This increases venous return and diastolic filling pressure of the heart.

A primigravid woman visits the emergency department at 30 weeks' gestation because of severe abdominal cramping, headaches, and photophobia. During a mental status examination, the patient appears confused and suddenly has a generalized tonic-clonic seizure. She is rushed to surgery for an emergency cesarean section.Shortly after birth, the neonate begins showing signs of dyspnea and appears cyanotic. The neonate's respiratory rate is 70, and oxygen saturation is 90% on room air. Nasal flaring, expiratory grunting, and intercostal retractions are observed on physical examination. No murmurs are auscultated over the heart. The neonate's chest x-ray is shown. This infant′s condition is primarily caused by which of the following? A.Delayed clearance of fluid B.Diffuse atelectasis C.Impaired chest wall movement D.Impaired diffusion E.Right-to-left cardiac shunt

B.Diffuse atelectasis Decreased surfactant leads to alveolar collapse from increased surface tension, causing diffuse atelectasis.

A mother brings her 14-year-old son to a pediatrician and notes that her son's eyes are "kind of yellow." The son denies any symptoms, changes, or new medications and says, "Everything is fine. She's overreacting as usual." Vital signs are: T 36.7°C (98.1°F), BP 120/90 mm Hg, HR 85, and RR 15. Laboratory tests reveal:Total bilirubin: 4.5Direct bilirubin: 4.0The boy's mother recalls that her sister's son also appeared very yellow for some time after his birth. Her nephew ultimately underwent a biopsy, which demonstrated abnormallypigmented hepatocytes. Which of the following is the most likely diagnosis? A.Crigler-Najjar syndrome B.Dubin-Johnson syndrome C.Gilbert syndrome D.Rotor syndrome E.Wilson disease

B.Dubin-Johnson syndrome This autosomal recessive inherited form of conjugated (direct) hyperbilirubinemia presents with no other significant clinical symptoms. The disorder is caused by an impairment of the hepatocyte transport system for removal of conjugated anions (eg, bilirubin diglucuronide) into bile. The genetic defect that causes Dubin-Johnson syndrome is a mutation in the multidrug resistance protein 2 (ABCC2) gene.

Urinalysis shows positive leukocyte esterase, negative nitrite, and negative hematuria with a pending bacterial culture. Blood cultures are drawn and the patient is started on empiric antibiotics to treat the presumed infection. Which of the following pathogens is most likely responsible for this patient's symptoms? A.Enterobacter cloacae B.Enterococcus faecalis C.Escherichia coli D.Klebsiella pneumoniae E.Proteus mirabilis F.Staphylococcus saprophyticus

B.Enterococcus faecalis Why incorrect answers are wrong: Gram-negative rods, particularly coliforms like E. coli, are very commonly associated with UTI. However, E .coli is a gram-negative organism and is typically nitrite positive on urinalysis. Staphylococcus saprophyticus is a gram-positive, coagulase negative coccus most often associated with UTI in sexually active females. This organism is part of the normal skin flora that grow in moist areas like the folds in the groin and armpit areas. S. saprophyticusis typically not associated with urosepsis and endocarditis. Proteus mirabilis is a gram-negative rod characterized by urease production and associated with struvite stones. It is highly motile, giving rise to the "swarming" appearance on agar medium. It would produce a nitrite positive urinalysis and is not typically associated with endocarditis. Enterobacter cloacae is a gram-negative, lactose-fermenting rod characterized by extensive antibiotic resistance. Enterobacter is known to cause many hospital-acquired infections, including UTI, pneumonia, and surgical site infections. Urinalysis is typically nitrite positive. Klebsiella is a gram-negative, lactose fermenter that can cause hospital- and community-acquired UTIs, pneumonia and spontaneous bacterial peritonitis. It produces a mucoid capsule that gives rise to a mucoid colony on agar. Urinalysis of a UTI caused by this organism is typically nitrite positive.

A 34-year-old woman presents to the emergency department with acute renal failure. She describes progressive numbness in her distal extremities bilaterally. The patient reports that she was recently diagnosed with asthma and denies having a history of the condition as a child. Family history is negative for asthma, and she has never smoked cigarettes. Her temperature is 36.7°C (98°F), pulse is 78, and blood pressure is 115/68 mm Hg.Laboratory test results include:Eosinophils: 5000/μLBUN: 26 mg/dL,Creatinine: 2.4 mg/dL.CBC and renal panel are otherwise unremarkable.Renal biopsy with immunofluorescence reveals the absence of antibody deposition. The patient is positive for perinuclear antineutrophil cytoplasmic antibody (p-ANCA). Which of the following is the most likely diagnosis? A.Acute interstitial nephritis B.Eosinophilic granulomatosis with polyangiitis (EGPA) C.Goodpasture syndrome D.Granulomatosis with polyangiitis E.Poststreptococcal glomerulonephritis

B.Eosinophilic granulomatosis with polyangiitis (EGPA) eosinophilic granulomatosis with polyangiitis (EGPA), formerly known as Churg-Strauss syndrome. This granulomatous small- to medium-sized vessel vasculitis is characterized by necrotizing nephritis (without immune complex or antibody deposition), asthma, chronic rhinosinusitis, and elevated circulating levels of perinuclear antineutrophil cytoplasmic antibody (p-ANCA). This patient's progressive numbness in the distal extremities is also a symptom of this condition. Granulomatosis with polyangiitis, formerly called Wegener granulomatosis, is a systemic vasculitis affecting small- to medium-sized vessels. It typically involves the upper and lower respiratory tracts and the kidneys. The glomerulonephritis associated with this condition is characterized by crescent formation secondary to necrosis and the absence of antibodies and complement, which are not consistent with the immunofluorescence findings for this patient. Granulomatosis with polyangiitis is associated with and positive for PR3-ANCA and c-ANCA (anti-proteinase 3), not p-ANCA (anti-myeloperoxidase).

A 13-year-old boy who was recently adopted from another country is brought into the clinic by his parents for his first medical visit. According to the parents, the patient has had multiple skin infections in the past, but they were not explained during the adoption. The patient's pulse is 70/min, respirations are 16/min, and blood pressure is 120/80 mm Hg. Physical examination reveals two rows of upper and lower teeth, facial features that are "leathery" in appearance, and several small abscesses on his upper and lower extremities without surrounding erythema. Which of the following mechanisms is responsible for this patient's skin findings? A.Decreased B and T cells B.Failure to produce interleukin-17 C.Failure to synthesize major histocompatibility complex II antigens D.Inability of intercellular adhesion molecule 1 binding E.Microtubule dysfunction

B.Failure to produce interleukin-17 These are symptoms of hyper-IgE syndrome (Job syndrome). The gene mutation responsible for hyper-IgE syndrome is in STAT3. In this disease, TH17 cells fail to produce interleukin-17 (IL-17), leading to an inability of neutrophils to respond to chemotactic stimuli. Consequently, the inflammatory response is blunted in these patients, resulting in frequent staphylococcal abscesses that lack the expected warmth and erythema ("cold abscesses"). The presence and activation of immunoglobulin E (IgE) leads to eczema. A method to memorize the findings of Job syndrome is the FATED mnemonic: Facies that are coarse, Abscesses that are cold, retained primary Teeth, increased IgE, Dermatologic problems.

Which of the following features of this patient's history most increases her risk of breast cancer? A.Age of 42 years B.Family history of breast cancer C.Never having had a mammogram D.Nulliparity E.Occasional irregular periods

B.Family history of breast cancer Factors that increase breast cancer risk include, but are not limited to, advanced age, nulliparity, early onset of menarche, late menopause, genetic predisposition, and a family history of breast cancer at an early age (<40 years). Additional lifestyle factors such as excessive alcohol consumption, obesity, and insufficient exercise may also contribute to risk. In this patient, the strong family history of breast cancer at a young age is most pertinent. Having a first-degree relative diagnosed with breast cancer at <30 years of age triples the risk of breast cancer. This history should also raise red flags for a genetic susceptibility to breast cancer, such as a mutation in the BRCA1 or BRCA2 gene.

A 24-year-old man comes to the physician for a routine physical examination before beginning a new job. The physician notes a small, firm nodule on the patient's left testicle, and he is subsequently diagnosed with a testicular seminoma and referred to an oncologist. Soon after beginning high-dose chemotherapy, the patient returns to his primary care physician because of a 1-week history of dry cough and shortness of breath. His temperature is 98.7°F (37°C), and his pulse is 82/min. His respiratory rate is 17/min. A cardiac examination discloses no murmurs or gallops, and both lungs are clear to auscultation without wheezes; there is no dullness to percussion. Which of the following is the major mechanism of the chemotherapeutic agent that was most likely used to treat this patient's seminoma? A.Alkylation of DNA B.Formation of superoxide or hydroxide radicals C.Inhibition of microtubule polymerization D.Inhibition of topoisomerase II E.Stabilization of microtubules against depolymerization

B.Formation of superoxide or hydroxide radicals Bleomycin is a mixture of glycopeptides that have the unique feature of producing superoxide and hydroxyl radicals that attack DNA bonds and cause fragmentation. Bleomycin can also cause skin changes and alopecia.

A 45-year-old man comes to the clinic because of a 5-hour history of lethargy and a single episode of vomiting. He reports that he has not been eating well for the past 2 days because of a hectic work schedule. He takes no medication and has no significant medical history. His temperature is 36.7° C (98° F), pulse is 124/min, and respirations are 22/min. He is 177.8 cm (5 ft, 8 in) tall and weighs 77.1 kg (170 lb); his body mass index (BMI) is 24 kg/m2. Laboratory studies show that renal function is within the reference range, and glucose concentration is 65 mg/dL. Which of the following enzymes is currently more active in maintaining this patient's glucose level? A.Fructokinase B.Fructose-bisphosphatase 2 C.Phosphofructokinase-2 D.Phosphoglycerate kinase E.Pyruvate kinase

B.Fructose-bisphosphatase 2 This patient is a 45-year-old man with acute onset of lethargy and vomiting. He is stressed and has not been eating well. His BMI is 24 kg/m2. His glucose concentration is low. Based on these findings, the most likely diagnosis is mild hypoglycemia.In the fasting state, glucagon and epinephrine are stimulated via the β-receptors. β-Receptors stimulate the Gs pathway leading to increased cAMP, resulting in increased protein kinase A. Protein kinase A stimulates fructose-bisphosphatase 2 and inhibits phosphofructokinase-2, both of which lead to a decrease in fructose 2,6-bisphosphate, which is responsible for elevated expression of phosphofructokinase-1, the enzyme of glycolysis. Thus decreased glycolysis leads to decreased breakdown of glucose in a fasting state.The other options are incorrect for the following reasons: The phosphofructokinase-2 (PFK-2) level is increased in a fed state. During a fed state, insulin leads to a reduction in cAMP, which is responsible for increasing the PFK-2 level. Phosphoglycerate kinase is active in both fasting and fed states because it is a reversible enzyme. Activity of fructokinase is at a maximum after consumption of a meal high in fructose. This enzyme is not involved in glucose metabolism. Pyruvate kinase is the rate-limiting step of glycolysis and is most active in a fed state.

A four-year-old boy is brought to a pediatrician by his parents for a follow-up appointment after fracturing his arm after a fall from his bicycle. He has had two previous fractures. His parents are healthy, but his brother passed away last year after similar episodes of bone fractures. His temperature is 98.6° F (37° C), blood pressure is 130/78 mm Hg, pulse is 100/min, and respiratory rate is 20/min. Physical examination reveals deformity of the arm along with swelling and tenderness on palpation. Ocular examination reveals blue-gray tinted sclerae. Which of the following best describes the cause of this patient's presentation? A.Allelic Heterogeneity B.Germline mosaicism C.Heteroplasmy D.Incomplete dominance E.Somatic mosaicism F.Uniparental disomy

B.Germline mosaicism Osteogenesis imperfecta presents with blue-gray-tinged sclerae and the frequent fractures. Osteogenesis imperfecta is an autosomal dominant disorder that requires a mutated allele to be inherited from an affected parent in order to be expressed. There is a 50% chance of the child inheriting the disorder from an affected parent. However, in this case the mutated allele is not expressed in either of the patient's parents. Therefore, this patent most likely inherited the disorder due to germline mosaicism. Germline mosaicism occurs when more than one set of genetic information is found within the gamete cells. A pedigree is shown.

A 46-year-old man presents to the office complaining of muscle weakness for the past several months. He reports tiring easily after a workout and often experiences tingling sensations in his extremities. In addition, his frequent episodes of headaches and dizziness have affected his productivity at work. Review of systems is significant for increased frequency of urination. He has no significant medical history and takes no medications. His blood pressure is 156/98 mm Hg, which is elevated from his baseline of 120/72 mm Hg 1 year ago. His respiratory rate is 6-8/min. Physical examination shows no peripheral edema. Laboratory tests reveal a serum K+ level of 3 mEq/L, a serum pH of 7.49, and a reduced plasma renin activity. After confirming the diagnosis, the physician prescribes a drug to comprehensively treat the patient's symptoms. Which of the following is the most likely adverse effect of the drug that this patient was likely prescribed? A.Angioedema B.Gynecomastia C.Hyperuricemia D.Ototoxicity E.Pulmonary edema

B.Gynecomastia This patient presents with muscle weakness, episodic headache, frequent urination, hypertension, hypokalemia, and low plasma renin activity, which are all signs and symptoms highly suggestive of primary hyperaldosteronism, also known as Conn syndrome. In primary hyperaldosteronism, aldosterone secretion is excessive and non-suppressible, resulting in the classic presenting signs of hypertension and hypokalemia. It is most commonly caused by an aldosterone-secreting adenoma of the adrenal gland or bilateral adrenal hyperplasia.

An 8-year-old girl is brought for evaluation after her mother noticed some blood staining on the patient's underwear. The patient's mother states that it appears her daughter has started to develop breastsand is concerned that this is "happening too fast" because she is only 8 years old. The patient is otherwise healthy and takes no medications. Her vital signs are within normal limits. On physical examination, the patient is noted to have Tanner stage III breast budding, pubic hair, and underarm hair. Which of the following is the most appropriate next step? A.GnRH stimulation test B.Hand and wrist x-ray C.Hormonal analysis D.MRI E.Pelvic Ultrasound closeF.Reassurance

B.Hand and wrist x-ray A patient with Tanner stage III breast development, menarche, and pubic hair at 8 years old is should be evaluated for precocious puberty. When precocious puberty is suspected, the initial work-up involves radiography of the hands and wrists to determine the bone age. This helps determine if precocious puberty is actually present and, if so, its speed of progression. If the radiograph demonstrates bone age that is within 1 year of the chronological age, puberty has not started, or the pubertal process has been relatively brief. If the radiograph shows bone age that is 2 years or more from the chronological age, puberty has been present for a year or more, or is progressing rapidly.

A 34-year-old man is brought to the trauma unit with multiple gunshot wounds. The trauma team examines his entire body for the locations of the wounds. After restorative measures and appropriate treatment have been administered, the patient develops an abscess over a wound on his left flank. Wound cultures reveal methicillin-sensitive Staphylococcus aureus. Which of the following is the best way to reduce the risk of transmission of this microbe to other patients? A.Contact precautions B.Hand hygiene C.Isolation precautions D.Restricting visitors E.Wearing gloves

B.Hand hygiene Hand hygiene should be maintained: before eating; before and after having direct contact with a patient's intact skin (taking a pulse or blood pressure, performing physical examinations, lifting the patient in bed); after contact with blood, body fluids or secretions, mucous membranes, non-intact skin, or wound dressings, even if gloves are worn; after contact with inanimate objects (including medical equipment) in the immediate vicinity of the patient; when moving from a contaminated-body site to a clean-body site during patient care; after glove removal; and after using a restroom.

A 25-year-old woman presents to her physician with an upper respiratory tract infection. It is her fifth such infection in the past year. Along with her history of respiratory infections, she reports that she has a very poor sense of smell. She attributes her illness to increased stress at home, reporting that she and her husband have been trying to conceive unsuccessfully for 13 months. Physical examination reveals normal S1 and S2 heart sounds over the rightintercostal space in the mid-clavicular line. The apex beat is palpable over the right 5th intercostal space. Which organizational unit represents the structure most likely to be defective in this patient? A.Collagen α chains in triplets B.Heavy chains with coils and stems attaching clockwise from microtubule pairs toward adjacent pairs C.Multilobular nucleus with enzyme-filled granules D.Numerous adjoining E-cadherin molecules with actin filaments E.Two longer heavy chains and two shorter light chains linked by disulfide bonds

B.Heavy chains with coils and stems attaching clockwise from microtubule pairs toward adjacent pairs In Kartagener syndrome, there is a defect in dynein, leading to poor mucus clearance. Dynein arms are heavy chains with coils and stems that attach from a microtubule pair within the axoneme of cilia or flagella toward an adjacent pair. During a power stroke, the motor domain of the dynein arm undergoes a conformational change that causes the microtubule-binding stalk to pivot, resulting in one microtubule sliding relative to another. This produces the bending movement needed for cilia or flagella to beat and propel the cell. In men, the syndrome can lead infertility due to azoospermia, motile spermatozoa but immotile cilia, or immotile spermatozoa. In women, it can cause subfertility due to decreased fallopian tube function, leading to delay in the transit of the ovum from the ovary to the ampulla, the most common site of fertilization. Also, in rare cases, the decreased function of the fallopian tube may lead to ectopic pregnancy. Situs inversus is also associated with this disorder.Collagen α chains in triplets describes the structure of collagen fibrils. A defect in collagen structure results in connective tissue diseases such as scurvy, Ehlers-Danlos syndrome, or osteogenesis imperfecta. Numerous adjoining E-cadherin molecules with actin filaments describes the zona adherens junction. Modifications in the expression and function of adherens junction components can result in pathologic conditions such as cerebral cavernous malformation and alterations of vascular morphology seen in tumors. Multilobular nucleus with enzyme-filled granules describes neutrophils. A defect in neutrophils can result in conditions such as chronic granulomatous disease and Chédiak-Higashi syndrome, both of which affect immunity but would not cause infertility. Two longer heavy chains and two shorter light chains linked by disulfide bonds describes antibodies. A defect in antibody structure may affect immunity, but not fertility.

Physical examination of the child discloses fusion of the third and fourth digits of the hands bilaterally but is otherwise normal. Mutation of which of the following genes is most likely responsible for this patient's findings? A.FGF B.Hox C.SHH D.SRY E.Wnt-7

B.Hox This 2-month-old infant has a condition known as syndactyly, which can manifest as fusion of the fingers or toes. This condition can present itself for a variety of reasons; however, a common cause is due to dysregulation of Hox genes.Hox genes come from a family of homeobox genes that encode transcription factors. These transcription factors bind to the DNA and allow for activation or repression of specific genes. Hox genes are primarily responsible for patterning of the anterior-posterior axis of the vertebrate embryo and limb bud. A characteristic unique to Hox genes is that they are found in clusters, instead of scattered across the genome as most genes are. This clustering of the genes allows for the sequential expression during development to correctly form body parts. However, if there is a mutation in these genes it can cause syndactyly, polydactyly, an extra cervical rib, and an increased risk of cancer.

A 38-year-old woman presents to her primary care doctor complaining of recent bouts of nausea associated with dizziness. She says the episodes started 4 months ago, and tend to last 1-2 hours. A particularly severe flare-up this morning resulted in vomiting and buzzing in her right ear. On physical examination, she appears healthy with normal vital signs. There is no nystagmus. When a tuning fork is placed on her forehead, she says that the sound is better heard on the left. Sound from air conduction is greater than bone conduction in both ears, and the rest of her neurologic exam is normal. A hearing screen shows low-frequency hearing loss in the right ear. Basic lab test findings are unremarkable. Which of the following disease processes is most likely the cause of the patient's symptoms? A.Degeneration of hair cells in the cochlea B.Increased volume of endolymph C.Inflammation of the vestibulocochlear nerve D.Positional decrease in brain perfusion E.Presence of pathologic crystals in the semicircular canal

B.Increased volume of endolymph This patient presents with recurrent episodes of vertigo along with sensorineural hearing loss in the right ear. These findings are suggestive of Ménière disease, which classically manifests with a triad of sensorineural hearing loss, vertigo, and tinnitus. Although the etiology of Ménière disease is unclear, it is thought to be caused by an increase in the volume of endolymph (hydrops), which can result from either increased production or decreased excretion of the fluid contained in the membranous labyrinth of the inner ear. Malfunction of the endolymphatic sac, which is responsible for the filtration and excretion of endolymph, is one proposed mechanism.

A 27-year-old homeless man presents to the clinic because of a 5-day history of pain and swelling in his right upper arm. MRI of the area reveals diffuse soft tissue and bone inflammation. Bone biopsy is performed. Blood culture speciation is pending, but preliminary results have grown out a gram-negative, oxidase-positive rod. Which of the following is a complete history of this patient most likely to reveal? A.Corticosteroid use B.Intravenous drug use C.Miliary tuberculosis D.Multiple sexual partners E.Sickle-cell anemia

B.Intravenous drug use So that leaves IVDU, and such patients with osteomyelitis are most likely to be infected with S. aureus (a gram-positive coccus), Candida (a yeast), or Pseudomonas aeruginosa (a gram-negative, oxidase-positive rod.) P. aeruginosa enters the body via the needle puncture.

Which of the following best describes the effect of ketamine in critically ill patients? A.It decreases Km and Vmax B.It decreases substrate efficacy without affecting substrate affinity. C.It has no effect on substrate efficacy D.It increases EC50 E.It renders the substrate permanently ineffective.

B.It decreases substrate efficacy without affecting substrate affinity.

A 68-year-old woman presents to a physician with severe nausea and bloating. Symptoms are most prominent after a meal. Findings on physical exam are notable for epigastric tenderness and distention, with no guarding. The patient's last HbA1c was 9.2. A scintigraphic gastric emptying study shows gastric retention of 35% at 4 hoursafter the test. Upper endoscopy and imaging studies reveal no evidence of mechanical bowel obstruction. Which of the following is the best initial treatment for this patient? A.Esophageal resection B.Metoclopramide C.Omeprazole D.Ondansetron E.Vagotomy

B.Metoclopramide This patient presents with nausea, bloating, and increased gastric retention, with no evidence of mechanical bowel obstruction. She has also had an elevated HbA1cand gastric retention of 35% at 4 hours. Together these symptoms support a diagnosis of gastroparesis. Note that normal gastric retention is <10% after 4 hours. Although you are not expected to know this exact number, it should be apparent that retaining over one third of all gastric contents after 4 hours is not normal. The patient's high HbA1c is evidence of poorly controlled diabetes, a common cause of delayed gastric emptying. Metoclopramide, a promotility agent, is first-line therapy for this condition. Other drugs that are also effective for gastroparesis include domperidone and erythromycin. Esophageal resection is not appropriate in this patient, since there is no evidence of obstruction within the esophagus. Omeprazole is a proton pump inhibitor used to treat acid reflux. Although ondansetron is highly effective for treating nausea and vomiting, it would not address the underlying cause of this patient's symptoms. Vagotomy is an effective treatment for reducing acid secretion, but it would not address this patient's gastric retention.

A 60-year-old woman comes to her primary care physician because of a runny nose, headache, and aching muscles in her arms, legs and chest that began 2 days ago. She did not receive the influenza vaccine this year. Rapid influenza screening confirms the diagnosis of influenza A. Oseltamivir is prescribed. Which of the following processes begins the clearance of the infection? A.Directly kill virally infected cells B.Perforin- and granzyme-induced apoptosis C.Produce anti-inflammatory cytokines D.Produce antibodies E.Release azurophilic granules to kill cells F.Release major basic protein

B.Perforin- and granzyme-induced apoptosis The innate immune system is responsible for rapid recognition and clearance of initial infections but does so indiscriminately. It includes cells of the myeloid lineage (eg, neutrophils, monocytes, and macrophages), natural killer (NK) cells, the complement system, and physical barriers such as mucous membranes. Although they are members of the lymphoid cell lineage, NK cells are the component of the innate defense responsible for eliminating intracellular, viral infections, such as influenza A via apoptosis. Additionally, they play a role in killing cancerous cells. NK cells accomplish cell destruction by first releasing perforins that create pores in the cell membrane, followed by release of granzyme, which activates BCL-2, a pro-apoptotic mitochondrial mediator. When NK cells come into contact with cells that lack major histocompatibility complex (MHC), this pro-apoptotic pathway is induced, and the cell perishes. NK cells also utilize a process called antibody-dependent-cell-mediated cytotoxicity, which utilizes the CD16 receptor binding to the Fc portion of cell-bound antibodies, triggering degranulation into a lytic synapse.

A 52-year-old man is referred to the cardiology clinic because of chest pain and shortness of breath during his morning walks that quickly resolves with rest. Stress echocardiography findings are consistent with exercise-induced ischemia, and the patient is sent for cardiac catheterization. During catheterization, pressure readings at one location range from 9 to 25 mm Hg. As the catheter is advanced forward, the pressure range subsequently becomes 9 to 12 mm Hg. Assuming these findings are normal, the catheter is located within which of the following cardiac structures during the first pressure reading? A.Left ventricle B.Pulmonary artery C.Pulmonary artery wedge D.Right atrium E.Right ventricle

B.Pulmonary artery Once during this patient's cardiac catheterization, the catheter recorded a pressure reading of 9-25 mm Hg. Upon advancement of the catheter, the pressure range subsequently became 9-12 mm Hg. These values are consistent with an initial reading in the pulmonary artery (normal: 4-30 mm Hg), with the catheter then being advanced to the pulmonary capillary wedge (a much smaller range, 4-12 mm Hg).

A 49-year-old man with known esophageal varices is brought to the emergency department after falling while crossing the street. On arrival, he is mildly combative, and he is not oriented to where he is or the current year. He is also unable to state how he got to the hospital. On physical examination his vital signs are within normal limits, but he is unable to track the physician's finger from side to side. Laboratory findings are significant for a mild macrocyticanemia, but no hypersegmented neutrophils are seen. Imaging is negative for any cerebral lesions. The vitamin deficient in this patient serves as a cofactor for which of the following enzymes? A.Δ-Aminolevulinate synthase B.α-Ketoglutarate dehydrogenase C.Dopamine hydroxylase D.Homocysteine methyltransferase E.Pyruvate carboxylase

B.α-Ketoglutarate dehydrogenase This patient presents with ataxia, ophthalmoplegia, and confusion, which are symptoms of Wernicke encephalopathy. This syndrome is caused by a thiamine deficiency that is most commonly seen in malnourished alcoholic patients. Other clues that this is an alcoholic patient include the known history of esophageal varices, witnessed fall, disorientation, combativeness, and inability to track the physician's finger. Thiamine, also known as vitamin B1, is used as a cofactor for multiple enzymes, including α-ketoglutarate dehydrogenase, pyruvate dehydrogenase, and transketolase. Homocysteine methyltransferase is an enzyme involved in folate metabolism, in which it acts to convert homocysteine to methionine. Cyanocobalamin (vitamin B12) is the cofactor for this enzyme. A deficiency can result in megaloblastic anemia and peripheral neuropathy.

A 24-year-old man comes to the emergency department after experiencing hand pain for 1 week after injuring the hand while skiing. Physical examination reveals tenderness over the scaphoid tubercle, significant bruising, and absent radial pulse. However, the hand is warm and color returns to the hand within 10 seconds after release of compression of the artery on the medial side of the wrist while the lateral side remains compressed. Ultrasonography confirms complete clotting in the radial artery. Which of the following arteries have ensured adequate collateral arterial flow in this patient's hand despite his injury? A.Anterior and posterior interosseous arteries B.Arcuate artery C.Arteries of deep and superficial palmar arch D.Common and proper palmar digital arteries E.Radial collateral and recurrent radial arteries

C.Arteries of deep and superficial palmar arch

A 3-year-old girl is brought to her pediatrician because of a progressive loss of motor function and a decline in her cognitive abilities. On physical examination, it is noted that the patient has decreased deep tendon reflexes, truncal ataxia, and a decreased attention span in comparison with her last visit 6 months ago. A deficiency of which of the following enzymes leads to this condition? A.α-Galactosidase A B.β-Galactocerebrosidase C.Arylsulfatase A D.Hexosaminidase A closeE.Sphingomyelinase

C.Arylsulfatase A Arylsulfatase A converts sulfatide to galactocerebroside. This enzyme is deficient in patients with metachromatic leukodystrophy, an autosomal recessive lysosomal storage disease in which patients cannot degrade sulfatides, leading to accumulation of cerebroside sulfate in both neuronal and nonneuronal tissues. There is abnormal myelination with widespread loss of myelination in the central nervous system and peripheral nerves, leading to the clinical signs. Metachromatic granules can be seen on histologic examination.

An obese 43-year-old woman is brought to the emergency department because of abdominal pain and fever. The abdominal pain is intermittent, located in the epigastrium is provoked by food consumption. While brushing her teeth 2 days ago, she noticed that her eyes were uncharacteristically yellow. She has no medical history and takes no medications. Her temperature is 38.8 °C (101.8 °F), pulse is 92/min, blood pressure is 118/72 mm Hg, and respiratory rate is 17/min. Physical examination reveals jaundice and scleral icterus. There is right upper quadrant tenderness without rebound, rigidity or guarding. There is no abdominal organomegaly. Laboratory testing shows: RBCs: 4.1 million/mm3Hb: 12.3 g/dLHct: 36.1%WBC: 14,200/mm3 (with 74% segmented neutrophils)Platelets: 359,000/mm3 Lipase: 78 U/L (normal range 0-160 U/L). Which of the following is most likely responsible for this patient's condition? A.Acute pancreatitis B.Acute viral hepatitis C.Bacterial cholangitis D.Cholelithiasis E.Primary biliary cholangitis

C.Bacterial cholangitis This patient presents with the classic Charcot triad of right upper quadrant pain, jaundice, and fever, typically found in patients with cholangitis.

A 20-year-old man presents to the physician with a facial lesion on his left cheek. It began as a 1-cm painless indurated mass but over the the past 3 weeks has grown to a swollen localized area, 5 cm in diameter, with several abscesses draining to the skin. The patient mentions that yellow discharge sometimes comes out of the mass. His medical history is notable for oral surgery 1 month earlier. He denies any fevers, chills, weight loss, sore throat, cough, or changes in bowel movements or urination.Vital signs are a temperature of 37.7°C (99.8°F), blood pressure of 132/73 mm Hg, pulse of 78, and respiratory rate of 14. His left jaw is not tender or erythematous but is indurated. A yellow discharge is expressed and examined under the microscope with a modified acid-fast stain known as the Fite-Faraco stain (as shown in the image). Which of the following best describes the mechanism of action of the antibiotic most likely prescribed? A.Binds ergosterol, forming pores in the membrane B.Binds to 50S ribosomal subunit C.Blocks bacterial cell wall synthesis by inhibiting transpeptidase cross-linking D.Blocks bacterial nucleotide synthesis E.Inhibits ergosterol synthesis

C.Blocks bacterial cell wall synthesis by inhibiting transpeptidase cross-linking This patient presents with a painless, growing mass that periodically exudes a yellow discharge. In the context of his oral surgery 1 month earlier, his symptoms suggest a related dental infection caused by Actinomyces israelli, an obligate anaerobe that is part of the normal oral flora.The genus Actinomyces is closely related to the genus Nocardia. Both are unique among bacteria in that they exhibit a branching, rod-like morphology. A major difference between the two genera is that Nocardia organims are weakly acid-fast positive, whereas Actinomyces organisms are acid-fast negative. The image of the modified acid-fast stain shows no acid-fast bacilli, helping to exclude Nocardia as the cause of the infection. The large blue regions in the image are sulfur granules that are made up of mass growth of the actinomycetes. On Gram staining and microscopic examination, A israelli appear as gram-positive branching rods with clustering that froms "sulfur granules" in the thick yellow exudate (as described previously). Trauma associated with dental work often leads to the invasion of A israelli into the cervicofacial area. Infection caused by this organism typically presents as a chronic, slowly progressing mass that eventually evolves into a draining sinus tract. First-line treatment involves administration of penicillin G, an antibiotic that inhibits transpeptidase cross-linking and thus prevents cell wall synthesis, and surgical debridement. Sulfonamides, which are first-line treatment for Nocardia infections, act by blocking bacterial nucleotide synthesis. Macrolides exert their antibacterial effect by binding to the 50S ribosomal subunit, but they lack anaerobic coverage. Amphotericin B, which is used to treat systemic mycoses, binds ergosterol, forming pores in the membrane. Azoles, which are also used to treat fungal infections, work by inhibiting ergosterol synthesis.

After the patient is transferred to the intensive care unit (ICU), the decision is made to insert a central venous catheter., Which of the following interventions has been shown to prevent intravascular catheter-related infections? A.Application of topical antimicrobial ointment at the site of insertion B.Catheter replacement every week C.Daily chlorhexidine bathing D.Daily vancomycin therapy E.Femoral vein as a preferred site of insertion F.Prophylactic amoxicillin therapy before catheter insertion

C.Daily chlorhexidine bathing Daily chlorhexidine bathing is an effective way to reduce the incidence of catheter-related bloodstream infections and has minimal adverse effects; it is recommended for all patients in the ICU. Other preventive measures include: Choosing appropriate sites for catheter insertion (femoral vein is less preferred) Using the appropriate type of catheter material (antimicrobial-impregnated catheters) Using barrier precautions during and after insertion (site preparation, sterile techniques, dressing and daily chlorhexidine bathing) Changing catheter administration sets at appropriate intervals Ensuring proper catheter site care Ensuring removal of catheters when no longer essential

A 63-year-old man comes to his primary care physician for a well-patient examination. During this visit, the patient admits that he is having difficulty hearinghis wife clearly when she speaks to him. However, he seems able to hear the cowboys on the television shows he watches, which angers his wife. The physician orders audiology testing but also reassures the patient that his pathology is most likely part of the normal process of aging, and not a newfound aversion to his wife. Which of the following changes is most likely to be found in this patient? A.Compression of cranial nerve VIII as it exits the internal auditory meatus B.Degeneration of the distal hair cells at the apex of the organ of Corti C.Degeneration of the proximal hair cells of the organ of Corti D.Degeneration of the utricle and the saccule E.Trauma to the cochlear nuclei

C.Degeneration of the proximal hair cells of the organ of Corti Age-related degeneration of the organ of Corti (or presbycusis) is a type of sensorineural hearing loss that is part of the normal aging process. It is the most common cause of sensorineural hearing loss in the elderly and manifests with gradually worsening high-frequency hearing loss. The cause of presbycusis is degeneration of the proximal hair cells of the organ of Corti (depicted in the drawing), which lie in the proximal portion of the basilar membrane (farthest from the helicotrema or apex and closest to the ossicles and oval window). This is the thinnest and least compliant portion of the basilar membrane and is responsible for sensing high-frequency sounds.

A 51-year-old man comes to the primary care physician because of concerns about burning sensation and discomfort in his legs. He has a history of high cholesterol and hypertension but has been noncompliant with pharmacotherapy. He works in an office all day and enjoys coming home and watching TV until bedtime. Temperature is 98.6° F (37° C), pulse is 110/min, and respiratory rate is 18/min. His blood glucose level is 285 mg/dl, and his BMI is 30. Which of the following most likely have prevent this patient's current symptoms? A.Compliance with cholesterol medication B.Compliance with hypertension medication C.Exercise D.Proper diet E.Proper sleep

C.Exercise Diabetes is managed by lifestyle modifications, oral therapy, and insulin injections. Of all the answer choices, exercise would have been the most effective factor in preventing or reducing complications from type II diabetes. Exercise leads to increased insulin sensitivity, which can prevent atherosclerosis, glycosylation of end products, and osmotic changes. This in turn reduces the risk of complications related to type 2 diabetes. Tight blood sugar control with medical compliance to control blood pressure and cholesterol and a proper diet helps prevent diabetic nephropathy. A proper diet is key to regulating BMI and blood glucose levels, but is not as effective as exercise in preventing diabetic neuropathy.

A 3-year-old girl is brought to the emergency department by her mother because of recent onset of shortness of breath. Her mother reports that the girl was playing in the garden when the symptoms started. Her mother mentions that the patient is allergic to pollen. Respirations are 25/min. On physical examination, no rashes are found on the child's body. Wheezing is present bilaterally. She is treated adequately with medication. The second messenger responsible for alleviating the patient's symptoms most likely acts through which of the following pathways? A.Gi pathway B.Gq pathway C.Gs pathway D.Nonreceptor tyrosine kinase pathway E.Receptor tyrosine kinase pathway

C.Gs pathway

A 63-year-old woman Leukocyte count: 64,000/mm3Differential: 70% neutrophils, 20% lymphocytes, 5% eosinophils, 3% bands, 2% promyelocytes and 1% myelocytes. No blasts are seen. Which of the following is the most appropriate treatment? A.Daunorubicin and cytarabine B.Fludarabine and rituximab C.Imatinib D.Leucovorin E.Observation

C.Imatinib The lab test results reveal leukocytosis, anemia, and thrombocytosis (can be normal), strongly suggesting a diagnosis of chronic myelogenous leukemia (CML). This disease is typically associated with the Philadelphia chromosome, a t(9;22) translocation that results in a fusion gene (bcr-abl) that encodes a constitutively active tyrosine kinase (Bcr-Abl protein).Imatinib has proven to be a very effective treatment for CML. Imatinib is a tyrosine kinase inhibitor that targets the abnormal bcr-abl tyrosine kinase produced by the Philadelphia chromosome translocation. Imatinib inhibits proliferation and induces apoptosis in cells positive for BCR/ABL. It has helped reduce the annual mortality rate for patients with CML from 15%-20% (before 2000) to 1%-2% today.

A biotechnology company is attempting to develop a new rapid-acting reversal agent for anaphylactic shock. The company's goal is to prevent the reaction from occurring by stopping the cascade of reactions that leads to the symptoms of anaphylactic shock. Multiple targets are considered for investigation. Which of the following molecules is the most promising target for blocking this pathway? A.C5b B.Hageman factor C.Immunoglobulin surface receptor D.Interferon-γ E.Interleukin-1 F.Nitric oxide

C.Immunoglobulin surface receptor Therefore the company should consider finding a way to prevent the initial cross-linking of the IgE receptors. If a surface receptor of IgE is blocked, the cross-linking cannot occur, and the anaphylactic reaction can be stopped.Symptoms of anaphylactic shock include itchy skin, difficulty breathing as airways swell, dizziness, fainting, and nausea or diarrhea.The other answer choices are not related to the cascade seen in anaphylactic shock. Interferon-γ is important for immunity against viruses and bacteria and, among many functions, acts to activate macrophages. C5b is a component of the membrane attack complex, which is an important part of the complement system. Hageman factor is the name for a form of factor XII that can activate the intrinsic pathway of the coagulation cascade. Interleukin-1 causes fever, which is not seen in anaphylaxis. Nitric oxide is usually an important mediator of septic shock but is not the primary mediator involved in the mechanism of anaphylaxis.

An 18-year-old woman comes to the clinic with a "stomach ache." Upon further investigation, she describes the pain as crampy and says it only occurs during menstruation. Ibuprofen provides only partial relief. She has no significant past medical history and previous investigations into her pain have been inconclusive. Her last menstrual period 4 weeks ago was regular. She is sexually active and endorses consistent condom use. She denies constipation, painful sexual intercourse, or heavy menstrual bleeding. Her temperature is 98.7° F (37° C), blood pressure is 110/72 mm Hg, pulse is 71/min, respiratory rate is 14/min, and oxygen saturation is 99% on room air. Her abdomen is soft and nontender, though she endorses that the pain occurs around the suprapubic area. Her uterus is anterior, not enlarged, and nontender. Bilateral adnexa are nontender to palpation. A urine beta-hCG is negative. Ultrasound evaluation reveals no significant anatomic changes. Which of the following is the most likely cause of the patient's condition? A.Ectopic endometrial tissue B.Follicle growth causing ovarian distension C.Increased prostaglandin production D.Ovarian cysts measuring greater than 8mm in diameter E.Smooth muscle neoplasm F.Thickened myometrium

C.Increased prostaglandin production The patient is an otherwise healthy, sexually active 18-year-old female presenting with abdominal pain occurring during her menstrual cycle. Her physical exam is unremarkable and urine pregnancy test is negative. Thus, her pain is most suggestive of primary dysmenorrhea.

A 74-year-old nursing home resident is brought to the emergency department (ED) after her caretaker noticed she was having difficulty speaking since waking up that morning. Her past medical history is notable for atrial fibrillation, long-standing diabetes, hypertension, and a 40-pack-year smoking history. Her vital signs are within normal limits, and she is able to follow commands, but her speech is labored and clipped. Which of the following vessels is most likely to have been occluded in this patient? A.Anterior cerebral artery B.Anterior communicating artery C.Left middle cerebral artery D.Posterior communicating artery E.Right middle cerebral artery

C.Left middle cerebral artery Broca (expressive) aphasia, a neurologic insult to the motor speech area of the brain. Patients with Broca aphasia have difficulty enunciating words, although they can understand both written and verbal language. In 95% of the right-handed population and approximately 90% of the left-handed population, the language centers are located in the left cerebral hemisphere, in a vascular area supplied by the left middle cerebral artery.

Which of the following is most likely to be normal in a patient with ARDS? A.Alveolar arterial gradient B.Diffusing capacity of carbon dioxide (DLCO) C.Left ventricular end diastolic pressure D.Lung compliance E.PaO2/FiO2 ratio F.Pulmonary arterial pressure

C.Left ventricular end diastolic pressure The diagnosis of ARDS is characterized by a PaO2/FiO2 ratio of ≤ 300 and bilateral infiltrates on chest x-ray, as shown on this patient's x-ray. Cardiogenic pulmonary edema can look very similar on imaging, so must be ruled out based on combination of history, BNP (high negative predictive value for congestive heart failure at levels <100pg/mL, high positive predictive value for values > 500 pg/mL), transthoracic echocardiogram (to evaluate cardiac function) and/or normal PCWP, or pulmonary capillary wedge pressure (normal: 6-12 mmhg). Pulmonary capillary wedge pressure serves as a measure of left ventricular end diastolic pressure. It is likely to be increased in cases of cardiogenic pulmonary edema secondary to left ventricular failure or in cases of volume overload). Left ventricular end diastolic pressure is normal in patients with acute pulmonary injury due to ARDS.

This patient's history and physical findings and an ECG showing coarse, irregularly shaped atrial conduction with no individual P waves and the presence of ventricular tachycardia indicate that he is experiencing atrial fibrillation (AF) causing supraventricular tachycardia (SVT). Specifically, the irregularly shaped, undulating P waves, which are sometimes almost absent in some leads, combined with ventricular tachycardia (of about 150 beats/min) are indicative of AF. Which of the following describes the mechanism of the medication that the physician most likely prescribes to treat? A.Decrease in conduction velocity through the Purkinje fibers B.Increase in cardiac dromotropy C.Negative dromotropy at the atrioventricular node D.Prolongation of the QT interval

C.Negative dromotropy at the atrioventricular node Diltiazem is a calcium channel blocker commonly used for short-term management of SVT. By inhibiting the voltage-dependent calcium channels in cardiac pacemaker cells and slowing the rate of recovery, diltiazem will decrease the rate at which impulses generated by ectopic foci in the atria are transmitted through the atrioventricular (AV) node. As a result, the PR interval will be prolonged, and the ventricles will be protected from rapid successions of impulses generated from the ectopic atrial foci, making this medication useful for preventing nodal arrhythmias.

Which of the following laboratory results is most likely to be present in ALL? A.Decreased serum lactate dehydrogenase B.Decreased serum uric acid C.Neutropenia D.Polycythemia E.Thrombocytosis

C.Neutropenia Neutropenia (along with anemia and thrombocytopenia) is present on the CBC and blood smear. Patients typically present with signs and symptoms of bone marrow failure, such as fatigue (due to anemia), infection (due to neutropenia), and nosebleeds or petechiae (due to thrombocytopenia).

A 23-year-old woman comes to the clinic because of pruritic warts on her vagina that appeared 3 days ago. Over the past year, she states she has had multiple sexual partners with inconsistent condom use, and she adds that she began having sex at a young age. Physical examination shows raised, flesh-colored lesions on the mucous surfaces of her vagina. A biopsy shows the presence of koilocytes in the tissue. Which of the following best describes the characteristic feature of this patient's causal organism? A.Double-stranded, enveloped, linear DNA virus B.Double-stranded, non-enveloped, linear DNA virus C.Non-enveloped, double-stranded, circular DNA virus D.Single-stranded, negative sense and circular RNA virus E.Smallest DNA virus (only non-enveloped, single-stranded linear DNA virus)

C.Non-enveloped, double-stranded, circular DNA virus Human papillomavirus (HPV) is a non-enveloped DNA virus from the herpesvirus family that is double-stranded and circular. It is transmitted from skin-to-skin contact from one epithelial surface to another. HPV infection leads to cutaneous warts, genital warts/condylomata acuminata, cervical intraepithelial neoplasia (CIN), and cervical cancer. Genital warts are associated with HPV serotypes 6 and 11, while CIN and cervical cancer is most commonly associated with serotypes 16 and 18. Of note, the papillomavirus can structurally modify squamous epithelial cells and create cells called koilocytes.

A 25-year-old woman presents to her primary care physician for an annual physical examination. During the visit she describes noticing less than a teaspoon of thick white vaginal discharge daily. Vaginal pH is 4.0. Saline microscopy of scant discharge found at the external os identifies epithelial cells with scarce polymorphonuclear leukocytes. What is the best description of these clinical findings? A.Bacterial vaginosis B.Gonococcal cervicitis C.Normal D.Trichomoniasis E.Vulvovaginal candidiasis

C.Normal This patient has a vaginal pH of 4.0 and thick white vaginal discharge daily that she describes as being less than a teaspoon in volume. Both her pH level and the amount and quality of her discharge are normal. The normal vaginal pH range is between 4.0-4.5. Moreover, women normally have 1-4 mL of clear to white to yellowish, mostly odorless vaginal discharge ("physiologic leukorrhea") daily. Not all discharge is pathologic.

An isotonic gastrointestinal fluid has the following flow-dependent composition when increasing flow rate: same Na, same K, incr HCO3, decr Cl Which of the following organs secretes fluid with this chemical composition? A.Duodenum B.Gallbladder C.Pancreas D.Salivary glands E.Stomach

C.Pancreas The exocrine pancreas is composed of duct epithelial cells and acinar cells. The duct epithelial cells are responsible for the secretion of bicarbonate (HCO3-)-rich fluid (isotonic sodium bicarbonate) that is added to the fluid secreted by the pancreatic acinar cells. The acinar cells are responsible for the secretion of pancreatic enzymes in an isotonic sodium chloride (NaCl) solution. The HCO3- and chloride (Cl-) in pancreatic secretions have an opposite relationship because they are transported via an antiporter that brings Cl- into the cell while simultaneously sending HCO3- out of the cell. When a meal is consumed, secretin is released by the S cells in the duodenum. This stimulates the exocrine pancreas to increase the flow rate of its secretions. This causes an increased concentration of HCO3- in the secretions, which helps neutralize the acidic stomach contents that will enter the duodenum. The neutralization of the acidic chyme is necessary because the pancreatic enzymes work best at a higher pH than the stomach enzymes. When in the resting state, the pancreas is not stimulated to release its secretions at a high flow rate. This causes a decreased level of HCO3- in the secretions due to a lack of acidic contents in the intestines. These pancreatic enzymes include amylase, lipases, proteases and trypsinogen. Alpha amylase is secreted in its active form and is important for starch digestion. Lipases are key components in fat digestion. Protein digestion requires proteases that are secreted as proenzymes (zymogens). These proteases include trypsin, chymotrypsin, elastase and carboxypeptidase. Trypsinogen is converted to its active form of trypsin by enterokinase, an enzyme found on the brush border of the duodenum and jejunum. Active trypsin then activates other proenzymes and can cleave additional trypsinogen molecules. This is a positive feedback loop.

A 34-year-old man who is HIV-positive comes to the emergency department becauase of fever, cough productive of sputum, hemoptysis, and malaise. He has never taken antiretroviral therapy before and denies history of any recent infections. His current CD4 count is 500 cells/mm3 and HIV-1 RNA copies by PCR is 10,000/mL, performed a month ago. His temperature is 101° F (38.3° C), blood pressure is 110/60 mm Hg, pulse is 104/min, and respiratory rate is 21/min. On examination, he appears to be in moderate distress and needle track marks are present on both sides of his forearms. Chest x-ray reveals infiltrates in the left middle lobe, and acid-fast staining shows rod-shaped bacilli under microscopy. In addition to standard precautions, which of the following should be used with this patient? A.Admit the patient in a private room B.Dispose of used needles promptly C.Place the patient in a negative pressure room D.Set positive pressure in the patient room E.Wear a surgical mask when within 3 ft of the patient

C.Place the patient in a negative pressure room This patient has a productive cough, hemoptysis, fever, and malaise consistent with tuberculosis. This is especially concerning given the acid-fast bacilli on sputum. Tuberculosis infection warrants airborne protection, which includes placing the patient in a negative pressure room. M. tuberculosis is an aerobic, acid-fast bacilli that infects the upper lobes of the lungs. Hygiene and precautions help to prevent transmission of pathogens. Specifically, these patients require additional aerosol precautions to prevent inhalation of airborne pathogens such as M. tuberculosis. These include placing patient in an airborne infection isolation room, use of N95 respirator mask with confirmed or suspected TB patients and minimizing patient transport.

A 3-week-old infant is brought to the clinic because of fussiness and a fever. The infant has been vomiting, refusing feedings, and crying more than usual. On evaluation, the patient's temperature is 39°C (102.2°F), pulse is 10/min, respirations are 30/min, and blood pressure is 90/45 mm Hg. Results of lumbar puncture show a leukocytecount of 25/mm3, protein concentration of 170 mg/dL, and glucose concentration of 20 mg/dL. The physician suspects bacterial meningitis. A culture of the patient's spinal fluid shows gram-negative bacilli. Which of the following additional findings would aid in identifying the most likely causative organism? A.Positive catalase test B.Positive for obligate aerobic metabolism C.Positive lactose fermentation test D.Positive oxidase test E.Positive urease test

C.Positive lactose fermentation test Because of the infant's presentation and the lumbar puncture findings, the physician suspects bacterial meningitis. In infants 0-6 months old, group B streptococci, Escherichia coli, and Listeria are the most common bacterial causes of meningitis. Group B streptococci are gram-positive cocci and the most common causative agent, usually because of colonization of the mother's vaginal tract. However, since the culture of the patient's cerebrospinal fluid reveals a gram-negative bacillus, E. coli is the likely culprit because it is the only one of the three that is a gram-negative bacillus (rod). This patient's positive lactose fermentation test would aid in identifying the causative organism as E. coli.

A 47-year-old woman comes to her physician's office because of nausea and pain in her upper abdomen. She says that she achieves mild relief of her symptoms when she eats and that this has caused her to gain weight. The patient's past medical and family history is unremarkable. Her temperature is 37.2° C (98.9° F), blood pressure is 127/83 mm Hg, pulse is 82/min, and respirations are 13/min. Physical examination shows a soft, nondistended abdomen with marked tenderness in the epigastric region. An upper endoscopy is performed and shows hypertrophy of glandsresponsible for bicarbonate secretion the duodenal submucosa. Which of the following regulatory substances stimulates the release of a substance produced by the hypertrophied glands seen in this patient? A.Gastrin B.Pepsin C.Secretin D.Somatostatin E.Vasoactive intestinal polypeptide

C.Secretin This 47-year-old woman has epigastric pain that decreases with meals, nausea, and weight gain. An esophagogastroduodenoscopy shows hypertrophy of the Brunner glands, which is seen in peptic ulcer disease. The patient's statement that the pain improves with food is a stronger indicator for duodenal ulcer versus gastric ulcer; with the latter diagnosis, pain increases with meals resulting in weight loss. Brunner glands are responsible for the secretion of bicarbonate. Secretin, which is released by the S cells in the duodenum, is responsible for increasing bicarbonate and bile secretion while decreasing gastric acid secretion. (In a healthy individual, increased secretion of bicarbonate occurs due to acid and fatty acids in the duodenum and causes the Brunner glands and pancreas to secrete HCO3-)

A 40-year-old nulligravid woman comes to her gynecologist because of irregular periods for the past 6 months. She also reports recent-onset bilateral nipple discharge (galactorrhea). Her medical history is significant for type 2 diabetes, hypothyroidism, Tourette syndrome, and a seizure disorder. She is compliant with all the medications prescribed for each condition. A urine pregnancy test is negative. Treatment for which of the following conditions is most likely the cause of the patient's symptoms? A.Hypothyroidism B.Seizure disorder C.Tourette syndrome D.Type 2 diabetes mellitus

C.Tourette syndrome This patient is compliant with all her medications. A common treatment for Tourette syndrome is antipsychotics, which act as dopamine antagonists. Prolactin secretion is tonically inhibited by dopamine. Therefore, dopamine inhibition (from antipsychotic drugs) leads to increased secretion of prolactin, and the resultant symptoms of hyperprolactinemia.

what do I cells secrete in the GI tract? what does that substance do?

CCK: contract gallbladder, incr pancreatic enzyme secretion

which mutation confers immunity to HIV?

CCR5 homozygous

what CD marker does G-CSF activate?

CD-34

All T cells are positive for what CD? [what does this do?] What are the sxs of DiGeorge syndrome? what causes it?

CD3; [associates with the TCR for signal transduction] Cardiac defects Abnormal face Thymic hypoplasia/aplasia Cleft lip/palate Hypocalcemia 22 deleted CATCH 22

what does gp120 bind?

CD4

The patient is a 28-year-old gravida 2, para 1 woman who presented with calf tenderness, warmth, and pain with non-compressible veins on ultrasound. This is most suggestive of deep vein thrombosis (DVT). Which of the following is the most appropriate pharmacotherapy for this patient? A.Apixaban B.Argatroban C.Clopidogrel D.Enoxaparin E.Unfractionated heparin F.Warfarin

D.Enoxaparin compared to unfractionated heparin, LMWH has a reduced incidence of adverse effects such as osteoporosis and thrombocytopenia and a greater ease of administration (once daily, no aPTT monitoring required).

During a routine yearly physical examination, a previously healthy 48-year-old woman is found to have a blood pressure of 150/85 mm Hg. She has had no physical symptoms and no history of palpitations, tremors, diaphoresis, or other signs of cardiovascular disease. She begins strictly adhering to a diet and exercise program, but on her next visit, her blood pressure is 170/95 mm Hg. A physical exam reveals an abdominal bruit, and a blood test demonstrates elevated plasma renin levels. Results from her renal function test reveal: Na+: 137 mEq/LK+: 3.2 mEq/LCl-: 100 mEq/LHCO3-: 27 mEq/LBUN: 26 mg/dLCreatinine: 1.0 mg/dL Which is the most likely explanation for this patient's condition? A.Aldosterone-producing tumor B.Atherosclerosis of the renal artery C.Essential hypertension D.Fibromuscular dysplasia E.Pheochromocytoma

D.Fibromuscular dysplasia In this young, healthy woman with sudden-onset hypertension, it is important to consider atypical causes of hypertension including fibromuscular dysplasia, pheochromocytoma, and an aldosterone-producing tumor. This patient's relatively young age without comorbidities, elevated renin, abdominal bruit, and lack of associated symptoms of pheochromocytoma (tremor, palpitations) make fibromuscular dysplasia (FMD) the most likely diagnosis. Fibromuscular dysplasia can lead to concentric thickening of the renal artery wall, leading to decreased luminal diameter and renal blood flow.

A 35-year-old man comes for evaluation of a large, non-painful bump on the posterior side of his neck. It started as a small bump several months ago, which he forgot about, but it recently became more noticeable and is causing him emotional distress. Physical examination shows a 3-cm x 3-cm x 3-cm enlarged suboccipital lymph node that is painless to palpation. Examination of a node biopsy specimen is characteristic of Hodgkin lymphoma. A chemotherapeutic regimen that includes procarbazine is initiated. The patient is warned that he must follow certain dietary precautions while undergoing therapy. The precautions are to prevent which of the following adverse effects? A.Drug inactivation B.Dystonia C.Hypercholesterolemia D.Hypertensive crisis E.Serotonin syndrome

D.Hypertensive crisis Procarbazine is an alkylating agent used in cancer chemotherapy that also inhibits monoamine oxidase (MAO). MAO in the gut normally degrades tyramine obtained from the diet. Therefore, patients taking procarbazine should be cautioned against ingesting tyramine-containing foods (eg, aged cheeses, chicken liver, beer, and wine). Excess tyramine causes nerve terminals to release large amounts of catecholamines. In addition to monitoring dietary intake, patients should avoid over-the-counter medications with sympathomimetic properties, such as ephedrine, pseudoephedrine, and phenylpropanolamine, which are components of several cough medicines and decongestants. The massive release of catecholamines can cause a hypertensive crisis with symptoms of headache, nausea, hypertension, tachycardia, and possibly cardiac arrhythmias and stroke. The other answer options are incorrect: Dystonia is induced by antipsychotics and is not dependent on diet. Hypercholesterolemia occurs secondary to genetic and dietary factors. Some drugs, such as protease inhibitors and anti-diabetic medications, also can cause an increase in cholesterol. However, procarbazine does not affect cholesterol concentrations. Drug inactivation is a concern with many types of drugs that inhibit the drug directly (eg, antacids with magnesium or calcium inhibiting the efficacy of doxycycline) or indirectly via activation of the CYP450 system in the liver (eg, omeprazole accelerating the degradation of other CYP450 dependent drugs). This is not a concern with procarbazine. Serotonin syndrome is a concern with monoamine oxidase (MAO) inhibitors in conjunction with other drugs, not diet.

Which of the following laboratory abnormalities would most likely be due to an adverse effect of levodopa? A.Decreased urobilin levels B.Increased direct bilirubin level C.Increased haptoglobin D.Increased indirect bilirubin level E.Negative Coombs test

D.Increased indirect bilirubin level Although rare, one of the adverse effects of levodopa is drug-induced immune hemolytic anemia (DIHA). The drug stimulates the body's immune system to produce antibodies directed against its own RBCs. These antibodies, or immunoglobulins (Ig), attach to the RBCs, then are phagocytosed by reticuloendothelial cells. As the RBCs break down, the heme is metabolized to biliverdin, then to unconjugated bilirubin; thus, a rise is seen in the indirect (unconjugated) bilirubin level. Because the rate of RBC destruction is accelerated in DIHA, the breakdown products accumulate in the blood and may cause jaundice, dark urine, and decreased hemoglobin and hematocrit levels. Haptoglobin decreases, not increases, because it binds to the hemoglobin released as RBCs break down and are removed from circulation. Urobilin levels, however, increase rather than decrease as excess bilirubin is metabolized in the liver and then excreted in the urine. Direct bilirubin levels are normal. The Coombs testmeasures the presence of IgG or complement on the surface of RBCs and thus would be positive.

A 70-year-old woman is brought to the physician by her daughter for evaluation of memory impairment. The daughter states that her mother has become increasingly forgetful over the past year and has even gotten lost driving on several occasions while taking familiar routes. She also has difficulty performing simple household chores like cooking and laundry, which she was able to perform previously. Recently, she has become more reserved, avoiding social gatherings. The recommended treatment exerts which of the following effects on the central nervous system? A.Agonism at GABA receptors B.Antagonism at β-adrenergic receptors C.Conversion into active neurotransmitter D.Inhibition of acetylcholinesterase E.Inhibition of dopaminergic activity F.Inhibition of neurotransmitter reuptake G.Interaction with cation transport

D.Inhibition of acetylcholinesterase The patient is brought to the physician by her daughter, who helps explain the patient's cognitive decline. The combination of memory impairment, inability to perform activities of daily living, and retreat from the world suggests that she has Alzheimer disease. Although no cure for AD exists, donepezil (a cholinesterase inhibitor/indirect cholinomimetic) may improve cognitive measures by increasing the amount of available acetylcholine at the synapse. It does not, however, alter the course of the disease.

A right-handed 65-year-old man is being evaluated for difficulty speaking. The patient is able to obey verbal commands such as raising his hand or pointing to the window. However, he is unable to speak coherently, often having to pause midsentence. When handed a piece of paper with the printed instruction, Fold this page in half and place it on the table, the patient complied. Despite having normal motor function, he had trouble answering the examiner's questions in writing. The patient failed immediate and delayed (5-min) recall of three common words. He becomes markedly irritable over the course of the examination. Which area of the brain is most likely affected in this patient? A.Arcuate fasciculus B.Left hemisphere C.Left parietal lobe D.Left posteroinferior frontal lobe E.Left posterosuperior temporal gyrus F.Left prefrontal region

D.Left posteroinferior frontal lobe Stroke or other injury to the brain can result in various types of aphasias. This patient's presentation is consistent with Broca (expressive) aphasia, which is classically characterized by both comprehension without fluency and difficulty finding certain words. His aphasia is consistent with damage to the dominant inferior frontal gyrus, which is most often located on the left, even in patients who are left-handed.

Which of the following epidermal layers distinguishes the palmar surface from dorsal surface of the hand? A.Basale B.Corneum C.Granulosum D.Lucidum E.Spinosum

D.Lucidum The stratum lucidum is not found on the dorsal surface of the hand, and can be used to distinguish the palmar from the dorsal surface.

A 26-year-old woman comes to the clinic for follow-up after recently receiving a diagnosis of multiple sclerosis. She states that she has been managing her symptoms well since she started taking glatiramer. On neurologic examination, decreased sensation is detected in her legs bilaterally, and decreased patellar and Achilles tendon reflexes are observed. Proprioception of the great toe is diminished. Which of the following best describes the receptors responsible for proprioception? A.Action potentials B.Golgi tendon organs C.Ion channels D.Muscle spindles E.Proteolysis F.Stretch receptors

D.Muscle spindles Muscle spindles are small sensory organs enclosed in a capsule. They are found throughout the body of the muscle in parallel with extrafusal fibers. The intrafusal fibers have contractile proteins at either end with a central region where sensory dendrites and the muscle spindle afferents sit. When the muscle lengthens, the muscle spindle is stretched open, and this opens mechanically gated ion channels in the sensory dendrites leading to a receptor potential followed by action potentials in the spindle afferents. These pathways lead to the cortex where an individual then feels the toe moving in space. The activity of muscle spindles is reduced in multiple sclerosis, which results in diminished proprioception.

If this patient is not treated, which of the following histologic changes would most likely be found in the intestinal mucosa once irreparable damage has occurred? A.Cytoplasmic shrinkage B.Intact cellular membrane C.Membrane-bound apoptotic bodies D.Nuclear pyknosis E.Ribosomal disaggregation

D.Nuclear pyknosis 1. Reversible injury: Cellular and mitochondrial swelling Membrane blebbing Nuclear chromatin clumping Ribosomal detachment 2. Irreversible injury: Rupture of lysosomes and autolysis Plasma membrane damage Increased mitochondrial permeability Nuclear pyknosis (condensation) Nuclear karyorrhexis (fragmentation) Nuclear karyolysis (fading) 3. Cell deathFeatures such as an intact cell membrane, cytoplasmic shrinkage, and membrane-bound apoptotic bodies are associated with the cellular features of apoptosis, and not cellular necrosis. Ribosomal disaggregation is an early-stage histologic finding of reversible ischemic cell injury.

A frail 71-year-old female presents to the emergency department with fatigue, dizziness, and palpitations. The patient used to eat regularly, but her daughter reports that since her mother was widowed 5 months ago, she frequently skips meals now that she lives alone. When she does eat, her meals consist of small quantities of coffee, rice crackers, and jelly. A blood draw reveals the following results:RBC: 4.3 × 1012/LHgb: 10.9 g/dLMCV: 105.9 fL This patient most likely has a defect in which process? A.Glutathione synthesis B.Heme synthesis C.Isomerization of methylmalonyl-CoA D.One-carbon-unit transfers E.Synthesis of β-globin chains F.Synthesis of spectrin-actin cytoskeleton

D.One-carbon-unit transfers Megaloblastic anemia has two principal causes: vitamin B12 (cobalamin) deficiency and folate deficiency. This patient is eating very little, including only rice crackers, which are probably not fortified, so her megaloblastic anemia is most likely due to folate deficiency. Body stores of folate are minimal and can be depleted within a few months if intake is inadequate. Leafy green vegetables are a natural source of folate. In developed nations, flour is fortified with folate, which has dramatically decreased the incidence of folate deficiency, though it is still the most common vitamin deficiency in the United States.The tetrahydrofolate form of folate (folic acid) functions as an intermediate in the transfer of 1-carbon units, which is important for purine synthesis. Folate deficiency does not cause neurologic symptoms. Causes of folate deficiency include malnutrition (eg, alcoholism), malabsorption, antifolates (eg, methotrexate, trimethoprim, phenytoin), and increased requirement (eg, hemolytic anemia, pregnancy). Lab findings include increased homocysteine but normal methylmalonic acid levels, separating this from the megaloblastic anemia caused by vitamin B12 deficiency.

A 42-year-old woman presents to a physician with lethargy, cold intolerance, and a 15-lb weight gain over the past 3 months, despite concerted efforts to cut her calorie intake. On examination she is found to be an obese, well-appearing woman in no acute distress. Her neck is supple and nontender with a diffusely enlarged thyroid gland. Patellar tendon reflexes are 1+ bilaterally. Her skin has patchy areas of dryness. If her symptoms are secondary to an essential amino acid deficiency, which amino acid would be deficient? A.Arginine B.Glycine C.Histidine D.Phenylalanine E.Tryptophan

D.Phenylalanine Phenylalanine is a precursor to tyrosine, which is needed to create thyroid hormone. During thyroid hormone synthesis, iodide is first oxidized to iodine. Iodine is then added to the tyrosine residues of thyroglobulin to create monoiodotyrosine (MIT) and diiodotyrosine (DIT).

This patient presents with increased daytime drowsiness. Despite an early bedtime and a consistent sleep schedule, he does not feel rested and falls asleep at work. He sometimes wakes up with a headache, and his wife reports being woken up by his snoring and hearing him grunt as if he is out of breath. With a BMI of 36, the patient is obese. He most likely has obstructive sleep apnea. Which of the following is the next best step for this patient? A.Bilevel positive airway pressure therapy B.Continuous positive airway pressure therapy C.Hypoglossal nerve stimulator therapy D.Polysomnography E.Uvulopalatopharyngoplasty F.Zolpidem therapy

D.Polysomnography Polysomnography is the first step in diagnosing obstructive sleep apnea. Polysomnography records brain waves, heart rate, oxygen level, and eye movements during sleep. This helps to track the amount of time someone is asleep and time spent in each phase of sleep. Also, it shows apnea by showing the drop in oxygen level during different phases of sleep. A continuous positive airway pressure device (CPAP) is a first-line treatment in patients with sleep apnea. It may be beneficial to this patient, but would not be used until after he is diagnosed. A bilevel positive airway pressure device (BiPAP).

A 20-year-old Caucasian man comes to the physician because of episodes of mucoid, bloody diarrhea for the past 9 months. In addition, he has been experiencing progressive fatigue and pruritus. Medical history is significant for an episode of severe knee pain 3 years ago. Physical examination reveals leg ulcerations and icteric sclerae. A colonoscopy is subsequently performed, which shows diffuse hemorrhagic ulcers with rectal involvement, as well as multiple areas of granulation tissue.Laboratory studies reveal the following: Aspartate aminotransferase: 19 U/LAlanine aminotransferase: 20 U/LAlkaline phosphatase: 410 U/LTotal bilirubin: 3.5 mg/dLDirect bilirubin: 3.1 mg/dL Which of the following disorders is associated with this patient's current condition? A.Chronic malabsorption B.Noncaseating granulomas C.Primary biliary cirrhosis D.Primary sclerosing cholangitis E.Small bowel stricture

D.Primary sclerosing cholangitis Ulcerative colitis (UC) is associated with primary sclerosing cholangitis (PSC). This patient presents with episodes of mucoid and bloody diarrhea, increased alkaline phosphatase activity, and a history of knee pain. Together with the abnormal colonoscopy findings (hemorrhagic ulcers and granulation tissue, also known as pseudopolyps), this patient most likely has UC. PSC which manifests as jaundice and increased alkaline phosphatase activity, as seen in this patient. Some individuals (2.5% to 7.5%) with UC also have PSC. A positive p-ANCA may also be found. Because this patient's condition is limited to involvement of the rectum and colon in the majority of cases, chronic malabsorption is not the most likely complication. Malabsorption is usually seen in patients with Crohn disease as a result of frequent involvement of the small intestine, which is the primary location for absorption.

A 62-year-old man is brought to the clinic by his wife, who states, "I think my husband has Alzheimer's." She explains that his symptoms started 3 months ago. Her husband has become absent minded and over time has forgotten "who or where he was." According to his wife, he began to hallucinate about 1 month ago and can no longer walk because "his movements have become jerky and uncoordinated." The patient has a past medical history of a corneal transplant. On physical examination, the physician notices bilateral hyperreflexia and a strong startle response, with a jerking motion of the arms. Which of the following pathologic agents is the key identifying feature of this patient's disease as revealed on biopsy of the brain? A.β-Amyloid plaques B.Bacteria C.Eosinophilic, intracytoplasmic neuronal inclusions D.Prion E.Virus

D.Prion the rapid progression of his symptoms, previous history of a corneal transplant, paired with his strong startle response and jerking movements of his arms (myoclonus), are most suggestive of a prion disease.

A 20-year-old woman presents to her college health clinic with a 1-day history of headache and stiff neck. On physical examination, she shows signs of nuchal rigidity. Nasopharyngeal swab resulted in growth of gram-negative diplococci on Thayer-Martin agar. Through which mechanism does the most likely causal organism achieve nasopharyngeal colonization? A.Downstream effects of endotoxin release B.Impairment of complement activation C.Impairment of opsonization and phagocytosis D.Production of Fc fragments E.Promotion of intracellular survival F.Triggering an autoimmune response

D.Production of Fc fragments N meningitidis has several virulence factors that help it achieve colonization and infection. This includes an IgA protease, capsule, endotoxin, and pili. Specifically, the IgA protease and pili achieve nasopharyngeal colonization by cleaving IgA and by facilitating bacterial adherence to the nasopharynx, respectively. Although Neisseria species has a capsule, this virulence factor aids in evading opsonization and phagocytosis; it has no role in colonizing the nasopharynx.

methicillin-resistant Staphylococcus aureus (MRSA). Which of the following mechanisms is most likely responsible for the lack of response to antibiotics? A.Modification of a ribosomal binding site B.Mutated DNA gyrase C.Production of ß-lactamase D.Production of a modified pencillin-binding protein E.Reduced permeability of bacterial cells

D.Production of a modified pencillin-binding protein MRSA produces a modified PBP (PBP2) that has a much lower affinity for methicillin compared with the unaltered PBP. When an infection with MRSA is suspected, the spectrum of antibiotic coverage should be broadened. Vancomycin and daptomycin are first-line agents used to treat cellulitis caused by MRSA.

Which of the following characteristics of the hormone explains this patient's rapid excretion? A.Hepatic removal of drug B.Highly water soluble C.Increased renal excretion D.Rapid glucuronidation E.Readily redistributed

D.Rapid glucuronidation lucuronidation renders molecules more soluble for excretion. This is used for hormones for facilitated transport through the body. This is inconsistent with a highly lipid soluble molecule.

A 56-year-old man visits a local clinic while on vacation because of increasing fatigue, insomnia, and intermittent feelings of unease and unhappiness. The patient reveals that he was diagnosed with major depressive disorder 6 months earlier. Since that time, he had been taking escitalopram, the medication prescribed by his doctor. Although his mood had remained consistently better, he had experienced erectile dysfunction as a side effect. Wishing to enjoy sexual intercourse with his wife on this trip, the patient abruptly stopped taking his medication 1 week before their vacation started. Based on this patient's current symptoms, which of the following is the best next step in management? A.Administer cyproheptadine B.Administer dantrolene C.Reassurance D.Resume escitalopram therapy E.Start bupropion therapy F.Start clozapine therapy

D.Resume escitalopram therapy Based on his report of abrupt cessation of his medication, which is a selective serotonin reuptake inhibitor (SSRI), he is likely experiencing antidepressant discontinuation syndrome (ADS). Symptoms include mood disturbances (dysphoria, anxiety, or agitation), flu-like symptoms (fatigue, nausea, headaches, muscle aches, and sweating), and sleep disturbances (insomnia and nightmares). Treatment involves resuming the discontinued medication as previously directed. Starting bupropion therapy would be appropriate after the escitalopram tapering period has ended. Bupropion treats major depressive disorder and rarely produces sexual side effects.

A 28-year-old man who is HIV positive comes to the physician because of worsening eyesight. He states that his loss of vision began in the center of his visual field and was associated with "a lot of floaters." The symptoms progressed rapidly, and now he can barely see out of his right eye. His last CD4 count was 43 cells/mm3. Fundoscopic examination reveals cotton-wool exudates, necrotizingretinitis, and perivascular hemorrhages. Which of the following complications is most likely to also be seen on fundoscopic examination? A.Macular cherry-red spot B.Neovascularization C.Papilledema D.Retinal detachment

D.Retinal detachment Retinal detachment, in which the neurosensory layer of the retina separates from the outermost pigmented epithelium, can occur as a later finding of CMV retinitis due to the accumulated damage to the retina. Retinal detachment typically manifests with sudden onset of floaters and/or photopsias with monocular vision loss. CMV retinitis is treated with the antivirals ganciclovir and foscarnet.

A 64-year-old man with a history of hypertension, coronary artery disease, and type 2 diabetes presents to his physician because he "has trouble seeing." Visual field testing reveals a defect in the left half of the visual field for both eyes, with sparing of central acuity. Which of the following is the most likely cause of the patient's symptoms? A.Infarction of the lower division of the middle cerebral artery B.Infarction of the upper division of the middle cerebral artery C.Left retinal artery occlusion D.Right posterior cerebral artery infarction with sparing of the occipital pole

D.Right posterior cerebral artery infarction with sparing of the occipital pole

A 6-year-old child is brought to the pediatrician because of right leg pain and a new rash. She also reports a mild nonproductive cough and shortness of breath. She was hospitalized with a spontaneous pneumothorax6 months ago. Temperature is 98.6 °F (37° C), blood pressure of 128/83 mm Hg, pulse is 91/min, and respiratory rate is 20/min. Physical examination reveals tenderness of a raised, soft bump on her right outer thigh and several similar painful lesions above her eyebrows. Skin examination reveals red bumps ranging from 5-10 mm on her back and chest. Abdominal examination reveals an enlarged liver and lymphadenopathy in superficial inguinal lymph nodes. A chest CT scan shows myriad nodules and cysts with interstitial thickening in the lungs bilaterally. A transbronchial biopsy specimen from the patient's lung is most likely to stain positive for which of the following? A.CD20 B.CD30 C.CD5 D.S-100 E.TdT

D.S-100 Cumulatively, these findings are suggestive of Langerhans cell histiocytosis (formerly histiocytosis X), a rare condition characterized by the abnormal proliferation of cells of mononuclear phagocytic origin, which are called histiocytes. These cells are related in lineage to Langerhans cells and will stain positive for S-100 and CD1a (OKT-6).

A microbiology laboratory is investigating the possibility of using enzymes as a novel class of antibacterial agents.These enzymes work by destabilizing the bacterial cell wall. These enzymes cannot traverse double-layer lipid membranes. Which of the following organisms would be affected by this type of drug? A.Brucella abortis B.Chlamydia trachomatis C.Escherichia coli D.Staphylococcus aureus E.Ureaplasma urealyticum

D.Staphylococcus aureus The enzymes that are being investigated can bind surface peptidoglycans directly from the extracellular space; they are unable to traverse double-layer lipid membranes. Peptidoglycan is a component of both gram-positive and gram-negative cell walls but is distinctly located in each. Gram-positive bacteria have a thick peptidoglycan cell wall as the outermost structure. Gram-negative bacteria also have a peptidoglycan layer, but it is surrounded by an outer lipid bilayer. Thus these enzymes will only affect gram-positive bacteria. Of the answer choices, the only gram-positive bacterium is Staphylococcus aureus.

An 8-year-old Caucasian boy is brought to the clinic by his mother. She states that, for the past 3 weeks, the boy has been extremely tired all the time and that his forehead is hot to the touch. His temperature is 39°C (102.2°F), pulse is 64/min, respirations are 12/min, and blood pressure is 100/60 mm Hg. On physical examination, he has a scaly, pruritic, papular rash on his scalp, which his mother states has been there for the past 3 weeks. His upper arms are swollen and tender to palpation. Electron microscopy reveals multiple granules that are striated and rod-like with single-sided outpouchings. Which of the following best describes the function of the cells seen on the biopsy specimen? A.To act as a precursor cell for granulocytes B.To costimulate B lymphocytes C.To initiate a rapid immune response on re-exposure to an antigen D.To phagocytose cell debris and pathogens E.To secrete antibodies

D.To phagocytose cell debris and pathogens This young boy's symptoms of fever, rash, and bone swelling and the presence of pathognomonic features detected by electron microscopy are most suggestive of a diagnosis of Langerhans cell histiocytosis (previously known as histiocytosis X, Letterer-Siwe disease, Hand-Schüller-Christian disease, and diffuse reticuloendotheliosis). Langerhans cell histiocytosis is a rare disease characterized by the infiltration of organs by a monoclonal population of myeloid progenitor-derived cells, which resemble the Langerhans cells of the dermis and mucosa. It is more common in children from birth to 15 years, and its prevalence appears to be higher among Caucasians. Its frequency is greater in males than in females, with a male-to-female ratio of 2:1.

what causes fanconi anemia?

DNA repair defect, causing bone marrow failure

which HLA is shared by hashimotos and pernicious anemia?

DR5

In an experimental study, 10 patients with no liver or renal dysfunction are administered drug Y, a novel immunosuppressant. Preliminary pharmacokinetic investigation has found that drug Y has an oral bioavailability of 50% and a clearance rate of 10 L/hr. The researchers intend for drug Y to have a concentration of 5 mg/L in the subjects' plasma. Which of the following is the most appropriate intravenous infusion rate needed to achieve this plasma concentration? A.100 mg/h B.25 mg/h C.250 mg/h D.5 mg/h E.50 mg/h

E.50 mg/h (Target plasma concentration [CP] × Drug clearance [CL] × Dosage interval [τ]) / Bioavailability [F]. In this case, the calculation would be (5 mg/L) × (10 L/h) / 1 = 50 mg/h. The dosage interval is not included in the calculation because infusion of drug Y is assumed to be constant. Also, intravenous administration of a drug assumes a bioavailability of 1, so the oral bioavailability given in the vignette is not relevant for the question. If the question asked for the maintenance dose for the oral formulation of drug Y, 100 mg/h would be the correct answer since the calculation would be (5 mg/L) × (10 L/h) / 0.5 = 100 mg/hr.

A 56-year-old man with hypertension, hyperlipidemia, and type 2 diabetes mellitus comes to his primary care physician with complaints of chestpain. He reports that he intermittently has chest pain when walking up several flights of stairs or helping his wife in the garden. The pain goes away when he takes a break from such activities. What is the minimum suspected percentage of arterial stenosis in this patient? A.10% B.100% C.45% D.60% E.75% F.90%

E.75% Luminal narrowing >70% causes temporary cardiac ischemia without cell death, known as stable angina, in which the pain is relieved with rest (as seen in this patient)

A 55-year-old man comes to his physician because of a lump in his throat that remains throughout the day. On palpation of the neck, no enlarged lymph nodes or other masses are felt. Upper endoscopy reveals no changes to the epithelium of the upper, lower or transition zones. Imaging with barium swallow shows no abnormalities. When the physician asks if the patient has changed his diet, the patient mentions that he has been eating less because he recently got divorced and his ex-wife used to cook all his meals. The patient says he has not been sleeping well since the divorce was finalized 1 month ago, and just does not have the energy he used to. He denies any pain in his throat or chest while he sleeps. He works as a bank teller and admits he cannot seem to focus at work because he is unable to stop thinking of ways he could have saved his marriage. Which of the following is the most appropriate diagnosis? A.Achalasia B.CREST syndrome C.Esophagitis D.GERD E.Globus sensation F.Zenker diverticulum

E.Globus sensation Globus sensation is characterized by the sensation of a lump, foreign body, retained food bolus, or tightness in the throat despite an absence of any underlying structural changes such as GERD, mucosal protrusion, or an esophageal motility disorder. Etiology remains unclear but some studies have found a link between psychological distress and globus sensation. For the purpose of the USMLE, a psychological stressor such as work-related stress, depression over a divorce, or health anxiety will always be mentioned to help narrow down the diagnosis.

A 45-year-old white man comes to the physician because of weakness, lethargy, joint pains, and decreased libido during the past few months. The patient denies any past medical conditions. The patient's father died from a "long-standing liver problem." On physical examination, the heart and lungs are normal. The abdominal examination reveals the liver edge palpable two finger-widths below the costochondral angle. Laboratory studies are obtained. The patient is referred for a liver biopsy; findings show blue hemosiderin deposits on a Prussian blue stain. Which laboratory change is most likely to occur in this patient's disease? A.Decreased serum ceruloplasmin B.Decreased serum ferritin C.Increased iron-binding capacity D.Increased serum copper E.Increased transferrin saturation

E.Increased transferrin saturation Hereditary hemochromatosis is caused by a defect in a gene that encodes the protein HFE. This gene is near the MHC locus on chromosome 6. As a result of this defect, there is an increase in the efficiency of dietary iron absorption. This, coupled with the inability to excrete iron, leads to an increase in hepatic iron storage. Increased iron storage causes the liver to secrete more ferritin. Serum transferrin becomes increasingly saturated with iron. A complete iron study will show increased ferritin, increased serum iron, decreased total iron binding capacity and increased transferrin saturation.

A 45-year-old man presents to his primary care provider for his yearly checkup. Basic lab tests, including a lipid profile, are performed before the visit. They show a low-density lipoprotein (LDL) of 174 mg/dL, high-density lipoprotein (HDL) of 46 mg/dL, and a triglyceride level of 202 mg/dL. To reduce the patient's risks for morbidity and death, his physician recommends lifestyle changes and initiates therapy with a lipid-lowering medication. The doctor warns the patient that this medication may be associated with muscle pain. By what mechanism does this intervention affect cholesterol metabolism? A.Decreases the apparent dissociation constant of hepatic LDL cholesterol receptors B.Decreases the maximum velocity of HMG-CoA reductase C.Decreases the maximum velocity of hepatic LDL cholesterol receptors D.Directly increases apparent dissociation constant for LDL receptors E.Increases the apparent Michaelis-Menten constant of HMG-CoA reductase

E.Increases the apparent Michaelis-Menten constant of HMG-CoA reductase Statins are the gold standard for treatment of hyperlipidemia when a patient can tolerate their side effects. They have been proven to decrease morbidity and mortality rates associated with atherosclerosis. Because statins bind to the same binding site as the substrates of the enzyme, statins increase HMG-CoA reductase's apparent Km. Well-known adverse effects of statin use include myalgia, myositis, and rhabdomyolysis. Statins have no effect on the maximum velocity (Vmax) of the reaction because they reversibly bind to the same site as HMG-CoA and can be outcompeted in the presence of very high concentrations of HMG-CoA.

A 63-year-old woman comes to the office because of fatigue that has been worsening over the past 2 weeks. At first, she thought it was "part of the aging process," but then she started noticing "spots" and bruising on her skin and lips. She denies bone pain. Her temperature is 37.4°C (99.3°F), blood pressure is 135/80 mm Hg, and respirations are 16/min. The patient has a pale appearance. Her breath sounds are clear bilaterally, and heart sounds are normal. No enlarged lymph nodes are present. The abdomen is nontender, and there is no hepatosplenomegaly. Nonblanchable petechiae are present on the lower limbs. A complete blood count reveals the following:Hemoglobin: 8 g/dLPlatelets: 67,000/mm3Leukocytes: 3000/mm3 Myeloblasts: 30% Promyelocytes: 10% Myelocytes: <5% Metamyelocytes: <5% Mature myelocytes: 50% A peripheral blood smear shows Auer rods. Which of the following is most likely to be associated with this patient's condition? A.Blastic bone lesions on radiography of the lower extremities B.Decreased plasma von Willebrand factor activity C.Greater than 20% lymphoblasts in bone marrow D.Increased leukocyte alkaline phosphatase E.Microangiopathic hemolytic anemia

E.Microangiopathic hemolytic anemia The cytoplasmic inclusions known as Auer rods are myeloperoxidase positive. Spillage of Auer rod contents in a patient with APL may inappropriately activate the coagulation cascade throughout the body, causing platelet-fibrin microthrombi to form in both arterial and venous circulation; this is a known as disseminated intravascular coagulation. Furthermore, the microthrombi obstruct the lumina and shear erythrocytes as they pass through, a phenomenon known as microangiopathic hemolytic anemia. Increased leukocyte alkaline phosphatase (LAP) is used to differentiate a physiologic acute inflammatory response to infection from acute and chronic myelogenous leukemias. This patient with APL would have decreased LAP. Although the finding of more than 20% myeloblasts and the presence of multiple Auer rods are characteristic of APL, the finding of more than 20% lymphoblasts in bone marrow would be characteristic of acute lymphocytic leukemia. Decreased plasma von Willebrand factor activity would be found in a patient with von Willebrand disease, a mixed coagulation and platelet disorder. Although it would explain this patient's signs of thrombocytopenia (eg, petechiae on lower limbs), a decrease in plasma von Willebrand factor activity would not explain this patient's systemic illness and decreased leukocyte count. The finding of blastic bone lesions on radiography of the lower extremities is associated with significant bone pain and can be seen in patients with certain cancers that metastasize to bone, including prostate cancer and breast cancer. The infiltration of normal, functional bone marrow by cancerous cells results in cytopenias due to loss of hematopoiesis in the bone marrow. Although this patient has cytopenias, she denies bone pain, which should prompt exploration of alternative diagnoses. Furthermore, the presence of blastic bone lesions would not explain the presence of Auer rods, which are characteristic of hematologic malignancy of myeloid origin.

Which of the following will likely be found on a Goodpasture patient's pulmonary function tests? A.Decreased DLCO B.Increased functional residual capacity C.Increased residual volume D.Increased total lung capacity E.Normal FEV1:FVC

E.Normal FEV1:FVC Pulmonary function tests would show a restrictive lung disease pattern with a decreased total lung capacity, decreased forced expiratory volume in 1 second (FEV1), decreased forced vital capacity (FVC), and a normal FEV1:FVC. Treatment involves plasmapheresis, prednisone, and cyclophosphamide. In the case of Goodpasture syndrome, the excess hemoglobin present due to alveolar hemorrhage binds more carbon monoxide than could normally be bound by the capillaries in restrictive diseased lungs, giving a falsely elevated DLCO.

This patient has an alcohol abuse disorder, and is recently hospitalized for severe acute alcoholic pancreatitis. His presentation initially improved with supportive care, but he now has acute-onset hypoxemic respiratory failure. This is indicated by his bilateral crackles on auscultation and noted as bilateral pulmonary infiltrates on chest x-ray. He has no prior nor current signs of cardiac disease (normal jugular venous pressure and no distention, no peripheral edema, and normal BNP levels). Presentation is concerning for acute respiratory distress syndrome (ARDS). Which of the following is the most likely additional finding in this patient? A.Decreased left ventricular ejection fraction B.Increased PaO2/FiO2 ratio C.Increased diffusion capacity for carbon monoxide D.Normal lung compliance E.Normal pulmonary capillary wedge pressure F.Reduced peak pulmonary pressures produced by ventilator

E.Normal pulmonary capillary wedge pressure Because PCWP serves as a measure of left ventricular end diastolic pressure, it is more likely to be increased in cases of cardiogenic pulmonary edema secondary to left ventricular failure or volume overload rather than acute pulmonary injury due to ARDS. He has no prior nor current signs of cardiac disease (normal jugular venous pressure and no distention, no peripheral edema, and normal BNP levels). Why the incorrect answers are wrong: ARDS is associated with a severely decreased PaO2/FiO2 caused by impaired gas exchange in the injured lung. Decreased left ventricular ejection fraction can lead to bilateral infiltrates on chest x-ray study secondary to acute cardiogenic pulmonary edema. However, this patient, who presents within 24 hours of admission for severe acute pancreatitis and has no jugular vein distension or peripheral edema is more likely to have ARDS. Diffusing capacity of the lungs for carbon dioxide (DLCO) would likely be reduced in ARDS secondary to thickened alveolar membranes (fluid, hyaline membrane lining). Lung compliance is usually reduced in patients with ARDS secondary to interstitial edema, hyaline membrane formation, collapse of fluid-filled alveoli, and atelectasis. Because lung compliance is usually reduced in patients with ARDS, peak pulmonary pressures are increased on the ventilator. Peak pulmonary pressures correlate inversely with lung compliance and correlate directly with airway resistance.

A 4-day-old neonate to the emergency department by her mother because of failure to thrive. The mother says the child has had vomiting and diarrhea for the last 2 days and is very irritable. The infant was born at term via normal spontaneous vaginal birth at home; there were no complications during delivery. The mother has a history of poorly controlled Graves disease and non-compliance with her anti-thyroid medications. The infant's temperature is 36.7° C (98.2° F), blood pressure 116/72 mm Hg, pulse is 175/min, and respiratory rate is 53/min. On physical examination, the neonate is noted to have a prominent forehead with a head circumference less than the 5th percentile. A large mass on her anterior neckis palpated. Laboratory studies show a TSH of 0.2 μU/mL and T4 of 30 μg/dL. Which of the following is the most likely cause of this newborn's symptoms? A.Congenital hypothyroidism B.Congenital viral infection C.Inadequate thymus development D.Narcotic withdrawal E.Placental transmission of TSI antibodies F.TSH-secreting pituitary tumor

E.Placental transmission of TSI antibodies Neonates born to mothers with Graves disease who are non-compliant with their anti-thyroid medications are at risk of neonatal hyperthyroidism due to placental transmission of thyroid-stimulating immunoglobulin (TSI) antibodies. These are IgG antibodies that stimulate the TSH receptors on the thyroid gland to increase production of thyroxine. The TSH receptors on a fetus' thyroid gland become responsive to these antibodies at about 20 weeks' gestation; however, approximately 50% of TSI antibodies are transferred in the third trimester. This can lead to intrauterine growth restriction, fetal tachycardia, spontaneous abortion, and fetal death.

An 18-year-old man comes to the clinic for a routine annual examination before starting university. He was previously diagnosed with homocystinuria. He has an intellectual disability and history of lens dislocation. He takes no medications. He is 154 cm (6'1) and 63.5 kg (140 lb). Vital signs are within normal limits. The patient has long limbs and digits. Physical examination shows hyperflexibility of his extremities. The chest wall caves inward. Which of the following genetic principles best explains the findings associated with this patient's disorder? A.Anticipation B.Codominance C.Incomplete penetrance D.Locus heterogeneity E.Pleiotropy F.Variable expressivity

E.Pleiotropy Pleiotropy occurs when mutation of a single gene produces multiple phenotypic effects due to affecting multiple organ systems. Therefore, a patient will present with multiple traits associated with one disorder.

The immunologic mechanism underlying Lupus renal disease is most similar to the mechanism underlying which of the following conditions? A.Anaphylaxis B.Contact dermatitis C.Goodpasture syndrome D.Graves disease E.Polyarteritis nodosa (PAN)

E.Polyarteritis nodosa (PAN) Deposition of antigen-antibody complexes is the immunologic mechanism of a type III hypersensitivity reaction, detailed in the table. Polyarteritis nodosa(PAN) is caused by a type III hypersensitivity reaction. PAN is a medium-vessel vasculitis most often found in middle-aged men. It is also associated with Hepatitis B seropositive patients. Typically the renal and visceral vessels are affected which can lead to hypertension, neurologic dysfunction.and renal damage. Biopsy of the vessels affected shows transmural inflammation with fibroid necrosis.

A pediatric geneticist is working on a case series about a rare pulmonary disorder. From her literature review, she discovers eight case reports of children who had the condition. In addition, two of her patients with the condition recently died. The lifespans of her patients were 26 and 36 months and those of patients reported in the literature were 9, 10, 11, 11, 11, 12, 12, and 13 months. She is astounded that her two patients (with lifespans of 26 and 36 months) had far outlived the other eight patients. What statistical distribution will be reflected if the lifespans of the patients described in the literature and the lifespans of the geneticist's patients are plotted on a histogram? A.Bimodal distribution B.Chi-square distribution C.Gaussian distribution D.Negatively skewed distribution E.Positively skewed distribution

E.Positively skewed distribution The study described in this question provides data on a small sample comprising 10 patients. According to the data, eight of the patients had lifespans between 9 and 13 months, but two had longer lifespans of 26 and 36 months, respectively. These two patients had far outlived the others and are clearly outliers. Three units are used as "averages," and they are the mean, median, and mode. The different units are affected by outliers to a different extent, and this can determine whether the data are "skewed" and in which direction. Skewness is the asymmetry of the distribution of variables around the mean. The presence of outliers can cause this asymmetry. Outliers will always have a stronger impact on the mean than on the median. In this example, we see that there are a couple of numbers on the positive end that are much higher than the rest of the data set. What this does is lead to a sample mean that is "positively skewed" relative to the median. Plotted data are considered positively skewed when the mean is higher than the median and are considered negatively skewed when the mean is lower than the median.

The patient is started on an older chemotherapy regimen that includes a synthetic corticosteroid that requires bioactivation. Which of the following corticosteroids was most likely prescribed? A.Betamethasone B.Budesonide C.Dexamethasone D.Hydrocortisone E.Prednisone

E.Prednisone Prednisone is a prodrug of prednisolone. Prednisone is an inactive compound that must be reduced to its active form, prednisolone, which has anti-inflammatory effects. The enzyme that catalyzes this reaction is 11-β-hydroxysteroid dehydrogenase.

A 20-year-old college student who is 6 weeks' pregnant goes to her local clinic for information on pregnancy termination. She is fearful of an operation and asks about medical abortion. Clinic staff explain that mifepristone can be used early in pregnancy for termination, with an efficacy rate of around 85%. When a second drug is administered 24-48 hours after taking mifepristone, efficacy rises to almost 100%. The second medication facilitates the passage of the products of conception; adverse effects of the second drug include abdominal cramps and bleeding. Which of the following drugs is coadministered with mifepristone? A.A cyclooxygenase inhibitor B.A partial estrogen receptor agonist C.A progesterone receptor agonist D.An oxytocin receptor agonist E.Prostaglandin E1 agonist

E.Prostaglandin E1 agonist A few days after mifepristone is administered, misoprostol, a prostaglandin E agonist, is taken to stimulate uterine contraction and thus expel the embryotic contents from the uterus. Combining misoprostol with mifepristone increases the abortifacient effects of the regimen.

A 20-year-old comes to the physician because of abdominal cramping and pain that radiates to her lower back and inner thighs during menses. The pain has occurred for the last 6 menstrual cycles. Menses are regular and occur every 28 days. Her last menstrual period was 7 days ago. She denies heavy periods, pain during intercourse, or vaginal discharge. She is sexually active with one partner and uses condoms inconsistently. On physical examination, temperature is 36.7° C (98.1° F), blood pressure is 130/84 mm Hg, and respiratory rate is 18/min. Pelvic examination is unremarkable. Laboratory studies are negative for an elevated beta-hCG. Which of the following best describes the pathogenesis of this patient's likely condition? A.Benign uterine tumors composed of myometrium B.Endometrial glands and stroma within the myometrium C.Endometrial infection with plasma cell predominance D.Implantation of an embryo outside the uterine cavity E.Prostaglandins causing uterine contraction

E.Prostaglandins causing uterine contraction This patient has primary dysmenorrhea defined as crampy, lower abdominal pain during menses without a clear explanation of symptoms. Thus, it is a diagnosis of exclusion. Primary dysmenorrhea has a high prevalence, with 50% to 90% of reproductive-age women experiencing painful menses making it the most common gynecologic condition. The most important risk factor is age under 30 years, as prevalence declines with increased age. The pathogenesis of this condition is due to elevated prostaglandin release during menses which causes vasoconstriction, ischemia, and high-frequency myometrial contraction. First-line therapy is estrogen-progestin contraceptives (eg, OCP, transdermal patch, vaginal ring). NSAIDs are also used due to their inhibition of prostaglandins. Endometrial glands and stroma within the myometrium describes adenomyosis. Implantation of an embryo outside the uterine cavity describes ectopic pregnancy. Endometrial infection with plasma cell predominance describes chronic endometritis. Benign tumors composed of myometrium describes uterine fibroids (leiomyomas).

A 45-year-old African-American man presents to his primary care provider with left-sided flank pain of three weeks' duration. Physical examination demonstrates a palpable mass on the left flank in the area of his pain. The physician orders a urinalysis.The results are shown below: Color: PinkpH: 7.0Specific Gravity: 1.015Protein: noneGlucose: noneKetones: noneBlood/HPF: 15 (normal <4)WBC/HPF: noneNitrites: negativeLeukocyte Esterase: negativeA CT scan of the chest reveals multiple foci in the lungs. Surgery is performed to remove the left flank mass. Follow-up imaging done several weeks later demonstrates shrinking of the lung foci. Treatment with which of the following immunotherapeutic agents would most likely benefit this patient? A.An anti-CTLA4 antibody B.Interferon α-antagonist C.Interferon-γ D.Monoclonal antibody against CD20 E.Recombinant interleukin-2

E.Recombinant interleukin-2 Management of metastatic renal cell carcinoma may include the use of chemotherapy drugs following surgical tumor debulking. Recombinant interleukin-2 (IL-2), also known as aldesleukin is a T lymphocyte growth factor. It stimulates T lymphocytes proliferation and increases the production of natural killer (NK) cells. NK cells help target cancerous cells for immunological attack. Recombinant IL-2 activates the innate immune-mediated antitumor activity to help destroy cells affected by the metastatic RCC cells. Patients with RCC have benefitted from recombinant IL-2 treatments.

A 40-year-old woman who was recently diagnosed with metastatic melanoma comes to her physician to discuss feelings of hopelessness. She states that she met with a surgical oncologist about treatment, but thinks it probably will not help her get better. The patient is worried that she will die before her next birthday. The physician asks, "Why do you believe the treatment will not work?" Which of the following interview techniques is the doctor using? A.Confrontation B.Countertransference C.Facilitation D.Recapitulation E.Reflection F.Transference G.Validation

E.Reflection Her physician responded by asking why she believes these things. He is employing reflection, an interview technique that requires active listening and includes synthesizing the emotions and facts offered by the patient. It is best practice to ask a patient to explain the thoughts and feelings behind his or her initial statements or reactions. Validation, or providing a positive response, can be useful in some situations. This patient, who is depressed, would benefit from a more empathetic response that helps explore her reaction and potentially alleviate her distress to some degree. Confrontation is an interview technique in which the patient is made to face his/her attitudes and biases. This patient is unlikely to benefit from this aggressive form of communication. Recapitulation involves repeating to the patient what they are going through. This patient is more likely to benefit from an approach that asks her to consider why she feels the way she does. Transference and countertransference are not interview techniques. Respectively, in this scenario, they would involve the transfer of feelings or anxieties from the patient to the physician, or from the physician to the patient. These phenomena would not be helpful to the patient described in this scenario. Facilitation, or encouraging the patient to share his or her experience spontaneously, relies on the physician allowing the patient to speak freely without guiding his or her response. The physician in this scenario is trying to elicit a specific response.

A 76-year-old man presents to his primary care physician for his annual checkup. The patient has no major signs or symptoms of illness. Vital signs are: temperature, 37.9°F (98.5°F); blood pressure, 130/80 mm Hg; heart rate, 85/min; and respiratory rate, 15/min. Results of physical examination are within normal limits. The physician notes that the patient is in good health for his age. Compared with a young adult, for which of the following does this patient have an increased risk? A.Decreased percentage of body fat B.Increased frequency of loose stools C.Increased rapid-eye-movement sleep D.Low intelligence E.Suicide

E.Suicide Other conditions seen in aging individuals include: A decline in immune function, leading to increased risk of infection Presbyopia Loss of high-frequency hearing, taste, and olfaction—all of which can lead to nutritional deficiencies Atrophy of the skin, decreased elasticity of the skin, declining muscle mass, and loss of bone mass Urinary incontinence, urinary tract infections, and erectile dysfunction Reflux esophagitis and constipation With advancing age, individuals have an increased percentage of body fat and decreased total muscle mass. Intelligence is not affected by age. Chronic constipation, rather than increased frequency of loose stools, is expected as individuals age. Decreased rapid-eye-movement sleep is expected as individuals age.

An 84-year-old man presents to his community health clinic, saying that over the past year he's had difficulty walking and "holding" his urine." He also feels that he has become more forgetful. His daughter, who accompanies him, says that his family and friends had assumed the memory and gait difficulties were just "his age catching up with him," but the incontinence caused enough concern that they decided to seek medical advice. Physical examination reveals no visual field changes or tremors, but the patient has an unsteady gait and shuffling feet. A noncontrast CT is shown. TThe primary defect in this patient's disorder occurs in a structure that drains into which of the following venous sinuses? A.Cavernous sinus B.Confluence of sinuses C.Sphenoid sinus D.Straight sinus E.Superior sagittal sinus F.Transverse sinus

E.Superior sagittal sinus This patient has NPH caused by inadequate resorption of CSF by "clogged" arachnoid granulations, which drain into the superior sagittal sinus. The superior sagittal sinus is a collection of venous blood where arachnoid granulations drain CSF from the subarachnoid space into the bloodstream.Defects to the other listed sinuses produce different effects: Cavernous sinus damage produces visual deficits and paralysis of cranial nerves III, IV, V1, V2, and VI. Fluid buildup in the sphenoid sinus leads to symptoms similar to those from cavernous sinus damage. The straight sinus, transverse sinus, and confluence of sinuses are rarely impinged and would feature excessive symptoms of increased intracranial pressure.

An 8-year-old boy is brought to the hospital because of weakness and abdominal pain. A careful history reveals that he had a skin infection 2 weeks ago. Lately his urine has been dark brown, and he has not urinated during the past 8 hours. His vital signs are within normal limits except for elevated blood pressure. On examination, there is periorbital swelling under his eyes. Laboratory tests show:Hemoglobin: 13.0 g/dLHematocrit: 40%WBC count: 7,000/mm³Blood urea nitrogen: 18 mg/dLCreatinine: 1.5 mg/dLAntistreptolysin O titer: 184 UUrinalysis revealed +3 protein, dysmorphic erythrocytes, and red blood cell casts Which of the following is the most appropriate treatment? A.Corticosteroids B.Doxycycline C.Hydrochlorothiazide D.Penicillin V E.Supportive therapy

E.Supportive therapy The clinical presentation of acute poststreptococcal glomerulonephritis (PSGN) includes (1) acute-onset hypertension; (2) dark, "tea-colored" urine; (3) edema (periorbital, peripheral, or pulmonary); and (4) fatigue. PSGN gradually resolves within 1-2 weeks, with hematuria disappearing within 6 months. Because PSGN is mediated by an autoimmune process, antibiotics have little utility, and most patients experience a complete recovery with supportive therapy alone.

This patient's enlarging solitary thyroid nodule (discrete and firm neck mass), dysphagia, hoarseness of voice, increased serum calcitonin levels, and FNAC findings are concerning for a diagnosis of medullary thyroid carcinoma. This patient is at greatest risk for which of the following post-surgical complications? A.Drooping of the right upper eyelid and constriction of the right pupil B.Enlarging hands and feet C.Milky drainage from incision site D.Tense and immobile cervical swelling E.Tingling of the lips

E.Tingling of the lips This pt may have MEN 2A, which includees medullary thyroid cancer (calcitonin = low Ca) and parathyroid hyperplasia (incr Ca). The latter overrides the former --> hyper Ca. The two most common complications associated with total thyroidectomy are hypoparathyroidism and recurrent laryngeal nerve injury. Tingling of the lips in this patient can occur due to hypocalcemia, resulting from hypoparathyroidism if one or more of the parathyroid glands are accidentally removed during total thyroidectomy.

A 25-year-old man comes to the clinic because of a 2-week history of a runny nose and sinus congestion, which has now progressed to include a severe cough. The patient is concerned because he is a professional flute player and his symptoms are affecting his career. He does not appear to be in respiratory distress. Temperature is 99.7° F (37.6° C), blood pressure 110/72 mm Hg, pulse is 88/min, respiratory rate is 16/min, and SpO2 is 98% on room air. On auscultation of his lungs, the patient begins to cough and subsequently vomits. Cultures of the throat and nasal swabs grow gram-negative coccobacilli on Bordet-Gengou agar isolates that produce an exotoxin. Which of the following organisms produces a toxin that inactivates a similar target to the toxin produced by the causative organism in this patient's condition? A.Clostridium botulinum B.Corynebacterium diphtheriae C.Shigella dysentariae D.Streptococcus pyogenes E.Vibrio cholerae

E.Vibrio cholerae Pertussis toxin leads to increased cAMP levels through unopposed activation of adenylate cyclase. Vibrio cholerae also produces an exotoxin that leads to increased cAMP levels by ADP-ribosylating a Gs, causing a profuse watery diarrhea.

A 68-year-old man with a history of stroke and hypertension comes to the emergency department complaining of palpitations and lightheadedness for the past 5 hours. He states that he has experienced shorter episodes of palpitations in the past, but they were never as severe as the current one. On physical examination, the patient has an irregularly irregular pulse. Which of the following drugs should be prescribed long-term to prevent further complications? A.Aspirin B.Protamine sulfate C.Streptokinase D.Unfractionated heparin E.Warfarin

E.Warfarin People with chronic AF and multiple risk factors, including the patient in this question, require ongoing anticoagulation with warfarin. Warfarininhibits γ-carboxylation of vitamin K-dependent clotting factors and is used for long-term anticoagulation. It is taken orally and has a long half-life. The International Normalized Ratio (INR, or the ratio of the patient's prothrombin time to normal mean prothrombin time) is used to monitor the degree of anticoagulation.

As part of an experimental study investigating carbon dioxide transportation mechanisms, a 26-year-old woman is participating in a simulated long-distance bike ride in a laboratory-controlled setting. The woman exercises frequently and feels that she is in good shape. As part of the study, the researchers require the patient to consume a special glucose solution that contains a radiolabeled carbon isotope for several days before the bike ride and during the ride. After she has been riding the bike for 10 miles, several samples are taken from the patient. Where will the greatest concentration of the radiolabeled carbon isotope be found? A.In sweat B.In urine C.Within a stool sample D.Within red blood cells E.Within the plasma

E.Within the plasma The majority of CO2 is transported in the blood plasma as bicarbonate (HCO3-), after undergoing a reaction catalyzed by carbonic anhydrase. A small proportion of CO2 is transported in the plasma as dissolved CO2. About 80% of the output of CO2 from tissue is transported as bicarbonate, 10% as carbaminohemoglobin, and 10% in physical solution.

what are chelators used for lead poisoning?

EDTA dimercaprol succimer penicillimine

which bloody diarrhea causing E.coli can ferment sorbitol on mcconkey agar?

EIEC (EHEC cant ferment sorbitol)

which heart rate condition is associated with Alternations of QRS height with each beat?

Electrical alternans (shown in the image below) is pathognomonic for tamponade, representing alternate-beat variation in the direction, amplitude, and duration of any component of the ECG.

acute CHF exacerbation with pulmonary edema Which of the following is the site of action of the drug most likely to be administered to this patient? A.All water-permeable portions of the nephron via an osmotic effect B.At angiotensin II receptors in the heart, blood vessels, and kidneys C.At sites of enzymatic conversion of angiotensin I to angiotensin II in the lungs D.At the Na+- Cl- channel of the early distal tubule E.At the Na+- K<>- ATPase transporter on the myocardium F.At the Na+- K+- 2Cl- triple transporter of the thick ascending limb of the loop of Henle

F.At the Na+- K+- 2Cl- triple transporter of the thick ascending limb of the loop of Henle Furosemide acts by binding to and blocking the co-transport system of Na+- K+- 2Cl- in the thick ascending limb of the loop of Henle.This induces a diuretic effect, lowering blood volume and improving the efficiency of the heart. Long-term treatment of CHF includes lifestyle changes and pharmacotherapy. Osmotic diuretics (eg, mannitol), which function at water-permeable portions of the nephron, can worsen an acute CHF exacerbation.

A 4-day-old newborn is brought to the emergency department by his parents immediately after he sustained a seizure. The parents report that their son has had a weak cry, difficulty feeding, and a few episodes of vomiting. The infant's delivery was not complicated. Physical examination shows low muscle tone and an enlarged liver. Blood work is significant for drastically elevated levels of propionic and odd-chain fatty acids. Methylmalonic acid levels are undetectable. Further studies show:Na+: 125 mEg/LHCO3-: 24 mEq/LCl-: 83 mEq/LBlood glucose: within normal limitsUnrine ketones: positiveBlood ammonia: elevated Which of the following enzymes is most likely deficient? A.Acetyl CoA Dehydrogenase B.Carnitine C.MCAD D.Methylmalonyl CoA mutase E.Peroxisome PEX F.Propionyl-CoA carboxylase

F.Propionyl-CoA carboxylase Propionic acidemia is an autosomal recessive disorder of metabolism of odd chain fatty acids and certain amino acids (methionine, valine, isoleucine, and threonine). These substrates cannot be converted to succinyl CoA and inserted into the Krebs cycle. Clinical signs are apparent within the first few days of life and include vomiting, poor feeding, hypotonia, hepatomegaly, seizures, and increased anion gap with concomitant metabolic acidosis. Treatment consists of a low-protein diet that is free of methionine, valine, isoleucine and threonine.

A 2-year-old girl is taken to her primary care physician by her mother due to a cold that has been ongoing for 3 weeks. The patient's medical history is unremarkable and her mother is her primary caregiver. On physical examination, she appears to be in pain and shows resistance to forward flexion of the neck. Temperature is 38.4° (101.2° F), blood pressure is 110/80 mm Hg, and respiratory rate is 16 breaths/min. Culture of her cerebrospinal fluid shows:Opening pressure: 18 cm H2O (normal: 10-20 cm H20)Protein: 50 mg/dL (normal: <40 mg/dL)Glucose is 50 mg/dL (normal: 40-70 mg/dL)Leukocyte count: 300, predominantly lymphocytes (nl 0-5 x10^6/L). Which of the following describes the most likely causative agent of the child's symptoms? A.Acid-fast bacteria B.Enveloped double-stranded DNA with glycoprotein C.Enveloped positive sense single-stranded RNA virus D.Gram-negative diplococci E.Gram-positive diplococci F.Single stranded positive sense naked RNA virus G.Spirochete transmitted by ticks

F.Single stranded positive sense naked RNA virus Aseptic (culture-negative) or viral meningitis in children is often asymptomatic, but may slowly progress into a symptomatic form of the condition over a span of about three weeks. Viral meningitis is typically self resolving with a good prognosis compared to bacterial meningitis. The most common cause of viral meningitis in children are Enteroviruses, which are single stranded positive sense RNA viruses without an envelope (naked). They do not have a lipid envelope and are, therefore, resistant to acidic environments and disinfectants. The viruses are transmitted fecal-oral route. The number of cases usually decreases with the start of school in the fall. Meningitis due to enteroviruses is seen in all age groups, however, children are more commonly affected due to the mode of transmission. Management is supportive and infection self-resolves.

A 25-year-old man is brought to the emergency department following a bar fight where he sustained a deep laceration on his left cheek. He has kept pressure on the laceration with a rag to stop the bleeding. His pulse is 106/min, respirations are 16/min, blood pressure is 129/84 mm Hg, and oxygen saturation is 98% on room air. He is alert and oriented with no deficits noted on mental status examination. Pupils are equal and reactive to light, extraocular motions are intact, and there is no jaw deviation. He is able to blink, close his eyes tightly, wrinkle his forehead, and move his tongue symmetrically. Sensation throughout the face is intact. However, on closer inspection, the left corner of his mouth droops. The partially injured cranial nerve exits the skull through which of the following structures? A.Foramen magnum B.Foramen ovale C.Hypoglossal canal D.Internal auditory meatus E.Jugular foramen F.Stylomastoid foramen

F.Stylomastoid foramen The motor component of CN VII (along with CN VIII) enters, rather than exits, the internal auditory meatus; it exits the skull through the stylomastoid foramen.

This woman with progressive constipation, urinary frequency, a history of smoking, and hypercalcemia presents with a clinical picture that is suggestive of A.Cervical sympathetic chain compression B.Congenital chloride channel dysfunction C.Dynein arm defect in cilia D.Ectopic antidiuretic hormone production E.Long-term exposure to silica F.Solitary parathyroid adenoma G.Squamous cell carcinoma

G.Squamous cell carcinoma These are classic signs of squamous cell carcinoma, which can be associated with a paraneoplastic syndrome caused by secretion of parathyroid hormone-related peptide (PTHrP), resulting in hypercalcemia and a decreased endogenous parathyroid hormone (PTH) level. PTHrP binds PTH receptors, thereby increasing bone turnover, gastrointestinal absorption of calcium and phosphate, renal reabsorption of calcium, and secretion of phosphate.

A previously healthy 28-year-old woman comes to the clinic because of a 2-month history of fatigue, episodic low-grade fever, and 2.27-kg (5-lb) unintentional weight loss. She also mentions that her wrists feel slightly sore and that 2 weeks ago her knees were sore. Her only medication is an occasional acetaminophen. Temperature is 36.7°C (98°F), blood pressure is 120/86 mm Hg, pulse is 90/min, respiratory rate is 18/min, and oxygen saturation is 98% on room air. Physical examination shows symmetrically swollen, erythematous, and tender wrists and metacarpophalangeal joints bilaterally. There is no lymphadenopathy or hepatosplenomegaly. Laboratory studies show: Hemoglobin: 9.2 g/dL Mean corpuscular volume: 87 fLMCHC: 34%Reticulocyte count: 4%WBC: 5000/mm3Platelet count: 90,000/mm3Serum creatinine: 1.6 mg/dLUrinalysis is positive for 1+ blood and 1+ protein. Which of the following is the most likely underlying mechanism of this patient's anemia? A.Alpha globin gene deletion B.Bone marrow infiltration C.Destruction of RBCs by parasite D.Dysfunctional RBC membrane protein E.Immune system sequestration of iron F.Spirochete infection G.Type II hypersensitivity response H.Vitamin B12 deficiency

G.Type II hypersensitivity response The patient is an otherwise healthy 28-year-old woman with insidious-onset fatigue, weight loss, and migratory joint pains with signs of joint inflammation on examination. Lab values show normocytic anemia and increased creatinine concentration; urinalysis is positive for blood and protein. Peripheral smear is significant for spherocytes (red arrows in image). Overall, her presentation is concerning for systemic lupus erythematosus (SLE) with associated normocytic anemia and glomerulonephritis. Autoimmune hemolytic anemia is a type 2 hypersensitivity reaction because it involves the formation of IgG antibodies against antigens on the RBC surface. RBCs are subsequently coated in RBCs and cleared from circulation by the spleen. Patients with SLE can also develop cytopenias of other cell lines (thrombocytopenia, leukopenia) secondary to this same mechanism.

top 3 causes of meninigitis in newborns?

GBS Ecoli listeria/

what cancers metastasize to the ovaries?

GI

sxs of iron tox?

GI bleed hypovolemic shock

where are the non-skin sites of kaposi sarcoma?

GI tract lungs

RAS / KRAS function?

GTPase oncogene

what do most intron sequences start and end with?

GU - AG

malignancy associated with Helicobacter pylori infection, nitrosamine exposure, excessive salt intake, and low intake of fresh fruits and vegetables

Gastric adenocarcinoma

Which of the following describes the mechanism of action of the senna glycoside used to treat this patient's constipation? A.Acetylcholine receptor agonists B.Agonist of the guanylate cyclase-C receptor C.Dopamine receptor antagonist D.Increasing chloride-rich intestinal fluid secretion E.Osmosis of water into lumen F.Peripherally acting opioid antagonist G.Soluble fiber drawing water into the gut lumen H.Stimulating peristaltic activity on the intestine

H.Stimulating peristaltic activity on the intestine Drug-induced constipation is common, especially in patients who receive chronic opiate therapy. Mu-receptors in the intestines are stimulated by opioids, which results in slowed peristalsis. Slowed transit time increases water reabsorption from stool in the colon, making stools harder and less lubricated. Opiates also stimulate the tone of the Ampulla of Vater resulting in decreased biliary and pancreatic secretion. All patients with constipation, regardless of cause, should include lifestyle changes for prevention of constipation. This includes increasing dietary fiber and water intake and increasing mobility to stimulate the gastrointestinal tract. Laxatives can also be provided, which have different mechanisms of action. Stimulant laxatives, such as senna glycoside, stimulate peristalsis through enteric nerve stimulation to promote transit time.

when is budd chiari syndrome seen?

HCC myeloproliferative disorders (Polycythemia, ET, CML) OCP/Pregnancy Hypercoagulability state

Malignant conversion due to chronic cirrhosis is the most common reason for developing HCC. This mechanism is typically associated with which infection?

HCV

Focal segmental sclerosis often presents in who?

HIV

Which is the most likely probability that this patient will have an identical HLA match to his brother?

HLA genes are inherited in a codominant fashion, similar to red blood cell antigens. Individuals inherit one haplotype from each parent. Therefore, there are a total of four different haplotype combinations from two parents. There is a basic rule in HLA inheritance. The rule is: individuals have a 25% chance of inheriting all of the same HLA genes (ie, the same two haplotypes) as any one of their siblings. Individuals have a 25% chance of not inheriting any of the same HLA genes (ie, none of the same haplotypes), and individuals have a 50% chance of sharing one haplotype with one of their siblings. Therefore, individuals have a 1 in 4 chance of being an identical match with one of their siblings.

why does EDV increase -barely- with beta blockers? why does ejection time increase?

HR decreases, so there is more time for filling same reason

which vasculitis is caused by IgA deposition?

HSP

what will urine pH be like when you have hypochloremic metabolic alkalosis due to vomiting?

Hypochloremic metabolic alkalosis due to vomiting will be seen with a high urine pH because parietal cells continue to pump out HCl in exchange for bicarb. This increases the sodium bicarb in the blood which is then filtered by the kidneys and sent into the urine.

Daclizumab target?

IL-2R

which antibiotics can cause hemolysis in a G6PD deficiency?

INH sulfa dapsone primaquine nitrofurantoin

INH symptoms mnemonic?

INH = Injures Neurons and Hepatocytes

Aerosol precautions (known or suspected tuberculosis, varicella, measles, smallpox, SARS, and Ebola)

Place the patient in an airborne infection isolation room (AIIR) with monitored negative pressure and at least 6 to 12 air exchanges per hour. Room exhaust must be passed through a HEPA filter before indoor recirculation or discharged outdoors. N95 respirator should be worn with a patient with known or suspected tuberculosis. Susceptible individuals not to enter the room of patients with confirmed or suspected measles or chickenpox. Transport of the patient should be minimized; however, in unavoidable circumstances, patient should be masked during transport.

Which Ab is characteristic of primary biliary cholangitis, which is usually found in women?

Positive antimitochondrial antibodies

Effect of CHK1 and CHK2 in cell cycle?

Prevention of cell-cycle progression past the G2/M checkpoint is regulated by proteins such as CHK1 and CHK2 kinases, among others.

which cancer can transform into DLBCL?

SLL

what are the CD5 positive lymphocyte disorders?

SLL mantle cell DLBCL

what blood cancer are smudge cells seen?

SLL, CLL small lymphocytic lymphoma Chronic lymphocytic leukemia (crushed little lymphocytes)

Filamentous bacteria: what is the mnemonic for txment?

SNAP sulfa for nocardia actinomyces with penicillin

Which gene is found only on the Y gene and would only be associated with the testis determining factor for the development of the testes?

SRY

Hox gene mutations

These transcription factors bind to the DNA and allow for activation or repression of specific genes. Hox genes are primarily responsible for patterning of the anterior-posterior axis of the vertebrate embryo and limb bud. If there is a mutation in these genes it can cause syndactyly, polydactyly, an extra cervical rib, and an increased risk of cancer.

which neurocutaneous disorder causes hemangioblastoma?

VHL

where does burkitt lymphoma occur in north america?

abdomen or bone marrow

which muscles are affected by erb palsy?

abductors - deltoid, supraspinatus infraspinatus biceps

what causes epispadias?

abnormal genital tubercle positioning

which neurotransmitter signals on the adreenals?

acetylcholine (presympathetic)

where are extra-articular sites for gout?

achilles and ear

what do SERMS like tamoxifen and raloxifen do in bone?

act as estrogen agonists

what is the MOA of polymyxins and what is their use?

act like a detergent, damage the membrane; resistant Gram negative

what does angiotensin 2 do to the PCT?

activate the Na/H exchanger = more Na, bicarb and water reabsorbed; no change to H

AST≥ALT

acute alcoholic hepatitis

when does a pancrreatitic pseudocyst form?

acute or chronic pancreatitis; NOT cancer!

what kind of kidney injury do myoglobin and hemoglobin cause?

acute tubular necrosis

patient with myalgias, urinary frequency, and myoglobinuria has been consuming large quantities of grapefruit juice. In the context of statin use, these signs and symptoms point to a diagnosis of

acute tubular necrosis as a result of rhabdomyolysis Rhabdomyolysis is an adverse effect of statins, many of which are metabolized by cytochrome P-450 enzymes. Grapefruit juice inhibits P-450, thereby slowing metabolism of the statin. This results in higher concentrations of the drug and increased risk of rhabdomyolysis. Patients who take higher doses of statins are already at increased risk for rhabdomyolysis.

malignancy associated with gastroesophageal reflux disease

adenocarcinoma of the distal esophagu

Non-enveloped, double-stranded DNA

adenovirus, papillomavirus (HPV), and polyomavirus (JC, BK)

most common location of neuroblastoma?

adrenal gland medulla

what is achondroplasia associated with?

advanced paternal age

Drug of choice for treating sulfa allergic HIV pt with pnuemocystic PNA?

aerosolized pentamidine

when does brutton agammaglobulinemia present?

after 6 months

when does idiopathic thrombocytopenia purpura occur in children?

after a respiratory infection

MOA of cyclophosphamide?

alylating agent, crosslinking DNA at guanine N7

how are thrombolytics reversed?

aminocaproic acid

which stones are associated with proteus infections?

ammonium magnesium phosphate

what are the manifestations of antiphospholipid syndrome?

arterial and venous clotting recurrent miscarriages

which antimetabolites are nephrotoxic?

calcineurin inhibitors (cyclosporin and tacrolimus)

which drugs should not be taken with quinolones?

calcium - decreases absorption

what is decreased in osteomalacia? what is it also called?

calciuma and vitamin D rickets

what is the most serious complication of Monday sickness from nitrate exposure?

can develop cardiac ischemia if stopping abruptly

what is the mnemonic for triad of sxs in reactive arthritis?

can't see can't pee can't bend my knee

how do NRTIs terminate the DNA chain?

cannot form a 3' bond, whereas they can form the 5' bond and be incorporated

what is the p24 protein of HIV?

capsid

what is hb bound to CO called?

carboxy hemoglobin

what happens after a ventricular wall rupture after an MI?

cardiac tamponade

where in the lung is centriacinar emphysema found?

centriacinar is upper lobes think "smoke rises"

what are the liver findings in budd chiari syndrome?

centrolobular congestion and necrosis (zone 3)

which cephalosporins are used for pseudomonas? 3

ceraperazone -3rd ceftazadime -3rd cefepime

two common PID sxs?

cervical motion tenderness purulent discharge

Segmental sclerosis associated with hyalinosis and collapse of glomerular capillary tuft

characteristic of focal segmental glomerulosclerosis (FSGS), which often occurs in African-American patients. HIV infection is associated with the collapsing variant of FSGS (see image). Because this patient presents with cachexia, a history of IV drug use, and a history of an AIDs-defining illness (invasive cervical cancer), we can presume HIV/AIDs to be the pathology underlying the development of FSGS. Diagnosis is confirmed by renal biopsy, which shows focal and segmental sclerosis, hyalinosis, and the collapse of glomerular capillary tuft, with immunostaining showing IgM and complement (C3) deposits in a nodular and coarse pattern (large molecules like IgM and C3 trapped in sclerotic glomeruli, rather than a true immune complex-mediated glomerulonephritis).

what step is after a western blott diagnosis of HIV?

check viral load and CD4 count (more important than HAART)

immune disorder characterized by defect in microtubule function?

chediak- higashi (also has lysosomal trafficking)

what is the MOA of chelators?

chemical antagonist - binds directly to agonist, preventing it from binding receptor

tetanus toxin MOA?

cleave SNARE proteins, prevent release of inhibitory neurons

which abx is used to treat anaerobic infections above the diaphragm and which is below?

clinda above; metro below

first line treatment for lung abscesses?

clindamycin

which antihypertensive can cause depression?

clonidine

which antihypertensive can cause rebound HTN?

clonidine

what is the treatment for complicated community acquired PNA vs uncomplicated?

complicated community acquired PNA treatment: macrolide + 3rd gen ceph or levofloxacin; uncomplicated: macrolide or tetracycline

what is contraction alkalosis?

contraction alkalosis is seen where there is fluid loss, which activates the renin cascade, which increase Na reabsorption and increases K and H excretion, resulting in an alkalosis (high bicarb, high pH)

what is the drug used to treat serotonin syndrome?

cyproheptadine

what is osteoitis fibrosa cystica? what disease is it seen with?

cystic bone spaces filled with fibrous tissue primary hyperparathyroidism

where do purkinje cells send fibers in the cerebellum? what is the output? what neurotransmitter do they use?

deep cerebellar nuclei inhibitory GABA

what is the defect in hyper IgM?

defect in CD40L on T cells

why does osteomalacia occur in renal failure?

deficient vitamin D production

A flat affect in an uncooperative patient with a poor Mini-Mental State Examination score may suggest what?

delirium, dementia, or pseudodementia.

what are heinz bodies made of and when are they seen?

denatured Hb G6PD

which NRTI HIV drug can cause pancreatitis?

didanosine

what are the signs of cardiac tamponade? 3

distended neck veins muffled/distant heart sounds pulsus paradoxus - decrease in BP > 10 mmHg during inspiration

who should non selective beta blockers NOT be used in and why?

do not use non selective beta blockers in cocaine OD or lung disease because they block B2, which causes bronchoconstriction and unopposed alpha 1 presence on the vasculature

what does oculomotor palsy do to the eye? trochlear nerve? abducens? which nerve is impacted frequently in idiopathic intracranial hypertension?

down and out; vertical diplopia inward deviation abducens

where is iron absorbed?

duodenum

where does iron absorption occur in the intestines?

duodeunum and upper jejunum

does a durable power of attorney or living will hold more power?

durable power of attorney

impaired ability to perform rapid, alternating movements

dysdiadochokinesia

how does alpha 1 antitrypsin deficiency cause liver damage in addition to emphysema?

dysfunctional alpha 1 antitrypsin accumulates

what are the triad of sxs in plummer vinson syndrome and what is it associated with?

dysphagia iron deficiency anemia esophogeal webs glossitis

Protein that is basically deleted in Duchenne muscular dystrophy?

dystrophin

what are brachial clefts derived from?

eCtoderm - Clefts from eCtoderm

erythema migrans is seen in which disease?

early stage lyme disease

where are eccrine vs apocrine glands found and what does each secrete and what activates each?

eccrine sweat glands are located on the palms and soles and make watery sweat and are activated during exercise and are innervated by SANS; apocrine sweat glands are found in axilla and make oily substance and are activated by circulating catecholamines

which drugs target fungal cell walls by inhibiting B1-3 glucan synthesis?

echinocandins (all end in -fungin)

which bugs cause cysts in the liver?

echinococcus entameba histolytica

which RBCs are present in a pyruvate kinase deficiency?

echinocytes/burr cells

what are alkaline phosphate levels like in Paget's disease? phosphate?

elevated normal

Human chorionic gonadotropin is associated with which testicular tumor?

embryonal carcinoma and choriocarcinoma

which HIV drug inhibits fusion?

enFUvirtide = (inhibits gp41)

which type of bone formation is defective in achondroplasia? which is intact?

endochondral ossification intramembranous

which cancer risk is PCOS increased for?

endometrial

which cancers are lynch syndrome associated with?

endometrial ovarian skin

what is courvoisier's sign and what is it specific for?

enlarged palpable painless gallbladder along with jaundice; due to the pancreatic cancer of the head blocking bile flow over time, so its not painful

what are the environmental form vs tissue form of coccidiodes?

environmental form = arthroconidia tissue form = spherule

what is the morphology of poxvirus?

envloped, DS, linear DNA

what skin lesions is inflammatory bowel disease (UC and crohn) associated with?

erythema nodosum

side effects of bisphosphonates?

esophagitis (take with water and site upright) osteonecrosis of the jaw atypical stress fractures

which receptors do clomiphene act on?

estrogen in the hypothalamus, as an antagonist

cancer drug causing alopecia?

etoposide/tenoposide

how is a Rh positive newborn treated experiencing hemolysis?

exchange transfussion with Rh negative blood = do not give RhoGam, this will actually worsen the hemolysis because RhoGam binds Rh, causing further agglutination

how is failure to thrive diagnosed?

failure to thrive is diagnosed by growth measurements below the 5th percentile on multiple occasions or weight deceleration that crosses two major lines on the growth chart

what are ferritin and transferrin levels like in anemia of chronic disease?

ferritin increased = iron trapped in cells transferrin = decreased because iron body stores are high

what enzymes are inhibited by lead?

ferrochelatase Ala dehydratase

what are the pentad of sxs in thrombocytopenia purpura?

fever microangiopathic hemolytic anemia neuro sxs renal failure thrombocytopenia

what is lanugo and what causes it?

fine body hair all over; anorexia

what are the symptoms of interferon therapy?

flu like sxs depression myopathy neutropenia

what are the symptoms of leptospirosis?

flu like sxs myalgias in calves jaundice photophobia with suffusion (erythema without exudate)

what is a side effect of glitazones?

fluid retention which leads to heart failure, pulmonary edema and weight gain

inhaled steroid for asthma name?

fluticasone

histo findings in whipple disease?

foamy macrophages in the lamina propria

how does phenytoin cause megaloblastic anemia?

folate deficiency

what is the MOA of methotrexate?

folic acid analog

what is leukovorin and what does it treat?

folinic acid use to treat myelosuppresion with methotrexate

what happens to free T4 levels and total T4 levels in pregnancy?

free T4 is unchanged; total T4 is increased; (the total T4 buffers the increase thyroid binding globulin levels which increase in pregnancy)

what is the MOA of bleomycin?

free radical DNA damage

what infections do CVID patients get?

frequent sinopulmonary infections

txment of SAH with pt taking warfarin?

fresh frozen plasma

what disease is caused by a GAA repeat?

friedreich ataxia

what is the hypoglycemia and liver disease in fructose intolerance?

fructose 1 phosphate accumulates because aldolase B isnt working; thus there is not phosphate to break down glycogen; liver disease is just due to accumulation

antidiabetic agent which pancreatitis is a known adverse effect.

glucagon-like peptide 1 (GLP-1) analogs, such as liraglutide, has the possibility of developing pancreatitis, which should be discussed with the patient prior to initiating therapy.

Fatty acids, amino acids, and oral glucose reaching the duodenum and jejunumincrease release of

glucose-dependent insulinotropic peptide (GIP) from duodenal and jejunal K cells.

what is ammonia made into in muscle cells normally and with what?

glutamate; by combining alpha ketoglutarate and the amino from aspartate or alanine via AST or ALT respectively

what happens to the d-dimer level in DIC?

goes up

what casts are seen in ATN?

granular/muddy brown

what kind of inflammation is seen in giant cell and takayasu?

granulomatous

what is the MOA of ganciclovir?

guanosine analog that is monophosphorylated by CMV and then twice by host cells; incorporated by CMV DNA pol, inihibiting

what decreases a hypertrophic cardiomyopathy murmur?

hand grip squating

what is anti-microsomal enzyme assocaited with?

hashimotos

Enveloped, double-stranded DNA

herpesvirus, poxvirus (smallpox and molluscum contagiosum), and hepadnavirus (HBV).

where is the error in uniparental heterodisomy vs isodisomy?

hetero - meosis 1 iso - meosis 2

two glycolysis enzymes requiring ATP?

hexokinase/glucokinase PFK-1

what will renin plasma levels be like in secondary hyperaldosteronism?

high

what is the tonicity of saliva at high flow rates vs low flow rates?

high - isotonic low - hypotonic

what are LH and FSH levels like in klinefelter?

high LH and FSH

what is pes cavus?

high arches

what paraneoplastic syndrome do squamous cell carcinomas cause?

hyper PTHrP

what are the side effects of thiazide diuretics?

hyperGLUC glycemia lipidemia uricemia calciemia

which electrolyte disturbance will decrease DTRs?

hyperMg

treatment for carbon monoxide?

hyperbaric O2 - not methylene blue!

what should kidney stones make you think of?

hypercalcemia

difference between 17a hydroxylase deficiency and 21 hydroxylase deficiency as a cause of congenital adrenal hyperplasia?

hypertension and hypokalemia vs hypotension and hypovolemia

what are the sxs in Waldenström macroglobulinemia?

hyperviscosity microvascular complication -nosebleeds

is hypo or hypercalcemia seen in DiGeorge syndrome?

hypo - no parathyroid glands

what is the most common complication of a total thyroidectomy?

hypoPTH (tingling of face could be a symptom)

most common cause of tetany?

hypocalcemia

sulfonurea side effect?

hypoglycemia

what electrolyte abnormality can be caused by albuterol?

hypokalemia

side effects of amphotericin B?

hypokalemia hypomagnesium renal impairments

what can worsen hepatic encephalopathy?

hypokalemia (causes intracellular acidosis which causes renal cells to make more ammonia) alkalosis (steals H from NH4, creating more NH3)

what lab abnormality is seen in legionarres disease?

hyponatremia

which nerve runs on top of the spermatic cord?

ilioinguinal nerve L1

what is the first line treatment for CML?

imatinib (tyrosine kinase inhibitor)

what does the delta F508 mutation in cystic fibrosis cause to happen?

improper folding of CFTR

what are the triad of sxs of normal pressure hydrocephalus?

incontinence ataxia cog decline wet wobbly wacky

A 54-year-old postmenopausal woman comes to the clinic for her 12-month follow-up after starting hormone-replacement therapy for vaginal dryness, hot flashes, and dyspareunia. Her symptoms have improved ever since starting hormone-replacement therapy, and she denies any nausea, bloating, breast tenderness, heat intolerance, weight loss, or vaginal bleeding. On physical examination her vitals are within normal limits and her abdomen is soft and nontender. Her vaginal mucosa appears atrophic but moist. How are her Thyroxine-binding globulin, free T3, free T4, total T3, TSH? [incr/decr/normal]

incr TBG normal T3, T4 incr total T3 normal TSH

what happens to cAMP levels when glucose is absent in the lac operon?

increase and promotes transcription

Praziquantel MOA?

increase calcium permeability of cell membrane

what will erythromycin cause to happen when taken with certain drugs?

increase their concentration (p450)

what happens to sex hormone binding globulin levels in cirrhosis?

increased

what are MMA levels like in B12 deficiency?

increased (cant be converted to succ COA)

what are the side effects of integrase inhibitors for HIV?

increased creatine kinase

how do coporphyrin levels differ in Rotor syndrome vs Dubin johnson?

increased in rotor; normal in DJ

what is the MOA of fibrates?

increases LPL Activates PPAR-α to induce HDL synthesis

functions of gastrin

increases gastric acid secretion, growth of gastric mucosa, and gastric motility

Vasoactive polypeptide (VIP) function

increases intestinal water and electrolyte secretion and promotes relaxation of intestinal smooth muscle and sphincters

what does digoxin do to ejection fraction?

increases it (increases the amount of Ca in cells = increased contractility)

what does digoxin do to vagal tone?

increases it -thats why it slows heart rate

what does metoclopramide do to LES tone?

increases tone

when does the murmur in Hypertrophic cardiomyopathy increase and decrease?

increases with less blood filling -- tachycardia and valsalva decreases with more blood filling -- squatting, hand grip, passive leg raising

functions of secretin

increasing bicarbonate and bile secretion while decreasing gastric acid secretion

which HIV protease inhibitor causes crystal induced nephropathy?

indinavir

do direct or indirect sympathomimetics enter the CNS more rapidly?

indirect

An abrupt onset of amnesia for time, place, and recent memory is seen in?

individuals with transient global amnesia, which is more common in patients older than 65 years and usually lasts for less than 12 hours. Patients can perform previously learned tasks but must frequently be reminded where they are.

what vein drains external hemorrhoids?

inferior rectal vein

what is the second line txtment for UC after sulfasalazine fails?

infliximab (TNF ab)

Enveloped, negative-sense, single-stranded RNA virus

influenza virus

what is the formula for infusion rate?

infusion rate = steady state conc x clearance

abx causing hemolysis in G6PD?

inh sulfa dapsone primaquine nitrofurantoin

Pertussis toxin MOA?

inhibit Gi → ↑ cAMP → impairing phagocytosis to permit survival of microbe

two ways trastuzumab inhibits HER2 positive breast cancer cells?

inhibit HER2 cell signaling Ab dependent cytotoxicity

what is the MOA of bupropion?

inhibit NE and dopamine reuptake

what does digoxin do?

inhibit Na/K ATPase (out/in) which inhibbits Na/Ca exchanger (in/out) less Ca leaves

MOA of tetracyclines?

inhibit aminoacyl-tRNA binding, inhibiting elongation

azole antifungal MOA?

inhibit ergosterol synthesis

what does dopamine do to prolactin?

inhibit it (prolactin normally causes lactation)

what is the specific MOA of chloramphenicol?

inhibit peptidly transferase

what is the MOA of methylxanthines?

inhibit phosphodiesterase = increases cAMP = Gs effects

what does somatostatin do?

inhibit release of everything in the GI tract basically; eg no insulin, glucagon, exocrine enzymes, gastric acid

What is the MOA of trazodone?

inhibit serotonin reuptake and acting like a serotonin agonist

what is the specific MOA of tetracyclines?

inhibit tRNA binding to A site in 30s

what is the MOA of etoposide/tenoposide?

inhibit topoisomerase 2

what is the MOA of amantadine?

inhibit uncoating

cimetidine p450 effect?

inhibition

what does increased NADH/NAD do to the TCA cycle?

inhibits

what does increased NADH/NAD do to gluconeogenesis?

inhibits (because oxaloacetate is not made)

what does increased NADH/NAD do in the liver of alcoholics?

inhibits beta oxidation

what is the MOA of cyclosporine?

inhibits calcineurin by binding cyclophilin preventing IL-2 transcription

what are the three common manifestations of tuberous sclerosis? what is a less common symptoms?

intellectual disability seizures facial angiofibromas cardiac rhabdomyoma

back-and-forth tremor in the trunk of the body

intention tremor

how are internal vs external hemorrhoids different?

internal - not painful, but more likely to bleed

where are external vs internal hemorrhoids located?

internal above pectineal line external below

what are the drugs of choice for digoxin induced arrhythmias?

lidocaine and mexilitine

what is the morphology of herpes viruses?

linear DNA

what are the short acting insulins?

lispro aspart glulisine

what does entamoeba cause other than bloody diarrhea?

liver abscess by ascending the portal vein

why is there excess ammonia in liver disease?

liver can no longer operate the urea cycle

what is contraction alkalosis and when does it occur?

losing large amount of fluid without losing bicarb; occurs with loop diuretics (lots of fluid lost) and cystic fibrosis (cant excrete bicarb in sweat)

Hirchsprung pathophysiology

loss of both Auerbach (Auerbach in muscularis externa) and Meissner plexus (Meisser in submucosa)

Achalasia pathophysiology

loss of myenteric (Auerbach) plexus in muscularis externa due to loss of postgang (NO and VIP)

what are phosphate levels like in rickets?

low

how does the body get Biopterin?

makes it itself

most common childhood brain tumor and where it is found?

medulloblasotma cerebellum

Brain tumor with Deeply staining nuclei with scant cytoplasm arranged in pseudorosettes

medulloblastoma

what is the most common brain tumor in children?

medulloblastoma

Diffuse capillary and basement membrane thickening

membranous glomerulonephritis.

Immune complex deposition in subepithelial space

membranous nephropathy

Brain tumor with Whorls of meningothelium, oval nuclei with indistinct cytoplasm, and psammoma bodies

meningioma

brain tumor with whorls of fibrous tissue?

meningioma

what are brachial arches derived from?

mesoderm and neural crest cells

why is cystic fibrosis associated with infertility?

mesonephric duct improperly forms = defective vas deferens

what acid base disturbances does aspirin cause?

metabolic acidosis resp alkalosis

what does bulemia do to body pH?

metabolic alkalosis

what are complications of citrate contained in transfused blood?

metabolic alkalsois hypocalcemia - binds calcium

which lysosomal storage disease causes central and peripherla neuropathy?

metachromatic leukodystrophy

what has to be the target of a cancer drug if it affects mitosis?

microtubules/preventing spindle cell formation

what is Trousseau syndrome and what causes it?

migratory thrombophlebitis in pancreatic cancer; caused by release of clotting factors

what are the symptoms of Dubin johnson syndrome if present?

mild jaundice with illness

what are the two mitochondrially inherited diseases?

mitochondrial myopathy leber hereditary optic neuropathy

how do bone lesions differ in multiple myeloma vs metastatic prostate cancer?

multiple myeloma lesions are osteolytic, whereas metastatic prostate cancer are osteoblastic

how is mycoplasma diagnosed?

multiplex PCR from nasopharyngeal swab

deficient enzyme in McCardle disease?

muscle glycogen phosphorylase (myophosphorylase)

special instructions for neuroaminidase inhibitors?

must be given within 48 hrs of onset of sxs

what is wrong in nephritic vs nephrotic syndrome?

nephritic = GBM nephrotic = podocyte

what are the side effects of indinavir?

nephropathy hematuria thrombocytopenia

side effects of the platinum analogs according to chemo tox man?

nephrotoxic

side effects of polymyxins?

nephrotoxic neurotoxic respiratory failure

major side effect of acyclovir?

nephrotoxicity

What are melanocytes derived from?

neural crest cells

what cells give rise to neuroblastoma?

neural crest cells

what does increased alpha fetoprotein level in pregnancy mean?

neural tube defects twins

what symptoms do peroxisome diseases lead to?

neuro diseases

what is the most common cancer in infants?

neuroblastoma

what are the sxs of churg-strauss in addition to asthma, eczema, and rash?

neuropathy

What kind of patients does aspergillous like?

neutropenic

which drugs are used for HTNive emergencies?

nitroprusside fenoldopam

carmustine MOA?

nitrosurea - alkylate DNA

does the hemoglobin concentration change the PaO2 of blood?

no

is a negative PPD reliable with advanced HIV?

no

is amantadine and rimantadine still used for influenza?

no

is cervical cancer associated with paraneoplastic syndrome?

no dont assume a cancer causes paraneoplastic sydnrome

is pyruvate made in gluconeogenesis?

no (oxaloacetate is made directly into PEP)

how does multiclonal gammopathy of undetermined significance differ from multiple myeloma? what is the treatment?

no crab findings monitor 6 mo

which branching bacteria (nocardia or actinomyces) is acid fast staining?

nocardia

what are the symptoms of leber hereditary optic neuropathy?

painless vision loss movement disorders cardiac sxs MS sxs

only tongue muscle to get vagus input?

palatoglossus

What causes nephrotic syndrome?

podocyte damage → damages charge

Segmental, transmural fibrinoid necrosis

polyarteritis nodosa

what musculoskeletal condition is associated with giant cell arteritis?

polymyalgia rheumatica

increased endomysial infiltration of lymphocytic cells

polymyositis

how do you remember the enzyme in pompe diseease?

pompe trashes the PumP (1,4) glucosidase

which enzyme is deficient in acute intermittent porphyria?

porphobilinogen deaminase

is candida germ tube positive or negative?

positive

which nephritic syndrome is caused by subEPIthelial immune complex deposition and which is caused by subENDOthelial immune complex deposition?

post stEP - subEPithelial deposits diffuse proliferative - subendo

what is "starry sky" immunoflourescence on glomerulous seen in?

post strep GN

Enlarged hypercellular glomeruli with neutrophils

poststreptococcal glomerulonephritis

what causes congential indirect hernias?

process vaginalis (continuous with the peritoneal cavity) does not close

what is the most common type of pituitary tumor?

prolactinoma

what occurs in intraductal papilloma?

proliferation of normal glands and ducts

what are the xceptions to informed consent?

pt cant make decisions emergency information would cause too much harm patient waives right

side effects shared by busulfan and bleomycin?

pulm fibrosis hyperpigmentation

which antibiotic can cause tendon rupture/tendonitis?

quinolones

what are the classical sxs of juvenile idiopathic arthritis?

quotidian fevers (daily) arthritis salmon pink rash

eosinic intracytoplasmic inclusions

rabies

what is the Fick principle formula for cardiac output?

rate of O2 consumption/ (arterial - venous O2 content)

how do right sided colon cancers differ from left sided in presentation?

right - bleed more, anemia, fatigue left - obstruction, constipation

what does an increase in most things do to the Hb dissociation curve?

shifts it right

what does increased pH do to the hb dissociation curve?

shifts left

which diarrheal diseases are reportable?

shigella salmonella

how do shigella and salmonella differ on H2S production?

shigella produces H2S, salmonella does not sHegella H2s - has an H in it

which liver fluke penetrates the skin?

shistosoma monsai

how are sodium and potassium levels changed in saliva when flow rate increases RELATIVE to normal saliva?

sodium will be increase and potassium will be decreased because sodium is normally reabsorbed along the ducts and potassium is normally secreted along the ducts

what does dromotropic refer to and what controls it?

speed through the AV node; Calcium

imaging finding in budd chiari syndrome?

spiderweb collateral veins

what kind of fractures support child abuse diagnosis?

spiral

what is the treatment for hereditary spherocytosis?

splenectomy

what is the most common cyanotic Congenital heart defect?

tetrology of Fallot truncus arteriosus secoond

which microcytic anemia causes RBC to be normal or high?

thalasemia

insurance premium

the amount of money paid by the patient for an insurance policy. It is considered income to the insurance company once it is earned and represents a liability in that the insurer must provide coverage for claims being made against the policy.

which diuretic can be used to reduce risk of calcium kidney stones?

thiazide

main txtment for grave's disease? MOA?

thioamides; blocks tyrosine iodination and coupling

txtment of cyanide?

thiosulfate hydroxycobalamin

how do fat soluble vitamins first enter circulation?

thoracic duct - not portal vein!

what type of deletion causes cystic fibrosis?

three codon deletion

how does hartnup disease lead to pellegra?

tryptophan (and other neutral amino acids are not absorbed) and thus niacin is not made

which neurocutaneous disorder has a rhabdomyoma?

tuberous sclerosis

uberous sclerosis mutation on chromosome?

tuberous sclerosis (mutation on chromosome 9) can cause renal hamartomas

kidney disease associated with hep B and C?

type 1 membranoproliferative GN

which collagen is defective in vascular Ehlers Danlos?

type 3

primary virulence factor of N. gonorrhea?

type 4 pili

Coinsurance

type of insurance plan in which a portion of medical costs is paid by the patient until the deductible is met. Once the deductible is met, the health plan coverage can be implemented.

is typhoid or non typhoid salmonella treated with abx?

typhoid

chlorpromazine class? Chlordiazepoxide class?

typical antipsychotic benzo

how does the MOA of atypical antipsychotics differ from typical?

typical only block dopamine receptors; atypical block dopamine, serotonin, and adrenergic

Intramural inflammation and crypt abscesses are seen in which GI condition?

ulcerative colitis

what is the formula for GFR?

urine inulin conc x flow rate / plasma inulin conc

is Kd similar to Km?

yes

is a bronchogenic carcinoma any type of lung cancer?

yes

Does fifth's disease present after fevers have resolved? what is the txtment? what additional sxs does it cause in adults?

yes; no txtment; joint pain

Which enzyme does the toxin present in these mushrooms inhibit?

α-Amanitin is a selective inhibitor of RNA polymerase II and III. RNA polymerase II produces heterogeneous nuclear RNA, which is processed into mRNA. Inhibition of RNA polymerase II thus leads to a decrease in the production of mRNA, and cell metabolism ceases due to the lack of transcription and translation. The liver is often the first major organ that encounters the toxin following absorption from the gastrointestinal tract.

Norepinephrine [incr/decr] platelet aggregation?

α2-adrenergic receptors promote platelet aggregation when activated. Norepinephrine activates these receptors, so an increase, rather than a decrease, in platelet aggregation would be expected after norepinephrine is administered to this patient.

when does a child walk by?

12-18 months

treatment for n.meningitidis?

3rd gen ceph

what DEXA score diagnosis osteoporosis?

< -2.5

what CD4 count does toxoplasmosis occur?

< 100

Which cells maintain blood brain barrier?

Astrocytes

where do CLL/SLL lymphocytes originate? (like from what type of cells? what abnormal cell marker do they have?

B cells CD5 positive (normally seen on T cells

which type of cells are malignant in CLL/SLL?

B cells (remember, it can transform to diffuse large B cell lymphoma)

huntington repeat mnemonic?

CAG Caudate and decreased Ach and GABA

what is the treatment for von willebrand disease?

desmopressin

fenoldopam MOA?

D1 agonist

oral motor ataxia

dysarthria

what is the most common cause of hereditary aplastic anemia?

fanconi

which glomerular disease has dense deposits within the basement membrane?

MPGN 2

Mg antiarrhythmic use?

Torsades de pointes digitalis tox

A 19-year-old man comes to the university clinic because of nausea and fatigue since moving into his new dorm yesterday. He has been a bit nervous about beginning his first semester of college in Colorado, and finds himself missing his home on the West coast. He has had occasional difficulty breathing that is worse when arranging his furniture in the dorm. He recalls having a nosebleed this morning but was able to stop the bleeding with gentle pressure. He had to walk to the clinic this morning and mentions that he is having trouble catching his breath. He is a healthy, well developed man with no history of anxiety or depression. He denies any alcohol, tobacco, or illicit drug use. He does not take any medication and has been looking forward to visiting the mountains tomorrow before classes begin. The patient appears stable. The lungs are clear to auscultation bilaterally. Results of laboratory studies are pending. Which of the following is most likely to be found in this patient? A. Decreased PaCO2 B. Decreased ventilation C. Increased BPG (biphosphoglyerate) D. Increased PaO2 E. Increased blood Hb F. Increased renal excretion of HCO3-

A. Decreased PaCO2 This patient presents with nausea, fatigue, and dyspnea, symptoms that are fairly non-specific and require further investigation. He has traveled from the West coast to Colorado, an area of high altitude, in the last 24 hours. This patient is likely suffering from acute altitude sickness. The mechanism of altitude sickness begins when a person is exposed to a decreased atmospheric oxygen pressure. This causes a decreased PaO2 inspired by the lungs which is perceived as hypoxia by the body. The body attempts to compensate by increasing the ventilation to increase PaO2. However, increasing ventilation causes more CO2 to be blown off from the blood, decreasing the blood HCO3-, and increasing the pH. Respiratory alkalosis is defined by an increase in blood pH with a decrease in PaCO2 (less than 35 mmHg). Increased hemoglobin, increased biphosphoglyerate, and increased renal HCO3- excretion are chronic compensatory mechanisms that occur when a person is exposed to decreased atmospheric oxygen (high altitude), but this patient has acute symptoms. Ventilation is increased, not decreased, in response to decreased atmospheric oxygen to compensate for the hypoxia. PaO2 is decreased, not increased, in exposure to high altitude due to decreased atmospheric pressure.

A 17-year-old boy goes to the emergency department because of significant bulging of his left eye 2 weeks after being struck in the face by a baseball. The patient thinks that his eye is infected. His past medical history is noncontributory and he takes no medications. His temperature is 38.2° C (102° F), blood pressure is 118/70 mm Hg, and pulse is 120/min. Physical examination shows severe edema surrounding the left orbit, with diffuse redness, and tenderness to the touch. During a complete extraocular muscle assessment, the patient is unable to keep his left eye open for the full duration of the examination. The left eye is severely restricted in abduction and is significantly more constricted than the right eye, even in the absence of light stimulation. Which of the following structures is most likely involved? A.Abducens nerve B.Facial nerve C.Oculomotor nerve D.Optic nerve E.Trochlear nerve F.Vestibulocochlear nerve

A.Abducens nerve The patient has significant orbital swelling as the result of a traumatic injury and possible infection. He exhibits proptosis, chemosis (edema of the conjunctiva), and erythema with accompanying fever. The most likely diagnosis in this patient is cavernous sinus syndrome, in this case it is secondary to cavernous sinus thrombosis. Thrombosis is a life-threatening disorder that can follow a traumatic injury or surgery, especially in patients with a thrombophilic disorder. Cavernous sinus thrombosis can also complicate facial infections, sinusitis, orbital cellulitis, pharyngitis or otitis. Patients may experience fever and headache, and eye findings include periorbital swelling and ophthalmoplegia. Cavernous sinus syndrome presents with decreased corneal sensation, Horner syndrome, and variable ophthalmoplegia.The abducens nerve (also called CN VI) is the cranial nerve most susceptible to injury, which can result in partial ophthalmoplegia with limited eye abduction. The cavernous sinus refers to a collection of venous sinuses on either side of the pituitary. This structure acts as a conduit that drains blood from the eyes and superficial cortex into the internal jugular vein. Cranial nerves III, IV, V1, V3, and VI, plus postganglionic sympathetic pupillary fibers, all pass through the cavernous sinus. The cavernous portion of the internal carotid artery is also located within the cavernous sinus.

A 28-year-old man decides to quit his "bad habits for good." He has been consuming up to 15 drinks of alcohol per day for the past year and using cocaine on the weekends. He also uses marijuana, heroin, and opioids. After a day of abstinence, he begins to notice small monkeys on surfboards and continues to hear car horns even though he is 10 miles from the nearest road. He also feels like there are bugs crawling over his skin. He otherwise feels well and is alert. This patient is most likely suffering from which of the following? A.Alcoholic hallucinosis B.Cocaine withdrawal C.Delirium tremens D.Panic disorder E.Schizophrenia

A.Alcoholic hallucinosis This patient, who is experiencing hallucinations within 24 hours of stopping his substance abuse, is most likely suffering from alcoholic hallucinosis. Alcoholic hallucinosis refers to the onset of visual, auditory, and tactile hallucinations within 12-24 hours after the most recent drink. Delirium tremens causes hallucinations, disorientation, tachycardia, hypertension, hyperthermia, agitation, and diaphoresis in the setting of acute reduction or abstinence from alcohol. In contrast to delirium tremens, alcoholic hallucinosis is not associated with global clouding of the sensorium but only with specific hallucinations, and vital signs are usually normal.

A 68-year-old man comes to his physician because of cough, dyspnea, and weight loss. Radiographic studies of the chest show a "coin lesion" lung mass with mediastinal adenopathy. A biopsy specimen taken during bronchoscopy is stained with hematoxylin and eosin and reveals small blue cells with scant cytoplasm and areas of necrosis. The patient is started on a regimen of drugs, including one that inhibits topoisomerase II. Which of the following is the most likely adverse effect of this inhibitor? A.Alopecia B.Hemorrhagic cystitis C.Hepatotoxicity D.Peripheral neuropathy E.Pulmonary Fibrosis

A.Alopecia Etoposide is often used to treat small cell lung cancer as well as prostate cancer and refractory testicular carcinoma. The anticancer action of etoposide occurs during the transition from the S to the G2 phase of the cell cycle through inhibition of topoisomerase II. During the normal function of DNA gyrase (also known as topoisomerase II), DNA strands are broken and carefully re-ligated to pass one strand of DNA through another. Etoposide interferes with religation of the DNA strand, causing irreversible double-strand breaks in DNA. These breaks prevent the synthesis of components needed for mitosis. Side effects of etoposide include alopecia and myelosuppression.

A 62-year-old man with a history of hyperlipidemia and multiple transient ischemic attacks presents to the emergency department after 2 hours of left-sided weakness in his upper extremity and face. After a CT of the head shows no evidence of intracranial hemorrhage, treatment is administered. Two hours later the man develops bleeding from his gums as well as several large subcutaneous ecchymoses. Which of the following therapies would best stop this man's bleeding? A.Aminocaproic acid B.Argatroban C.Fresh frozen plasma D.Protamine sulfate E.Recombinant factor VIII F.Vitamin K

A.Aminocaproic acid Potential adverse effects of tPA include excessive bleeding, evidenced by this patient bleeding from his gums and developing subcutaneous ecchymoses. What agents can best stop the bleeding? Aminocaproic acid is a good choice because it is an antifibrinolytic that acts by blocking the conversion of plasminogen to plasmin, effectively counteracting the mechanism of tPA. None of the other answer choices would reverse bleeding associated tPA: Protamine sulfate reverses the effects of heparin. Vitamin K treats warfarin overdose. Argatroban is used for anticoagulation in patients with heparin-induced thrombocytopenia. Recombinant factor VIII is used to treat hemophilia A. Fresh frozen plasma is used for reversal of warfarin toxicity.

A 58-year-old man is transported to the emergency department with the complaint of chest pain after eating a large steak dinner. During evaluation, he describes the chest pain as a constant heavy pressure that is localized to the area beneath his sternum. He has a history of coronary artery disease, hyperlipidemia, hypertension, and is obese by BMI. His current medication regimen includes simvastatin for his hyperlipidemia, and enalapril for hypertension prescribed during a visit to his primary care physician earlier in the week. The patient also states that he has new-onset erectile dysfunction and wonders if these symptoms are connected in any way. While discussing the man's condition with his wife, the patient begins coughing, and within 10 minutes he experiences nausea, shortness of breath, and diaphoresis. Which of the following is the most likely cause of his symptoms? A.Angina secondary to acute coronary syndrome B.Autodigestion of the pancreas by pancreatic enzymes C.Esophageal spasm due to consumption of large meal D.Gastroesophageal reflux due to consumption of a large meal E.Pulmonary embolism due to venous stasis

A.Angina secondary to acute coronary syndrome The patient's symptoms of intense chest pain, nausea, shortness of breath, and diaphoresis suggest angina secondary to acute coronary syndrome,most likely myocardial ischemia. His history of coronary artery disease, hyperlipidemia, and obesity predispose him to rupture of cholesterol plaques, which can lead to coronary artery occlusion, which manifests as myocardial ischemia.

A 20-year-old man is brought to the emergency department by the police after being picked up in the streets for acting violently. He was reportedly trying to fight with a cashier and two other bystanders at a gas station convenience store, after attempting to shoplift alcohol. The physician has trouble getting the man to cooperate during the interview, and does not learn anything about his medical history. On physical examination, the patient's temperature is 38.5°C (101.3°F), heart rate is 115/min, blood pressure is 140/95 mmHg, and he has vertical and horizontal nystagmus. Which of the following treatments should this patient receive? A.Benzodiazepines B.Flumazenil C.N-Acetylcysteine D.Naloxone E.Sodium bicarbonate

A.Benzodiazepines His belligerent demeanor along with his physical symptoms—fever, elevated heart rate, hypertension, and nystagmus—are hallmark signs of phencyclidine (PCP) intoxication. PCP acts as an NMDA receptor antagonist. The treatment is typically benzodiazepines, though haloperidol may be used for severe symptoms.

A 27-year-old man comes to the clinic because of a 2-day history of "tender bumps" in his groin. He is sexually active and reports having unprotected intercourse with men and women. He denies any dysuria, discharge, fevers, joint pain, myalgias, rash, or recent illness. He does, however, report that he noticed a small painless ulcer on his penis that resolved on its own. The ulcer started at approximately the time he was in the Caribbean for vacation 1 month ago. His temperature is 37.4°C (99.3°F), pulse is 88/min, respirations are 16/min, and blood pressure is 122/70 mm Hg. There is bilateral tender inguinal lymphadenopathy with 2-cm overlying ulcerations. Which of the following is the most likely cause of this patient's condition? A.Chlamydia trachomatis B.Haemophilus ducreyi C.Herpes simplex D.Klebsiella granulomatis E.Treponema pallidum

A.Chlamydia trachomatis Lymphogranuloma venereum (LGV) commonly occurs in hetereosexuals in tropical/subtropical areas or in men who have sex with men. The first stage of infection is a painless ulcer that spontaneously heals. This painless ulcer is often missed due to its small size and asymptomatic nature. The secondary stage of LGV infection involves buboes, which are ulcerated lymph nodes appearing in the inguinal region. Infection can be characterized by the groove sign (shown below by the bilateral inguinal bulging). Late-stage LGV involves fibrosis and strictures of the anal tract, infertility, or elephantitis but is rare as most cases are treated by doxycycline. For pregnant patients, azithromycin may be used.

A 15-year-old boy comes to the emergency department after falling off his skateboard. Physical examination reveals a swollen knee with bruising and erythema. Popliteal and pedal pulses are 2+, but he is unable to dorsiflex or evert at the ankle. In addition, the patient reports pain and numbness in the lateral leg and dorsum of the foot. When asked to walk, he raises his affected leg high off the ground, and his foot slaps the ground when walking. He is diagnosed with a fracture. Which of the following nerves is most likely to be compromised by the fracture? A.Common fibular (peroneal) nerve B.Deep peroneal nerve C.Femoral nerve D.Superficial peroneal nerve E.Tibial nerve

A.Common fibular (peroneal) nerve The common fibular (peroneal) nerve gives rise to the superficial and deep fibular (peroneal) nerves. The superficial fibular (peroneal) nerve innervates the muscles of the lateral compartment of the leg that cause eversion (fibularis [peroneus] longus & brevis). The deep fibular (peroneal) nerve innervates the anterior compartment of the leg that is responsible for dorsiflexion and eversion (extensor digitorum longus, extensor hallucis longus & tibialis anterior). The deep peroneal nerve innervates the anterior compartment muscles responsible for ankle dorsiflexion and toe extension. Damage to the deep peroneal nerve would cause inability to dorsiflex at the ankle as well as foot drop, both seen in this patient. Yet an injury only to the deep peroneal nerve would not cause pain and numbness in the lateral leg and dorsum of the foot, as seen in this patient.

A 35-year-old woman who is a pianist comes for evaluation because of worsening pain in her fingers. She has had rheumatoid arthritis for the past 10 years. She has tried many medications over the years, but her symptoms have not resolved. Recently, the pain in her fingers has hindered her ability to play the piano. At her last appointment, she was prescribed a new medication. Since starting this medicine, she has noticed ulcers in her mouth and persistent diarrhea. On physical examination today, she has eight obvious ulcers in her mouth that are extremely painful to the touch. There is no discharge from them, and she does not have any exudate on her tongue or mouth. There is no cervical lymphadenopathy. Which of the following enzymes is likely targeted by the patient's new medication? A.Dihydrofolate reductase B.Dihydroorotate dehydrogenase C.Inosine monophosphate dehydrogenase D.Phosphoribosyl pyrophosphate amidotransferase E.Ribonucleotide reductase F.Thymidylate synthase

A.Dihydrofolate reductase The side effects of methotrexate are caused by depletion of folate in the body. The ulcers and diarrhea this patient is experiencing are common side effects because tissues that have higher cell turnover are most affected by the blockade of pyrimidine synthesis. Pyrimidines and purines are used as the basis for nucleic acids. If the body does not have nucleic acids then the cells cannot replicate.

A 12-year-old boy pitches a fastball in a baseball game and immediately feels a sudden, tearing pain in his right shoulder. He is rushed to the emergency department. Physical examination reveals localized tenderness around his right shoulder. The physician asks the patient to stand with his right arm at his side, his elbow at 90 degrees, and his humerus medially rotated to 45 degrees. Application of medial rotation force by the physician elicits pain when the patient tries to laterally rotate his arm. Motions of abduction, adduction, and internal rotation of the shoulder are all intact. His medical history is not significant, and he is currently not taking any medications. Which of the following muscles is most likely injured? A.Infraspinatus B.Long head of biceps brachii C.Subscapularis D.Supraspinatus E.Teres major F.Teres minor

A.Infraspinatus

A 2-month-old male infant is brought to the pediatrician with signs of visual impairment. He was born at 40 weeks by a then gravida 1, para 0, mother who had a history of genital lesions and who did not receive prenatal care. Genital lesions were said to be absent during the childbirth. Fundoscopic examination of the infant at the current visit reveals inflammation of the choroid and scarring of the retina. Vital signs are normal and physical examination is otherwise unremarkable. A CT scan is ordered and diffusely scattered intracranial calcifications are seen. Which of the following is the most likely source of this patient's condition? A.Maternal contact with cat feces B.Maternal contraction of mononucleosis-like illness during pregnancy C.Maternal history of a painless ulcer D.Maternal ingestion of raw milk during pregnancy E.Neonatal contact with genital lesion during birth F.Presence of a rash, lymphadenopathy, and joint pain during pregnancy

A.Maternal contact with cat feces An infant born to a mother with absent prenatal care who presents with chorioretinitis and intracranial calcifications is likely to have contracted congenital toxoplasmosis.

The image shown is a photomicrograph depicting normal cerebellar architecture. The large cells in the image integrate the cerebellar cortical activity and transmit that information to the deep cerebellar nuclei. Which of the following neurotransmitters is used by the large integrating neurons? A.γ-Aminobutyric acid B.Acetylcholine C.Dopamine D.Glutamate E.Glycine

A.γ-Aminobutyric acid The question shows an image of a normal cerebellum and is asking about the large cells transmitting cerebellar cortical activity to deep cerebellar nuclei. The cells in question (circled) are Purkinje cells. Since Purkinje cells are inhibitory cells in the cerebellum, they primarily release GABA or γ-Aminobutyric acid (an inhibitory neurotransmitter) onto the neurons of the deep cerebellar nuclei.

A 2-year-old girl is brought to the clinic by her adoptive parents for a well-child examination. The patient's parents do not know the detailed medical history of her biological parents. Her height is below the second percentile, and her weight is below the third percentile. On examination, the patient appears to have a normal trunk size, but she has a large head, short arms, and short legs. The remainder of the examination is within normal limits. Which of the following is associated with this condition? A.Advanced maternal age B.Advanced paternal age C.Growth hormone deficiency D.Iodine deficiency in utero E.Thalidomide exposure in utero

B.Advanced paternal age This patient is much shorter than children of the same age. In addition, her limbs are proportionately shorter than her torso, and her head is relatively enlarged. It is likely that she has an autosomal dominant disorder known as achondroplasia, which is the most common form of congenital dwarfism. The condition is the result of a genetic defect in the fibroblast growth factor receptor 3 (FGFR3), which causes an abnormality in cartilage formation. Mutations in the FGFR3 gene are associated with advanced paternal age. Gain-of-function mutations in FGFR3 cause a constitutively active receptor with impaired chondrocyte proliferation and endochondral bone formation. Patients affected by achondroplasia present with growth defects that are particularly pronounced in the limbs. As shown in the images below, achondroplasia results in short limbs with a normal sized trunk and a large head relative to the limbs (like the disproportionate growth observed in this patient).

A 30-year-old man in acute distress is brought by ambulance to the emergency department after being involved in a motor vehicle collision. On arrival, his blood pressure is 90/60 mm Hg, pulse is 110/min, and respiratory rate is 24/min. After fluid resuscitation, the patient's blood pressure is 92/65 mm Hg, pulse is 101/min, and respiratory rate is 20/min. CT scan of the abdomen shows bleeding, and an x-ray shows a fracture of the right femur. The patient is scheduled to undergo an exploratory laparotomy and internal fixation of the right femur by two surgeons simultaneously. Before surgery can proceed with this patient, which of the following should occur with respect to the timeout? A.A single timeout covering both surgical procedures can be performed B.All relevant members of the surgical team should communicate actively during timeout C.The surgeon must start the timeout. D.The timeout can be minimized because of this patient's emergent status

B.All relevant members of the surgical team should communicate actively during timeout The Joint Commission (JC) is an independent, not-for-profit organization that accredits and certifies more than 21,000 health care organizations and programs in the United States. In 2004, the JC introduced the Universal Protocol as a means of ensuring patient safety during surgical procedures. The Universal Protocol is summarized in the table.

A 52-year-old woman comes to the office because of a lump on her left breast. She feels well and denies any medical problems. She takes a multivitamin, aspirin, and "an antacid" daily. She stopped smoking after getting married 20 years ago. Her only surgery was a bilateral tubal ligation last year. Physical examination reveals an approximately 1.5-cm mass in the left breast that is firm to palpation; there are no skin changes, nipple inversion, or nipple expression. Mammography shows irregular microcalcification. The result of fine-needle aspiration biopsy is suggestive of infiltratingductal carcinoma, and the patient undergoes lumpectomy. Which of the following is a method of secondary prevention shown to reduce mortality rates from this disease? A.Annual breast ultrasound B.Annual mammography C.Annual screening X-ray of the chest D.Monthly breast self-examination E.Prophylactic bilateral mastectomy F.Smoking cessation

B.Annual mammography Among the choices offered, annual mammography is the best secondary prevention measure and has been shown to reduce breast cancer mortality rates by promoting early detection. In this case, the disease has already been detected by biopsy; so annual mammography would prevent the recurrence of local or metastatic disease.

The dentist knows the patient is allergic to penicillin, so he prescribes an alternate regimen. Ten days later, the boy develops fever and watery, foul-smelling diarrhea and visits an urgent care clinic. His temperature is 39.1°C (102.4°F), pulse is 92/min, respirations are 14/min, and blood pressure is 114/72 mm Hg. Findings on cardiorespiratory examination are unremarkable, but hyperactive bowel sounds are auscultated over all four abdominal quadrants. What is the primary antimicrobial mechanism of action of the antibiotic the patient is taking? A.Blocking the 30S ribosomal subunit B.Blocking the 50S ribosomal subunit C.Forming a toxic metabolite D.Inhibiting peptidoglycan cross-link formation E.Inhibiting synthesis of essential vitamins

B.Blocking the 50S ribosomal subunit Clindamycin is active against gram-positive aerobes and gram-negative anaerobes, making it an excellent choice for managing dental infections, particularly in patients with penicillin allergy. (Amoxicillin is usually the first-line treatment.) Clindamycin primarily disrupts bacterial protein synthesis by reversibly binding the 50S subunit of the bacterial ribosome, inhibiting ribosomal translocation and peptide synthesis.

A 69-year-old man who has advanced colorectal adenocarcinoma comes to the oncologist because of dizziness, difficulty walking, double vision, and slurred speech. He has been receiving chemotherapy with fluorouracil for the past 3 months. In prior office visits, he has had no adverse effects other than mild nausea and vomiting. Temperature is 37° C (98.6° F), blood pressure is 118/68 mm Hg, pulse is 70/min, and respiratory rate is 18/min. Physical examination shows nystagmus; finger-to-nose tests are abnormal bilaterally. This patient's signs and symptoms are most likely attributable to degeneration of which of the following structures? A.Basal ganglia B.Cerebellum C.Frontal cortex D.Parietal cortex E.Pons F.Thalamus

B.Cerebellum Chemotherapy can be associated with reversible and permanent cerebellar ataxia. Cytarabine, often used in the treatment of leukemias and lymphomas, and fluorouracil, which is used in various cancer treatments, including colon cancer therapy, are the most common chemotherapeutics associated with acute cerebellar ataxia. An acute cerebellar syndrome occurs rarely in patients receiving fluorouracil; the ataxia associated with fluorouracil may not develop until weeks to months after the chemotherapy has been administered. Signs and symptoms of cerebellar degeneration may include a wide-based, uncoordinated walk (limb ataxia, dysmetria); a back-and-forth tremor in the trunk of the body (intention tremor); uncoordinated movements of the arms and legs (dysdiadochokinesia); slow and slurred, monotonous and irregular speech (scanning speech); oral motor ataxia (dysarthria) and nystagmus (ipsilateral, gaze evoked, broken smooth pursuits).

The patient's protuberant abdomen, palmar erythema (redness of the palms), pronounced gynecomastia (swollen breast tissue), jaundice, and altered mental status are most concerning for hepatic encephalopathy from liver disease (cirrhosis). Which of the following is a consequence of this patient's most likely elevated compound? A.Decrease in levels of carbamoyl phosphate B.Decreased levels of α-ketoglutarate C.Decreased production of glutamine D.Decreased production of pyruvate E.Decreased utilization of glutamate

B.Decreased levels of α-ketoglutarate Excess ammonia is shuttled to the liver via alanine which transaminates alpha-ketoglutarate to produce pyruvate and glutamate. Glutamate dehydrogenase then deaminates glutamate, producing ammonia which can be used to synthesize carbamoyl phosphate, the substrate for the urea cycle. Therefore, if there is an excess level of ammonia in the body, as in our patient with hepatic encephalopathy, the high levels of ammonia will deplete the levels of α-ketoglutarate during its conversion into glutamate and ultimate conversion into urea.

A female newborn is brought to the emergency department because of persistent vomiting and decreased muscle tone. The patient was delivered at term via a normal spontaneous vaginal birth with no complications. According to the patient's mother, her aunt has a history of a genetic disease that causes low blood pressure but is unsure of the name of the disorder. The infant's blood pressure is 45/55 mm Hg and her pulse is 175/min. She appears ill. On physical examination, she has low muscle tone. Laboratory studies show no abnormalities, but genetic testing shows an inborn error in catecholamine synthesis. Which of the following is most likely deficient in this patient? A.Catechol-O-methyltransferase B.Dopamine -hydroxylase C.Homogentisate oxidase D.Tetrahydrobiopterin E.Tyrosinase F.Vitamin C

B.Dopamine -hydroxylase Dopamine -hydroxylase deficiency is an autosomal recessive mutation that results in the production of a non-functional dopamine -hydroxylase. There is decreased conversion of dopamine to norepinephrine and epinephrine which leads to a dysfunctional autonomic system. This results in hypotension and poor muscle tone. While it manifested in this patient at birth, it may also present later in life, usually as orthostatic hypotension. It may also be accompanied by ptosis, nasal congestion, and reduced exercise tolerance. Patients may experience vomiting, as dopamine has inhibitory signaling in the gut. The increased levels of dopamine can also suppress prolactin secretion, which would be apparent in pregnant women.

Which of the following additional laboratory findings would be expected in a patient with ulcerative colitis and alternating strictures and dilations in the bile ducts? A.Decreased ceruloplasmin B.Elevated alkaline phosphatase C.Elevated carbohydrate antigen 19-9 D.Positive antimitochondrial antibodies E.Positive cytoplasmic antineutrophil cytoplasmic antibodies

B.Elevated alkaline phosphatase The patient's symptoms of bloody diarrhea and cramping are consistent with a diagnosis of ulcerative colitis (UC), while the presence of fatigue, pruritus, and scleral icterus are suggestive of primary sclerosing cholangitis (PSC). Alkaline phosphatase is elevated in patients with PSC. PSC is characterized by inflammation, obliterative fibrosis, and segmental constriction of intrahepatic and extrahepatic bile ducts. On endoscopic retrograde cholangiopancreatography (ERCP) (a radiographic visualization of the pancreatic duct and biliary tree), these bile duct changes are viewed as alternating strictures and dilations, referred to as "beading" or "chain of lakes," like those shown in this image.

What is the morphology of influenza virus? A.Enveloped, double-stranded DNA B.Enveloped, negative-sense, single-stranded RNA virus C.Enveloped, positive-sense, single-stranded RNA D.Non-enveloped, double-stranded DNA E.Non-enveloped, positive-sense, single-stranded RNA F.Non-enveloped, single-stranded DNA

B.Enveloped, negative-sense, single-stranded RNA virus Enveloped, positive-sense, single-stranded RNA is the morphology of the Coronaviridae family of viruses, which includes common cold viruses. Non-enveloped, positive-sense, single-stranded RNA is the morphology of picornaviridae (rhinovirus), caliciviridae (norovirus), and hepeviridae (HEV) viral families. Non-enveloped, single-stranded DNA is the morphology of Parvovirus B19. Enveloped, double-stranded DNA is the morphology of herpesvirus, poxvirus (smallpox and molluscum contagiosum), and hepadnavirus (HBV). Non-enveloped, double-stranded DNA is the morphology of adenovirus, papillomavirus (HPV), and polyomavirus (JC, BK).

An 8-month-old boy is brought to the pediatrician by his parents because of general weakness. His parents state that he has progressively lost the ability to hold his toys and can no longer support his weight when crawling. On examination, the patient is tachypneic and breathing with considerable effort.The liver is palpable five finger widths below the right costal margin. The patient has a difficult time sitting upright, and he cannot squeeze the physician's fingers with any appreciable strength. A chest X-ray is performed and reveals an enlarged cardiac silhouette. Despite a number of interventions, the child's symptoms continue to worsen until his death 2 weeks later. This patient's disease is most likely caused by intracellular accumulation of which of the following substances? A.Glucose B.Glycogen C.Oxaloacetate D.Pyruvate E.Urea

B.Glycogen This patient presents with weakness, increased respiratory effort, an enlarged liver on exam, and a chest X-ray suggesting an enlarged cardiac contour. Based on his presentation and X-ray, he most likely has Pompe disease. This disease is also called glycogen storage disease type II (shown in the diagram at square II).

A 2-day-old boy has recurrent episodes of vomiting and failure to pass stool since birth. His mother describes the vomit as yellow to green in color. She notes that the boy was born with his heart on the right side of the chest but this was the only organ with an abnormal anatomic position. A follow-up echocardiogram performed on discovery of the heart position revealed no other cardiac abnormalities. On physical examination, the patient appears to be in severe discomfort and begins to cry when light pressure is applied to the abdomen. Chest and abdominal radiography reveal dilated loops of bowel with loss of haustra. Additionally, the bowel in the right upper-mid abdomen is massively engorged. The boy is immediately sent for surgical decompression. Which of the following vessels provides venous drainage of the affect region of intestine? A.Appendicular vein B.Ileocecal vein C.Middle colic vein D.Right colic vein E.Sigmoid veins

B.Ileocecal vein This presentation is typical of malrotation with midgut volvulus, specifically of the ileocecal junction. This is the second most common site for volvulus after the sigmoid colon. Midgut volvulus occurs due to abnormal rotation of bowel around its mesentery which compromises blood flow from the superior mesenteric artery (SMA). Venous drainage of the cecum and ileum occurs via the ileocecal vein which then drains into the superior mesenteric vein. Obstruction of blood flow to the bowel can cause bowel necrosis and perforation. Therefore, midgut volvulus should always be treated as a surgical emergency to avoid risk of complications, such as sepsis. Most cases of volvulus occur at the sigmoid colon which drains via the sigmoid veins. The appendix drains via the appendicular vein. The right colic vein drains the right ascending colon. The middle colic vein drains the transverse colon which is not found in the RLQ but instead the epigastric region.

An abdominal paracentesis yields bacteria that demonstrates resistance to gentamicin. Which of the following best describes the mechanism of resistance to the antibiotic in this patient? A.Amino acid modification of D-Ala-D-Ala to D-Ala-D-Lac B.Inactivation via enzyme modification C.Methylation of 23S rRNA-binding site D.Plasmid-encoded transport pumps E.Structural change in transpeptidases

B.Inactivation via enzyme modification The aminoglycoside class of antibiotics—including gentamicin, tobramycin, neomycin, amikacin, streptomycin, plazomicin, and paromomycin—are bactericidal. Aminoglycosides work by irreversibly binding to the 30S ribosomal subunit and inhibiting the initiation complex. This causes a misreading of mRNA and blocks translocation. Bacterial resistance to aminoglycosides commonly occurs due to transferase enzymes. This causes drug inactivation via modification (acetylation, phosphorylation, adenylation, etc.).

A 45-year-old woman is admitted to the hospital after being paralyzed from the waist down in a motor vehicle crash. She does not have intracranial bleeding, and she is placed on subcutaneous heparin for deep venous thrombosis prophylaxis. She is also prescribed oxycodone and acetaminophen as needed for pain. Her platelet levels during her hospital stay are:Days 1-4: 200,000/mm3Day 5: 150,000/mm3Day 6: 80,000/mm3Day 7: 76,000/mm3 If this patient continues her current treatment regimen, she is at increased risk of bleeding and which of the following immediate additional conditions? A.Ascending paralysis B.Intravascular thrombosis C.Jaundice D.Rhabdomyolysis E.Skin necrosis

B.Intravascular thrombosis This patient is taking heparin and develops thrombocytopenia. The most likely diagnosis is thrombocytopenia secondary to heparin administration. Heparin-induced thrombocytopenia (HIT) is an immunologic reaction to heparin, producing antibodies that cross-react with platelet factor 4, leading to platelet activation and clumping. HIT type 1 arises within a few days of treatment and is not an antibody-mediated response. Platelet levels usually do not drop below 100,000/mm3. HIT type 2 is due to new formation of heparin antibodies and usually manifests 4-5 days after starting heparin therapy. Platelet levels typically drop below 100,000/mm3. Because of the activation and clumping of platelets in these patients, heparin must be stopped immediately and a nonheparin anticoagulant (ie, argatroban) must be used to prevent arterial thromboses. Thromboses may be venous and/or arterial.

A 7-day-old male infant is brought to his pediatrician because of lethargy, vomiting, and difficulty feeding. On physical examination, he is difficult to arouse, does not respond to stimulation, and has poor muscle tone. He is admitted to the hospital because of failure to thrive. Shortly after admission, the infant has a seizure. A biochemistry evaluation of urine shows ketoaciduria; lymphocyte analysis shows deficiency in α-keto acid dehydrogenase. To prevent intellectual disability and death, intake of which of the following amino acids should be restricted in this patient? A.Alanine B.Leucine C.Phenylalanine D.Threonine E.Tryptophan F.Tyrosine

B.Leucine The patient is a 7-day-old infant with lethargy, seizures, and deficiency of branched chain α-keto acid dehydrogenase, all of which are consistent with maple syrup urine disease (MSUD).The infant appears normal at birth but in the first week the infant will present with lethargy, vomiting, encephalopathy, seizures, "maple syrup odor," and protein intolerance. This disease can be remembered by the mnemonic, "I love Vermont maple syrup from maple trees (with B1 branches)." This represents the blocked degradation of branched-amino acids (Isoleucine, Leucine, Valine).

A 45-year-old white woman comes to the rheumatology clinic for evaluation. She was diagnosed with rheumatoid arthritis 10 years ago and was able to manage her symptoms with nonsteroidal anti-inflammatory drugs (NSAIDs) alone. However, several months ago, she began missing more days of work because of worsening symptoms of joint pain, swelling, and morning stiffness. She was then prescribed a new medication by her rheumatologist, which led to significant reduction in her arthritis symptoms. At today's visit, she reports nausea, fatigue, and thinning of her hair. Results of a complete blood count show:Hemoglobin: 10.2 g/dLMean corpuscular volume: 102 μm3Leukocyte count: 4600/mm3Platelet count: 150,000/mm3 Which of the following treatments is most appropriate for this patient? A.Iron supplementation B.Leucovorin C.Mannitol diuresis D.Vitamin B12 E.Vitamin B6

B.Leucovorin Leucovorin (folinic acid, the active metabolite of folic acid) helps replenish the body's endogenous supply of folic acid. Use of leucovorin can effectively inhibit the myelosuppressive effects of methotrexate.

In the mid-1990s, the Food and Drug Administration mandated the supplementation of grain products with a certain compound, in an attempt to reduce the incidence of a specific category of birth defects. This compound is a coenzyme for 1-carbon transfers and is absorbed in the jejunum. This compound exerts its greatest effect on which of the following areas of the developing embryo? A.Midgut B.Neural folds C.Notochord D.Urogenital sinus

B.Neural folds The U.S. Food and Drug Administration mandated in 1998 that cereal grain products be fortified with folic acid to decrease neural tube defects, since only one in three pregnant women were consuming enough folic acid in their diet at that time. The recommended dosage of folate in pregnancy is 4 mg daily and the supplementation should be started prior to conception, as closure of neural folds occurs very early in pregnancy, roughly around 4 weeks.

A 50-year-old black woman comes to the clinic because of knee pain began 2 months ago. She works as a nurse and is on her feet most of the day. The pain improves with rest but worsens with activity throughout the day. The patient achieves relief with ibuprofen. She has a history of hypothyroidism and she takes levothyroxine daily. Her temperature is 98.6° F (37° C), blood pressure is 128/86 mm Hg, pulse is 86/min, respiratory rate is 15/min, and BMI is 36. X-ray of the knee is shown. Which of the following is the greatest risk factor for osteoarthritis seen in this patient? A.Age B.Obesity C.Occupation D.Race E.Sex

B.Obesity This 50 year-old patient has a 2-month history of joint pain that worsens with activity. Her job keeps her on her feet for much of the day. Her BMI is 36, which would classify her as obese. Though the patient's sex, race, and age all increase her risk of osteoarthritis, her greatest risk factor is obesity. Obesity is a major risk factor for osteoarthritis because the joints are carrying extra weight and stress; arthritis is accelerated and can have an earlier onset. Sex, race, and age are all non-modifiable risk factors for osteoarthritis. Women overall are more likely to develop osteoarthritis than men. Black, Native American, and non-white Hispanic women are more likely to develop osteoarthritis as compared with white women. Age is a non-modifiable risk factor for osteoarthritis, usually seen in patients older than age 50 years. Her occupation as a nurse may help prevent progressive bone density loss, as physical activity is beneficial.

Which of the following is the underlying mechanism generating slow waves within the gastrointestinal tract? A.Alternating contraction of circular and longitudinal muscle B.Contraction of skeletal muscle C.Depolarization and repolarization of smooth muscle cells D.Hyperpolarization of smooth muscle cells E.Increase in secretions of smooth muscle cells F.Relaxation of smooth muscle cells

C.Depolarization and repolarization of smooth muscle cells Slow waves are precisely timed, rhythmic depolarizations and repolarizations of the muscularis propria of the stomach and intestines, independent of the presence or absence of stimulus. They move in an oral-to-anal direction and occur at a frequency specific to each organ, with the stomach having the lowest frequency (3 cycles per minute) and the duodenum of the small intestine having the highest frequency (12 cycles per minute). They represent the basal electric rhythm of gastric and intestinal motility, although they don't always result in mechanical activation.

A 17-year-old girl is brought to her family physician by her mother. Two days ago, the patient noticed slight twitching in her arms and legs. These movements are uncontrollable and have progressively worsened since then. Four weeks ago, the patient had a mild febrile illness that was left untreated. She says she has noticed that her urine appears brown and frothy every time she goes to the bathroom. The patient is not sexually active, has no recent travel history, and has not been injured. Her temperature is 37°C (98.6°F), pulse is 88/min, respirations are 14/min, and blood pressure is 118/78 mm Hg. Physical examination reveals a patient alert and oriented ×3, pupils equally reactive to light and accommodation, and 2+ reflexes present in all extremities. Cardiac examination reveals no murmurs, rubs, or gallops. Findings on neurologic examination are within normal limits, and cranial nerves are globally intact. The thyroid is nontender and nonpalpable. The patient says she has not noticed any recent weight loss. Laboratory studies show liver function and the blood urea nitrogen concentration to be within normal ranges. Which of the following is the most likely cause of this patient's condition? A.Elevated blood ammonia concentration B.Generalized seizure disorder C.Hypersensitivity cross-reaction D.Substance intoxication E.Subthalamic lesion F.Triple repeat expansion

C.Hypersensitivity cross-reaction This 17-year-old patient's history of an untreated illness and current production of frothy, dark urine should raise suspicion for glomerular disease. One common cause of postinfectious glomerulonephritis is infection with β-hemolytic streptococci, such as Streptococcus pyogenes. In addition to causing nephritic syndrome, S. pyogenes can also cause acute rheumatic fever if left untreated. Although rheumatic fever is a well-known cause of pancarditis, it can also induce Sydenham chorea, a restless, uncontrollable movement of the extremities. Poststreptococcal glomerulonephritis and acute rheumatic fever are immune-mediated type II hypersensitivity reactions, resulting in an M protein cross- reaction with host antigens (molecular mimicry). Although treatment with penicillin at the time of infection greatly reduces the risk of developing acute rheumatic fever, it does not prevent poststreptococcal glomerulonephritis sequelae.

A 60-year-old man presents to the clinic because he has been feeling unwell for the past week. His symptoms include difficulty falling asleep, a productive cough,and drenching night sweats. Past medical history is significant for severe persistent asthma, which is poorly controlled on high-dose oral glucocorticoids. A chart review reveals a 10-pound weight loss since his last office visit 3 months ago. Vital signs are temperature 100.8°F (38.2°C), blood pressure 98/62 mm Hg, pulse 90/min, respiratory rate 22/min, and SpO2 90% on two liters of O2. A CT scan of the chest is obtained and shows abscesses in the lungs. A purified protein derivative (PPD) skin test is negative. An acid-fast stain of the patient's sputum is revealing. What is the mechanism of action of the drug of choice for treating this infection? A.Binds ergosterol and forms membrane pores B.Decreases synthesis of mycolic acids C.Inhibition of dihydropteroate synthase D.Inhibits formation of the initiation complex by binding to the 30S ribosomal subunit and causing misreading of mRNA E.Inhibits peptidoglycan cross-linking to block bacterial cell wall synthesis

C.Inhibition of dihydropteroate synthase This patient's symptoms of difficulty sleeping, a productive cough, fever, and drenching night sweats, together with his recent weight loss and a history of high-dose oral glucocorticoid therapy for his persistent asthma, raises the suspicion for infection with Nocardia asteroides. Nocardia infections are known to present in immunocompromised patients, such as those on chronic glucocorticoid therapy. Additional symptoms of this infection can include fatigue, anorexia, dyspnea, cough, hemoptysis, and pleuritic chest pain. Nocardia is a gram-positive, weakly acid-fast bacterium that forms long branching filaments, as seen in acid-fast stain of this patient's sputum sample. The findings on CT support the diagnosis of pulmonary nocardiosis, which typically manifests with a single or multiple nodules, lung masses (with or without cavitation), interstitial infiltrates, lobar consolidation, or pleural effusions. Because of its acid-fast nature and its similar disease process, pulmonary nocardiosis is frequently (initially) mistaken for tuberculosis. Sulfonamides (TMP-SMX) are the treatment of choice for pulmonary nocardiosis; these agents act by blocking nucleotide synthesis by inhibiting dihydropteroate synthase.

A 69-year-old woman comes to orthopedic clinic for knee pain. When the front office staff asks her for information regarding her insurance, she states that she "doesn't remember" but that her hospital visits are "free," and that she pays a "really low monthly premium" for her doctors visits and medications. She also states that her insurance plan does not require a referral from a primary care provider (PCP). The patient is a retired schoolteacher. This patient most likely has coverage under which of the following insurance plans? A.Employer-sponsored insurance B.Medicaid C.Medicare D.No insurance plan E.Private individual health plan

C.Medicare Medicare part A is free, and there are typically low premiums for other parts, such as in this patient's case. She also said she is not required to have a referral from a PCP, which indicates that she has a Preferred Provider Organization (PPO) plan. PPO plans have network physicians, other health care providers and hospitals, and do not require referral from PCP to see a specialist. Medicare plan C has a PPO option, making it her most likely insurance. Medicaid is only available for patients who are low-income, pregnant women, or patients with disabilities; it is unlikely that this patient qualifies. Employee-sponsored insurance is not the best answer as the patient is retired and would qualify for Medicare at her age. A private individual health plan is also unlikely given cheaper options through Medicare. No insurance plan is incorrect because the patient does give some detail about her insurance coverage, despite not remembering what it is called.

Which of the following features is most likely to be associated with Lynch syndrome? A.Congenital mutation of APC gene B.Congenital p53 mutation C.Mismatch repair defects resulting in increased mutagenesis D.Multifocal lesions E.Polyploid lesions F.Well-differentiated histology

C.Mismatch repair defects resulting in increased mutagenesis

A 2-year-old black boy has required frequent transfusions throughout his life because of anemia. A peripheral blood smear shows microcytic, hypochromic red cells with target cells and anisopoikilocytosis. After further genetic testing, the child's spleen is removed to lessen the need for transfusions. A sample of the resected spleen is analyzed for genetic testing. The suspected gene is sequenced, and the patient is found to have the mutation shown below. Normal 5' UUCGUUCCGACU 3' Mutant 5' UUCAUUCCGACU 3' The genetic mutation is most likely affecting which of the following genetic processes? A.Capping B.Polyadenylation C.Splicing D.Transcription E.Translation

C.Splicing Introns are non-coding regions of RNA that are spliced out of pre-mRNA. This is achieved by utilizing the spliceosome, which recognizes splice sites as a 5' GU --- AG 3' sequence. As the mutant shown in the question changes from the normal 5′-GU to a 5′-AU, the intron is prevented from being properly spliced at the appropriate site. This type of mutation can cause beta-thalassemia.

How to distinguish CMV pneumonitis and Pneumocystis jirovecii?

CMV pneumonitis usually occurs 1-4 months after transplantation. P. jirovecii primarily affects patients with HIV infection who have a low CD4 count; recipients of hematopoietic cell and solid organ transplants; patients with cancer; and patients receiving glucocorticoids, chemotherapeutic agents, or other immunosuppressive medications. This patient is a solid organ transplant recipient and is currently taking cyclosporine, azathioprine, and prednisone for immunosuppressive therapy, putting him at risk for infection with P. jirovecii.

A child is able to stack 6 blocks, kick a ball, and play alongside other children in parallel play. He is unable to dress himself and is not toilet trained. He can speak in 2-word sentences. Based on these activities, how old is the child? A.5 months old B.9 months old C.12 months old D.2 years old E.3 years old.

D.2 years old

Which of the following will most likely occur in the COPD patient as a result of mechanical ventilation? A.Decreased alveolar ventilation B.Decreased gas exchange C.Decreased oxygenation of tissue D.Decreased work of breathing E.Increased work of breathing

D.Decreased work of breathing Mechanical ventilation is assisting ventilation via a machine that forces air into the airway (positive pressure). This can be invasive (eg, intubation) or noninvasive (eg, bilevel positive airway pressure [BiPAP] or continuous positive airway pressure [CPAP]). Indications for initiating mechanical ventilation include apnea, clinical signs of increased work of breathing (eg, nasal flaring, eyelid retraction, accessory muscle recruitment, expiratory muscle recruitment, tracheal tug, intercostal recession, tachypnea, tachycardia, diaphoresis, change in mental status), or inability to protect the airway.

The patient, who had a recent episode of mononucleosis, is presenting with increased reticulocyte count, scleral icterus, decreased hemoglobin with pain and discoloration of the extremities. Which of the following is the most likely diagnosis? A.Aplastic anemia B.Disseminated intravascular coagulation C.IgG-mediated hemolytic anemia D.IgM-mediated hemolytic anemia E.Paroxysmal nocturnal hemoglobinuria

D.IgM-mediated hemolytic anemia The patient, who had a recent episode of mononucleosis, is presenting with signs of hemolytic anemia (eg, increased reticulocyte count, scleral icterus, decreased hemoglobin) with pain and discoloration of the extremities. The patient's fatigue and lethargy are likely due to IgM-mediated (cold) autoimmune hemolytic anemia. This cold anemia may manifest soon after Epstein-Barr virus infection, also known as infectious mononucleosis (IM), or mono. It may also occur in the acute stage of HIV, in Mycoplasma pneumoniae infection, and in leukemia. Agglutination of red blood cells in the periphery may result in painful gray or purple discoloration of the fingers caused by cold exposure, as seen in this patient. A patient with chronic cold agglutinin disease will have more symptoms during the colder months.

Which of the following is the mechanism of the new oral anticoagulant most likely used for the long-term therapy of afib? A.Activation of antithrombin III B.Antibody to tumor necrosis factor-α C.Inhibition of cyclooxygenase D.Inhibition of factor Xa E.Inhibition of the ADP pathway of thrombus formation

D.Inhibition of factor Xa AF places the patient at higher risks for mural thrombosis and embolic complications. Direct factor Xa inhibitors (rivaroxaban, apixaban, and edoxaban) or dabigatran (a direct thrombin inhibitor) are the preferred anticoagulants in the setting of AF. They are taken orally and do not require weekly monitoring like warfarin does. A helpful way to remember the mechanism of action of these agents is that rivaroXaban and apiXaban ban Xa.

A 67-year-old man is brought to the emergency department because of chest pain. The pain started 3 hours ago when he was shoveling snow and resolved spontaneously on the ambulance ride to the hospital. He describes the chest pain as "squeezing" rather than sharp and denies any radiation. He denies any similar episodes in the past. He admits to having just finished eating a heavy meal (hamburger and fries) before shoveling snow. Medical history includes hyperlipidemia, hypertension, and gastroesophageal reflux disease. Medications include rosuvastatin, lisinopril, and pantoprazole. He was otherwise healthy besides an upper respiratory infection 1 week ago. On physical examination, no acute distress is noted. Temperature is 37.1°C (98.8° F), blood pressure is 160/80 mm Hg, and pulse is 88/min. Heart rate is regular without murmurs, rubs, or gallops. There is no jugular venous distension. Lungs are clear to auscultation without crackles or wheezes. There is no pedal edema. Results of a creatine kinase and troponin panel is within normal limits. An EKG shows ST elevation on V2-V4. Which of the following is most likely responsible for this patient's condition? A.Coronary artery spasm B.Esophageal reflux of acid C.Ischemia leading to myocyte necrosis D.Lipid plaque formation in coronary artery E.Non-peristaltic contractions of esophagus F.Obstruction of cystic duct G.Pericardial inflammation H.Tear in aortic intima secondary to high-blood pressure

D.Lipid plaque formation in coronary artery The most likely cause of rapidly resolving exertional chest pain in a patient with a history of hypertension and hyperlipidemia is stable angina secondary to coronary atherosclerosis. This patient's normal EKG and undetectable cardiac biomarkers (troponins and creatine kinase-mb) further support this diagnosis. A brief understanding of the pathogenesis of atherosclerosis is important for the USMLE examination. The pertinent risk factors for atherosclerosis (eg, hypertension, hyperlipidemia) can cause endothelial cell injury, which is the first step of atherosclerosis. Endothelial injury results in increased vessel permeability, monocyte adhesion/migration, and smooth muscle proliferation, all of which contribute to the formation of arterial plaques. These plaques prevent normal blood flow and result in downstream ischemia, especially during times of increased cardiac demand (ie, stable angina). Coronary artery disease can be treated with beta-blockers, nitrates, or calcium channel blockers, all of which decrease myocardial oxygen demand (by decreasing cardiac work) and, to a lesser extent, increase coronary artery blood flow. Coronary artery spasm occurs in Prinzmetal angina, which presents with chest pain that is unrelated to activity and episodic. An EKG typically shows ST-elevations during an episode of pain. This patient's exertional pain and cardiac risk factors suggest a different diagnosis.

A 28-year-old woman comes to the dermatologist because she has noticed a raised pigmented lesion on her left leg. She is concerned about it because her father was recently diagnosed with skin cancer. She denies any recent trauma or burn injuries to the left leg. Her vital signs are within normal limits. On physical examination, a 4 × 8 mm pinkish tan lesion with irregular borders is seen. A biopsy specimen of the lesion shows atypical melanocytes arranged in nests and as single cells at the dermoepidermal junction but with no continuous growth or spread of melanocytes into the upper epidermis. Which of the following is the most likely complication associated with this patient's skin condition? A.Acanthosis nigricans B.Basal cell carcinoma C.Marjolin ulcer D.Melanoma E.Seborrheic keratosis F.Squamous cell carcinoma

D.Melanoma Biopsy of the lesion reveals atypical melanocytes arranged in nests and as single cells at the dermoepidermal junction but with no continuous growth or spread of melanocytes into the upper epidermis. These findings suggest that this patient's lesion is a dysplastic nevus. A dysplastic nevus is a benign growth that resembles malignant melanoma. Patients who have multiple dysplastic nevi are especially at risk, and, although it is rare, a dysplastic nevus can undergo malignant transformation. Risk factors for the development of dysplastic nevus include: Genetics (parents with moles) Fair skin Intense sun exposure during childhood

A 46-year-old man comes to his physician because of abdominal distention. On physical examination, auscultation reveals diffuse hyperactive bowel sounds across the abdomen. He has mild tenderness to palpation in his lower right and lower left abdominal quadrants. The patient reveals that many of his relatives have had colon cancer, and they were diagnosed in their forties. An x-ray, followed by a contrast enema, yields distinctive findings, and surgery to remove a portion of the large intestine is recommended. The gross specimen shown in the image reveals the cause of the patient's abdominal distention—a circumferential tumor with central ulceration that is narrowing the colonic lumen. Which of the following mechanisms is likely defective in this patient? A.APC-mediated tumor suppression B.Base excision repair C.Double-strand break repair D.Mismatch repair E.Pyrimidine dimer repair

D.Mismatch repair This patient's abdominal distention, hyperactive bowel sounds, and tenderness on palpation should raise the suspicion of a bowel obstruction caused by an abdominal tumor. In light of his relative youth and the frequency of cancer in his close relatives, this patient most likely has hereditary nonpolyposis colon cancer (HNPCC), also called Lynch syndrome. HNPCC is an autosomal dominant condition and an example of the "two-hit hypothesis": the first mutation in the mismatch repair mechanism is inherited, and the second mutation is acquired spontaneously later in life. The process of mismatch repair is defective in patients with this condition.

A 35-year-old man with a history of ulcerative colitis comes to his primary care physician because of a 1-month history of increasing blood in his stools. He denies any other symptoms of a current ulcerative colitis flare. He has no pain during bowel movements and has not had abdominal pain. He was diagnosed with ulcerative colitis20 years ago and had poor control of his disease until the past 2 years, when he started mesalamine and infliximab. His family history is significant for a mother who died of colon cancer at age 50. On evaluation today he is afebrile. His blood pressure is 110/80 mm Hg, pulse is 95/min, and respirations are 18/min. His physical examination is unremarkable. On rectal examination, there is no evidence of fissures or hemorrhoids. Fecal occult blood test is positive. Blood work reveals a hemoglobin of 9 mg/dL. Colonoscopy confirms diagnosis of colorectal carcinoma. Which of the following features is most likely to be associated with this patient's colorectal malignancy? A.Congenital mutation of APC gene B.Congenital p53 mutation C.Mismatch repair defects resulting in increased mutagenesis D.Multifocal lesions E.Polyploid lesions F.Well-differentiated histology

D.Multifocal lesions Given his history of UC, this patient most likely has developed colitis-associated colorectal cancer (CRC). Chronic inflammation is a major risk for all types of cancer due persistent proinflammatory cytokine and chemokine production. This leads to dysplasia, driven by oxidative stress, and increased cell turnover, resulting in genetic and epigenetic changes. Poorly controlled UC results in extensive inflammation throughout the colon, leading to increased risk of CRC. Colitis-associated CRC has an increased risk of multifocal lesions (due to widespread inflammation) as compared to sporadic CRC.Colitis-associated tumors typically originate in areas that have been impacted by colitis.

A 16-year-old girl comes to the office because she has never had a menstrual period. On physical examination, the physician notes mild clitoromegaly and several dark hairs on her upper lip and chin. There is no evidence of increased muscle mass, and other examination findings are within normal limits. Sexual development is Tanner stage II, and her BMI is 20. A serum hormone panel shows elevated testosterone, low luteinizing hormone, and low follicle-stimulating hormone levels. Ultrasonography reveals a normal uterus and ovaries. The patient had a phenotypically normal female appearance at birth. Both her mother and sister had menarche at age 12 and now have regular cycles. Which of the following is the most likely diagnosis? A.Androgen insensitivity syndrome B.Androgen-producing tumor C.Early-onset polycystic ovary syndrome D.Non-classic congenital adrenal hyperplasia E.Phenotypic progression to male upon puberty

D.Non-classic congenital adrenal hyperplasia This teen-aged girl with primary amenorrhea, hirsutism, mild clitoromegaly, and Tanner sexual development incongruent with age likely has non-classic congenital adrenal hyperplasia (CAH). This diagnosis is further reinforced by her elevated testosterone levels and unremarkable abdominal ultrasonography findings. CAH is most commonly caused by congenital defects in the 21-hydroxylase enzyme, which catalyzes the conversion of 17-hydroxyprogesterone to 11-deoxycortisol in the cortisol synthesis pathway and catalyzes the conversion of progesterone to deoxycorticosterone in the aldosterone synthesis pathway. Androgen-producing tumors are characterized by rapid progression of symptoms caused by increased testosterone (abrupt increase in body hair, muscle mass, etc). In addition, such tumors most often present in the ovaries rather than the adrenals. This patient has only mild hirsutism without increased muscle mass,and sonography shows normal ovaries.

A 76-year-old man visits his physician because he is afraid of catching an illness that is going around his nursing home. His roommate and friends at the nursing home have been complaining of a runny nose, a dry cough, headache, and sore muscles. He has a history of well-controlled hypertension and diabetes and suffered a left-sided stroke 3 years ago that caused impaired mobility. He has received this year's flu vaccine and received a vaccine against Streptococcus pneumoniae at age 65. The physician prescribes a prophylactic agent. Which of the following best describes the mechanism of the drug used as prophylaxis for this patient? A.Blocks late protein synthesis and processing B.Blocks viral adsorption to the cell C.Prevents early protein synthesis D.Prevents the release of viral progeny from infected cells E.Prevents viral uncoating

D.Prevents the release of viral progeny from infected cells Neuraminidase inhibitors can be used as prophylaxis against influenza A and B in high-risk patients. These drugs act by blocking the normal function of viral neuraminidase enzymes and preventing the release of viral progeny from infected cells. The most common neuraminidase inhibitors are oseltamivir and zanamivir. Amantadine binds to the M2 protein on the influenza A virus only, thus preventing viral uncoating. Amantadine has historically been used in the treatment and prophylaxis of influenza A infection. However, 90% of all influenza A strains are now resistant to amantadine, so it is no longer typically used for these purposes in the United States. Amantadine is still used to treat symptoms of Parkinson disease.

A 43-year-old woman comes to the physician because of fatigue and yellowing skin. This is her third episode of such symptoms over the course of 2 years, and she says each episode has been getting progressively worse. She also complains of upper abdominal discomfort and anorexia. Her medical history is significant only for nonspecific joint pain. She reports having had more than 20 sexual partners in her life. Physical examination reveals a distended abdomen with fluid wave and edematous lower extremities. A right upper quadrant ultrasound reveals no stones or pericholecystic fluid. Laboratory test results are as follows: AST: 800 U/LALT: 760 U/L Total bilirubin: 8.5 mmol/L Direct bilirubin: 7.0 mmol/L GGT: 32 U/LAlkaline phosphatase: 112 U/L Antimitochondrial and anti-smooth muscle antibodies: Negative Which of the following is the most appropriate first-line therapy? A.Ganciclovir B.HAART C.Surgical resection D.Tenofovir disoproxil fumarate E.Ursodeoxycholic acid

D.Tenofovir disoproxil fumarate Given her history of numerous sexual partners and joint pain, the likely etiology is chronic hepatitis B infection. Hepatitis B is transmitted through blood and bodily fluids. Many people with chronic infections will remain free of symptoms, but 30%-40% will eventually develop cirrhosis. This patient's symptoms are classic for chronic hepatitis infection with acute exacerbations and impending liver failure.First-line therapy is tenofovir disoproxil fumarate, a nucleotide reverse-transcriptase inhibitor.

A 17-year-old girl is brought to the physician by her mother, who reports that her daughter has been experiencing a severe cough and spiking fevers for the past 3 days. The cough produces a moderate amount of yellow-green sputum. The girl's history is significant for chronic diarrhea, persistent wheezing, previous similar infections, and consistently poor growth. There is no history of recent travel and no known new contacts. Her temperature is 38.8°C (101.9°F), pulse is 128/min, respirations are 40/min, and blood pressure is 90/60 mm Hg. A sputum sample is obtained for culture, and the culture shows growth of pseudomonal organisms. Which of the following antibiotics would treat the bacteria that most likely grew on her sputum culture? A.Amoxicillin B.Ceftriaxone C.Linezolid D.Tobramycin E.Vancomycin

D.Tobramycin The other antibiotics are incorrect because they are not indicated in the treatment of Pseudomonas infections. Amoxicillin is indicated for many respiratory infections and methicillin-resistant Staphylococcus aureus (MRSA). Vancomycin is indicated to treat MRSA. Linezolid is commonly used to treat MRSA and vancomycin-resistant enterococci. Ceftriaxone is indicated for gram-positive bacteria and some gram-negative bacteria.

A 1-day-old female neonate is being evaluated because of cyanosis. She was born by uncomplicated, spontaneous vaginal delivery at term to a 23-year-old primigravid woman. The mother received routine prenatal care throughout the pregnancy. The infant's hands and feet have a bluish hue and are cool to touch. The child is tachycardic, breathing rapidly, and is grunting. The blood pressure is on the low end of normal. Physical examination reveals an active ventricle with a loud P2 and a continuous machine-like murmur. Breath sounds are normal. Follow-up echocardiography reveals no structural heart disease. Which of the following is the most appropriate first-line pharmacologic treatment? A.Amlodipine B.Corticosteroids C.Epoprostenol D.Indomethacin E.Inhaled nitric oxide F.Sildenafil

E.Inhaled nitric oxide Persistent pulmonary hypertension of the newborn (PPHN) is the most likely diagnosis. The presence of acrocyanosis suggests that the infant is hypoxemic and has a right to left shunt. The breath sounds are usually normal as the alveoli remain patent and continue to allow gas exchange. The increased pulmonary vascular resistance causes delayed closure of the pulmonic valve auscultated as a loud, split S2. Pulmonary arterial blood is then shunted through the PDA into the systemic vasculature without being oxygenated. Infants born at term or >34 weeks are at risk for PPHN and the cause is idiopathic. Treatment of PPHN first involves mask ventilation with positive end-expiratory pressure (PEEP) along with surfactant administration and oxygen. First line pharmacotherapy is with inhaled nitric oxide (iNO) used to dilate the pulmonary vasculature and promote blood flow through the lungs. Nitric oxide acts by stimulating guanylyl cyclase to increase production of cGMP, which aids in smooth muscle relaxation. The resulting dilation of the pulmonary vasculature increases blood flow through the lungs. This results in decreased right ventricle pressure and improved blood oxygenation. Indomethacin is an NSAID used to close a patent ductus arteriosus. Although a PDA is identified in this patient, it is not the cause of disease and should not be treated at this time.

A 42-year-old woman comes to her primary care physician because of difficulty losing weight. She states that she has been overweight since childhood. She has tried various diets and exercise programs to lose weight and has had no success. She is 163 cm (5'4") tall and weighs 90.7 kg (200 lb); BMI is 34 kg/m2. The physician discusses prescribing a non-anorexiant medication as adjuvant pharmacologic treatment. Which of the following medications is most appropriate for this patient? A.Docusate B.Liraglutide C.Lovastatin D.Octreotide E.Orlistat F.Phentermine

E.Orlistat Orlistat works by inhibiting gastric and pancreatic lipase, causing the inhibition of the hydrolyzation of fats. This causes an increase in fecal fat excretion. Orlistat decreases the breakdown and absorption of fats and works best when taken with fatty meals. It is used clinically for weight loss in patients with a BMI >30 kg/m2 or BMI > 27 kg/m2 with comorbidities. There are numerous adverse effects associated with orlistat. These adverse effects include abdominal pain, flatulence, steatorrhea, bowel urgency, increased bowel frequency, and decreased absorption of fat soluble vitamins.

A 61-year-old man presents with chronic cough, difficulty swallowing, and new-onset voice change. In recent months he has experienced fatigue and an unintentional 20-lb (9.07-kg) weight loss. He also complains of postprandial indigestion, especially while lying down. He is a retired rock singer, and he admits to smoking one pack of cigarettes per day for the past 40 years and drinking "more than his share" of alcohol. In addition, he reports unprotected sex with multiple partners in the past. The patient is found to have a mass arising from the left true vocal fold, as shown in the image. Biopsy confirms the presence of a neoplasticprocess. Which of the following risk factors is most closely associated with the development of this patient's condition? A.Chronic excessive alcohol consumption B.Chronic irritation from professional singing C.Gastroesophageal reflux disease D.Infection with human papillomavirus types 6 and 11 E.Smoking history F.Untreated Helicobacter pylori infection

E.Smoking history The biopsy shows laryngeal carcinoma and is supported by his symptoms of new-onset voice changes. The two most frequently identified modifiable risk factors for laryngeal carcinoma are smoking and alcohol use. Virtually all patients with this disease have a history of tobacco use, and tobacco use is the strongest risk factor for head and neck cancers (including cancers of the larynx and hypopharynx). In the evolution of the carcinoma, the squamous epithelium of the larynx transitions from a hyperplastic state to carcinoma in situ (shown in the image in the vignette) and then to invasive carcinoma.

A researcher is studying oxygen dissociation from hemoglobin in cold temperatures. He cools erythrocytes to 20° C (68° F) and measures their affinity for oxygen. The results are plotted on the graph shown. He finds that cooling of erythrocytes decreases the amount of oxygen released. Which of the following situations would most likely shift the hemoglobin dissociation curve to the left? A.Decrease in fetal hemoglobin B.Increase in 2,3-BPG C.Increase in CO2 concentration D.Increase in H+ concentration E.Increased concentration of inositol phosphate F.Increased concentration of methemoglobin

F.Increased concentration of methemoglobin Nitrate poisoningcauses increased formation of methemoglobin. Methemoglobin is a form of hemoglobin that contains the ferric form of iron (Fe3+). Ferric oxygen has a reduced affinity for oxygen. In addition, binding of oxygen to methemoglobin increases affinity for oxygen in another heme sites. This increased affinity for oxygen results in an oxygen dissociation curve shift to the left.

A 56-year-old man comes for evaluation because of blurred vision while reading. He has never worn glasses, but has noticed that his vision has been progressively worsening during the past few years. He denies any trauma to his eyes or head. The patient's medical history is remarkable only for hypertension, for which he takes hydrochlorothiazide. A Snellen test performed at 20 feet away shows visual acuity of 20/20 in both eyes. Fundoscopic examination reveals no papilledema, no pathologic cupping of the optic disc, no hemorrhages, and no exudates. Which of the following most likely explains the patient's blurry vision? A.Astigmatism B.Hyperopia C.Hypertensive retinopathy D.Myopia E.Open-angle glaucoma F.Presbyopia

F.Presbyopia This patient has blurred vision at a close distance from his reading matter and progressive worsening of vision, despite never needing to wear glasses in the past. Fundoscopic examination reveals no abnormalities and Snellen test shows a visual acuity of 20/20, suggesting this patient is suffering from presbyopia, a non-refractive error that occurs with age. Presbyopia is the normal loss of the accommodating power of the lens, and is characterized by difficulty focusing on objects viewed at arm's length or closer. In astigmatism, a warped corneal surface causes light rays entering the eye along different planes to be focused unevenly. This results in blurry vision at all viewing distances. It can be corrected with spectacles containing a cylindrical optical surface. This patient with a non-refractive error is experiencing blurred vision only when focusing on objects nearby or at arms length.

This patient is a middle-aged obese G2P2 female with a likely history of biliary colic (recurrent postprandial RUQ pain) with acute epigastric pain radiating to the back in the setting of fever, hypotension, tachycardia and elevated ALT, AST and lipase. CT scan shows a diffusely edematous pancreas with fat stranding. Overall, presentation is characteristic of gallstone pancreatitis. Formation of which of the following enzymes is the greatest contributor to this patient's symptoms? A.Amylase B.Elastase C.Enterokinase D.Lactase E.Pepsinogen F.Trypsin

F.Trypsin Zymogens are enzymes that are excreted in an inactivated state, but which are activated by other enzymes once they reach their location of action. Pancreatic enzymes are zymogens that are activated in the small intestine. Trypsinogen is converted to trypsin by brush border enzymes such as enterokinase. Trypsin then converts the other pancreatic zymogens to active enzymes and digests dietary protein. In pancreatitis, trypsinogen becomes inappropriately converted to trypsin which activates the other zymogens. As these active enzymes do not have dietary components to digest, they start to autodigest the pancreas, which can lead to necrosis. Pancreatic autolysis results in the massive systemic inflammatory response and pain seen in pancreatitis. Amylase is a pancreatic (and salivary) enzyme that digests carbohydrates. It is excreted in its active form and does not play a major role in autolytic digestion of the pancreas in pancreatitis.

A 5-year-old boy is brought to the clinic by his mother due to excessive vomiting and sweating. His symptoms began soon after his mother found him eating wild mushrooms. The patient is afebrile; blood pressure is 80/45 mm Hg, pulse is 58/min, and respiratory rate is 16/min. On physical examination, the patient is abnormally drowsy, sweating heavily, and salivating. Which of the following additional findings is associated with activation of the most likely receptor targeted by this toxin? A.Bronchodilation B.Decreased gastrointestinal motility C.Lipolysis D.Mydriasis E.Urinary retention F.Vasodilation

F.Vasodilation Activation of M3 receptors on endothelial cells results in nitric oxide (NO) production and relaxation of smooth muscle cells, which leads to vasodilation. Lipolysis can occur from beta-3 adrenergic activation, which is a type of sympathetic activation. However, this patient's symptoms are highly suggestive of parasympathetic activation of M1-M3 receptors.

Difference of PPSV23 and PCV13?

PCV13 is a conjugate vaccine that consists of capsular polysaccharides from 13 serotypes of S. pneumoniae covalently attached to inactivated diphtheria toxin (protein). B-cells endocytose polysaccharide-protein-conjugate vaccine and display protein component of vaccine on major histocompatibility complex class II (MHCII), which can subsequently be recognized by T-cell receptor that has been activated by the same protein. This process of T-cell-dependent activation induces a strong immune response involving T-cell-mediated B-lymphocyte activation, which generates higher-affinity antibodies and memory cells. T-cell-mediated B-lymphocyte activation also enables switching, which allows for the formation of IgA mucosal antibodies that reduce bacterial colonization rates. All individuals older than 65 years are advised to receive the pneumococcal conjugate vaccine (PCV13) followed by the polysaccharide vaccine (PPSV23) Individuals younger than 65 years with certain risk factors are also advised to receive PCV13 and PPSV23 (e.g., individuals who smoke, have chronic cardiovascularl disease, cirrhosis, or other immunosuppressive conditions) PPSV23 is a polysaccharide vaccine that contains capsular material from 23 S. pneumoniae serotypes. Because these polysaccharides are not conjugated to protein, they cannot be displayed to T-cell-receptors by MHCII. B-lymphocyte activation is therefore independent of T cells. This T-cell-independent immune response leads to more moderate antibody production that is effective for most patients but is ineffective for patients younger than 2 years of age who have yet to develop mature T-independent B cell activation systems.

A 64-year-old man presents to his primary care physician for an examination. This is his first wellness visit in 10 years. During his visit, he informs his physician of a recent 20-lb (9-kg) unintentional weight loss and pain in his lower back and pelvis. He has smoked two packs of cigarettes per day for the past 30 years, and reports unprotected sexual intercourse with multiple partners in the past year within his retirement community. Physical examination is notable for point tenderness along his spine and pelvis. On digital rectal examination, the prostate is found to have a generalized firmness, nodules, and an unusual shape. Which of the following laboratory values are expected in this patient? [incr, decr] PSA, Calcium, Alkaline phosphatase

PSA incr, Ca decr, ALK incr Metastases, from the prostate to the bone, disrupt the normal resorption process of bone formation, leading to a decreased serum calcium level. The prostate-specific antigen (PSA) level is typically elevated in prostate cancer. The alkaline phosphatase level should be increased as it is a marker of bone formation.

lack of coordination of movement typified by the undershoot or overshoot of intended position with the hand, arm, leg, or eye

dysmetria

This patient's history of COPD and smoking indicate she should receive which vaccine before age 65? MOA of the vaccine?

pneumococcal polysaccharide vaccine PCV13 is a conjugate vaccine that consists of capsular polysaccharides from 13 serotypes of S. pneumoniae covalently attached to inactivated diphtheria toxin (protein). B-cells endocytose polysaccharide-protein-conjugate vaccine and display protein component of vaccine on major histocompatibility complex class II (MHCII), which can subsequently be recognized by T-cell receptor that has been activated by the same protein. This process of T-cell-dependent activation induces a strong immune response involving T-cell-mediated B-lymphocyte activation, which generates higher-affinity antibodies and memory cells. T-cell-mediated B-lymphocyte activation also enables switching, which allows for the formation of IgA mucosal antibodies that reduce bacterial colonization rates.


संबंधित स्टडी सेट्स

Nursing - Communications & Family Dynamics

View Set

Chapter 8: Consideration of Internal Control in an Information Technology Environment

View Set

The Enlightenment and its Legacy: Art of the Late 18th Through the mid-19th Century ("Enlightenment and Revolution: Head vs. Heart")

View Set

2.10 Unit Test: Voices of an Emerging Nation - Part 1

View Set

Test: Money Management (correct answers)

View Set